Sei sulla pagina 1di 158

REVISION TEST PAPER: CAP-II: ADVANCED ACCOUNTING

QUESTIONS
Hire Purchase and Instalment Transactions

1. Mr. Homlal acquired on 1 st Baisakh, 2069 air-conditioner under a Hire-Purchase agreement which
provides for 5 half-yearly installments of Rs. 6,000 each, the first installment being due on 1 st Kartik,
2069. Assuming that the applicable rate of interest is 10% p.a. Calculate the cash value of the asset.
All working should form part of the answer.

2. Nepal House sells computers on hire purchase basis at cost plus 25%. Terms of sales are Rs. 10,000
as down payment and 8 monthly installments of Rs. 5,000 for each computer. From the following
particulars prepare Hire Purchase Trading Account for the year 2070.

As on 1 st Baisakh, 2070 last installment on 30 computers was outstanding as these were not due up
to the end of the previous year.

During 2070 the firm sold 240 computers. As on 31 st Chaitra, 2070 the position of installments
outstanding were as under:

Installments due but not collected:

2 installments on 2 computers and last installment on 6 computers

Installments not yet due:

8 installments on 50 computers, 6 installments on 30 and last installment on 20 computers

Two computers on which 6 installments were due and one installment not yet due on 31.12.2070 had
to be repossessed. Repossessed stock is valued at 50% of cost. All other installments have been
received.

Accounting for Branches

3. A company has three branches at Thimi, Kapan and Gwarko. The Head Office at Newroad purchases
goods and sends them to branches, to be sold at a uniform percentage of profit on cost. The following
particular are made available to you to enable you to prepare a combined Trading Account for the
year ended 31 st Ashadh, 2071.
Newroad Thimi Kapan Gwarko
Rs. Rs. Rs. Rs.
Stock on 1st Shrawan, 2070 54,000 16,000 12,500 10,000
Purchases in the year 274,000 - - -
Sales - 180,000 120,000 100,000
Stock on 31st Ashadh, 2071 28,000 6,000 5,000 2,500
Branch Accounts on 1st Shrawan, 2070:
Thimi 15,000
Kapan 32,000
Gwarko 4,000
Remittances from Branch 320,000 150,000 100,000 70,000
Newroad Office invoices goods to the branches at fixed sales prices but maintains branch accounts in its
ledgers at cost price.

Departments

4. Booming Limited has three departments. They are Alpha, Beta and Gamma. The profits of these
departments are Rs. 30,000, Rs. 40,000 and Rs. 17,400 respectively. (Before charging manager's
commission and Inter departmental transfers).
Departments Alpha transfers its goods @ 20% profit on cost to other departments while Beta transfers its
goods @ 10% profit on cost. Departments Gamma transfers its goods at cost to other departments.
However, respective department's original goods are only transferred.
On scrutiny of records you find,
(i) Purchase made for Alpha Dept. Rs. 10,000 has been debited in Beta Dept. Account.
RTP-CAP II –-2014-December@ICAN Page 1 of 158
(ii) Goods sent on 'Sale or return basis' by Beta Dept. @ 120% have been recorded as regular Sale at Rs.
8,400.
(iii) General expenses amounting to Rs. 2,100 have been excessively charged in Gamma Dept. instead of
Beta Dept.
(iv) The following transfers were made:
Dept. Alpha To, Beta 24,000 12,000 still in closing stock
To, Gamma 3,600
Dept. Beta To, Gamma 11,000 4,400 still in closing stock
Dept. Gamma To, Alpha 7,700 3,000 still in closing stock
(v) Commission payable to the Manager @ 10% on correct overall company profit after charging such
commission.
Find correct net profit of the company and the commission payable to the General Manager.

Insurance Claim

5. A trader intends to take a loss of profit policy with indemnity period of 6 months however he could
not decide the policy amount. From the following details, suggest the policy amount.
Turnover in last financial year Rs. 450,000
Standing charges in last financial year Rs. 90,000
Net profit earned in last year was 10% of turnover and the same trend expected in subsequent year.
Increase in turnover expected 25%
To achieve additional sales, trader has to incur additional expenditure of Rs. 31,250.

Investment Accounts
6. On 1.4.2070, Mrs. Sabina Thapa purchased 1,000 equity shares of Rs. 100 each in SBB Ltd. @ Rs.
120 each from a Broker, who charged 1% brokerage. She incurred 50 paisa per Rs. 100 as cost of
shares transfer stamps. On 31.1.2071, 50% Bonus share was declared. Before and after the record
date of bonus shares, the shares were quoted at Rs. 155 per share and Rs. 90 per share respectively.
On 28.2.2071 Mrs. Thapa sold bonus shares through a Broker, who charged 1% brokerage.

She held the shares as Current assets. Show the Investment Account in the books of Mrs. Thapa on
31.3.2071 when the market value per share was Rs. 79.

Issue of Shares
7. Everest Co. Ltd. issued 200,000 shares of Rs. 100 each at a premium of Rs. 20 per share payable as
follows:
on application Rs. 20
on allotment Rs. 50 (including premium)
on First/ call Rs. 30
on second and final call Rs. 20
Applications were received for 300,000 shares and pro-rata allotment was made to applicants of 240,000
shares. Amount excess received on application was employed on account of sum due on allotment as part
of share capital. Mr. Subash, to whom, 4,000 shares were allotted, failed to pay the allotment money and
on his subsequent failure to pay the first call, his shares were forfeited and Mr. Dhiraj, the holder of 6,000
shares failed to pay to calls and his shares were forfeited after the second call. Of the forfeited shares,
8,000 shares were reissued to Mr. Gopal at a discount of 10%, the whole of Dhiraj's forfeited shares being
reissued. Pass necessary journal entries in the books of Everest Co. Ltd.

Underwriting of Shares and Debentures


8. Kathmandu Ltd. came out with an issue of 450,000 equity shares of Rs. 100 each at a premium of Rs.
20 per share. The promoters took 20% of the issue and the balance was offered to the public. The
issue was equally underwritten by A & Co; B & Co. and C & Co.
Each underwriter took firm underwriting of 10,000 shares each. Subscriptions for 310,000 equity
shares were received with marked forms for the underwriters as given below:
A & Co. 72,500 shares
B & Co. 84,000 shares
C & Co. 131,000 shares
Total 287,500 shares
The underwriters are eligible for a commission of 5% on face value of shares. The entire amount
towards shares subscription has to be paid along with application. You are required to:
(a) Prepare the statement showing the underwriters‘ liability (number of shares)
(b) Compute the amounts payable or due to underwriters; and
RTP-CAP II –-2014-December@ICAN Page 2 of 158
(c) Pass necessary journal entries in the books of Kathmandu Ltd. relating to underwriting.

Liquidator’s Final Statement


9. The following is the Balance Sheet of Himchuli Co. Limited as at 31 st Ashadh, 2071:
Liabilities Rs. Assets Rs.
Share Capital: Fixed Assets:
2,000 Equity Shares of Rs. 100 Land & Buildings 400,000
each Rs. 75 per share paid up 150,000 Plant and Machineries 380,000
6,000 equity shares of Rs. 100 Current Assets:
each Rs. 60 per share paid up 360,000 Stock at Cost 110,000
2,000 10% Preference Share of Cash at Bank 60,000
Rs. 100 each fully paid up 200,000 Profit and Loss A/c 240,000
10% Debentures (having a floating Sundry Debtors 220,000
charge on all assets) 200,000
Interest accrued on Debentures
(also secured as above) 10,000
Sundry Creditors 490,000
1,410,000 1,410,000
On that date, the company went into Voluntary Liquidation. The dividends on preference shares were in
arrear for the last two years. Sundry Creditors include a loan of Rs. 90,000 on mortgage of Land and
Buildings. The assets realized were as under:-
Rs.
Land and Buildings 340,000
Plant & Machineries 360,000
Stock 120,000
Sundry Debtors 160,000
Interest accrued on loan on mortgage of buildings upto the date of payment amounted to Rs. 10,000.
The expenses of Liquidation amounted to Rs. 4,600. The Liquidator is entitled to a remuneration of
3% on all the assets realized (except cash at bank) and 2% on the amounts distributed among equity
shareholders. Sundry creditor included preferential creditors Rs. 30,000. All payments were made on
31st Ashwin 2071. Prepare the liquidator‘s final statement.

Business Combination
10. The following is the Balance Sheet of Blue Star Ltd. as at 31 st Ashadh, 2071:
Liabilities Rs. Assets Rs.
8,000 equity shares of Rs.100 each 800,000 Building 340,000
10% debentures 400,000 Machinery 640,000
Loan from A 160,000 Stock 220,000
Creditors 320,000 Debtors 260,000
General Reserve 80,000 Bank 136,000
Goodwill 130,000
Deferred Revenue Exp. 34,000

1,760,000 1,760,000

Big Star Ltd. agreed to absorb Blue Star Ltd. on the following terms and conditions:
(1) Big Star Ltd. would take over all Assets, except bank balance at their book values less 10%.
Goodwill is to be valued at 4 year‘s purchase of super profits, assuming that the normal rate of
return be 8% on the combined amount of share capital and general reserve.
(2) Big Star Ltd. is to take over creditors at book value.
(3) The purchase consideration is to be paid in cash to the extent of Rs. 600,000 and the balance in
fully paid equity shares of Rs.100 each at Rs.125 per share.
The average profit is Rs. 124,400. The liquidation expenses amounted to Rs. 16,000 to be
borne by Big Star Ltd. Blue Star Ltd. had purchased prior to 31 st Ashadh, 2071 goods costing
Rs. 120,000 from Big Star Ltd. for Rs. 160,000. Rs. 100,000 worth of goods is still in stock of
Blue Star Ltd. on 31 st Ashadh, 2071. Creditors of Blue Star Ltd. include Rs.40,000 still due to
Big Star Ltd.
Show the necessary Ledger Accounts to close the books of Blue Star Ltd. and prepare the Balance
Sheet (extract) of Big Star Ltd. as at 1st Shrawan, 2071 after the takeover.

Internal Reconstruction
11. The following is the Balance Sheet of Pokhara Light Ltd. as on 31.3.2071:
RTP-CAP II –-2014-December@ICAN Page 3 of 158
Liabilities Rs. Assets Rs.
Equity shares of Rs.100 each 10,000,000 Fixed assets 12,500,000
12% cumulative preference shares of 5,000,000 Investments (Market value 1,000,000
Rs.100 each Rs.950,000)
10% debentures of Rs.100 each 4,000,000 Current assets 10,000,000
Sundry creditors 5,000,000 P & L A/c 400,000
Provision for taxation 100,000 Preliminary expenses 200,000
24,100,000 24,100,000
The following scheme of reorganization is sanctioned by the AGM and Court:
(i) All the existing equity shares are reduced to Rs.40 each.
(ii) All preference shares are reduced to Rs.60 each.
(iii) The rate of interest on debentures is increased to 12%. The debenture holders surrender
their existing debentures of Rs.100 each and exchange the same for fresh debentures of Rs.
70 each for every debenture held by them.
(iv) One of the creditors of the company to whom the company owes Rs. 2,000,000 decides to
forgo 40% of his claim. He is allotted 30,000 equity shares of Rs.40 each in full satisfaction
of his claim.
(v) Fixed assets are to be written down by 30%.
(vi) Current assets are to be revalued at Rs. 4,500,000.
(vii) The taxation liability of the company is settled at Rs.150,000.
(viiii) Investments to be brought to their market value.
(ix) It is decided to write off the fictitious assets.
Pass Journal entries and show the Balance sheet of the company after giving effect to the above.

Incomplete Records
12. The following is the Balance Sheet of Pathibhara Enterprises on 31 st Ashadh, 2070 :
Rs. Rs.
Capital 1,000,000 Fixed Assets 400,000
Creditors (Trade) 140,000 Stock 300,000
Profit & Loss A/c 60,000 Debtors 150,000
Cash & Bank 350,000
1,200,000 1,200,000
st
The management estimates the purchases and sales for the year ended 31 Ashadh, 2071 as under:
upto 31.2.2071 Ashadh 2071
Rs. Rs.
Purchases 1,410,000 110,000
Sales 1,920,000 200,000
It was decided to invest Rs. 100,000 in purchases of fixed assets, which are depreciated @ 10% on cost.
The time lag for payment to Trade Creditors for purchase and receipt from sales is one month. The
business earns a gross profit of 30% on turnover. The expenses against gross profit amount to 10% of
the turnover. The amount of depreciation is not included in these expenses.
Draft a Balance Sheet as at 31st Ashadh, 2071 assuming that creditors are all Trade Creditors for
purchases and debtors for sales and there is no other item of current assets and liabilities apart from
stock and cash and bank balances.

Accounting for Banks


13. On 31st Ashadh, 2070, Nepal Bank Ltd. had a balance of Rs. 9 crores in ―Discount Charge Received
in Advance‖ account. During the year ended 31 st Ashadh, 2071, Nepal Bank Ltd. discounted bills of
exchange of Rs. 4,000 crores charging interest at 18% per annum the average period of discount
being for 73 days. Of these, bills of exchange of Rs. 600 crores were due for realization from the
customers after 31 st Ashadh, 2071, the average period outstanding after 31st Ashadh, 2071, being
36.5 days.
Nepal Bank Ltd. asks you to pass journal entries and show the ledger accounts pertaining to:
(i) Discounting of bills of exchange and
(ii) Discount Charge Received in Advance

14. From the following information calculate Core capital ratio and total capital adequacy ratio of BIG
Bank Ltd. and suggest management about the compliance of the same:
In lakh
RTP-CAP II –-2014-December@ICAN Page 4 of 158
Paid up Capital 20,000
General Reserve Fund 377
Retained Earnings 308
Profit for current year 1,945
General Loan Loss Provision 1,215
Investment Adjustment Reserve 22
Loan Given to Relatives of Staffs 37
Risk weighted Exposure for Credit Risk 213,546
Risk weighted Exposure for Operational Risk 4,235
Risk Weighted Exposure for Market Risk 1,618

Accounting for Electricity Company


15. Butwal Power Co. Ltd. Company laid down a Transmission Line at a cost of Rs. 1,600,000. Some
years later the company laid down an auxiliary Transmission Line for one fourth of the length of the
old Transmission Line at a cost of Rs. 600,000. It also replaced the rest of the length o f the old
Transmission Line at a cost of Rs. 1,800,000 the cost of material and labour having gone up by
15%. Sale of old materials realised Rs. 40,000. Old materials valued at Rs. 40,000 were used in
renewal and those valued at Rs. 60,000 were used in auxiliary Transmission Line.
Pass necessary Journal Entries for recording the above transactions. Show workings.

Cash Flow Statement


16. The following are the changes in the account balances taken from the balance sheet of Nepal Ltd. as
the beginning and end of the year.
Particular Debit Credit
8% Debentures 150,000
Debenture Discount 3,000
Plant and Machinery at cost 180,000
Depreciation on Plant and Machinery 43,200
Trade Receivables 150,000
Inventory including work in progress 115,500
Trade payables 35,400
Net profit for the year 229,500
Dividend paid in respect of earlier years 90,000
Provision for Doubtful Debts 9,900
Trade Investments at cost 141,000
Bank 211,500
Total 679,500 679,500

You are informed that:


 During the year plant costing Rs. 54,000 against which depreciation provision of Rs. 40,500 was
lying was sold for Rs. 21,000.
 During the middle of the year, Rs. 150,000 debentures were issued for cash at a discount of Rs.
3,000.
 The net profit for the year was after crediting the profit on sale of plant and charging debenture
interest.
Prepare a cash flow statement which will explain why bank borrowing has increased by Rs. 211,500
during the year end, ignore taxation.

Profit or Loss Pre and Post Incorporation


17. From the following information, calculate the ratio of sales in each case separately.
(a) (i) Date of acquisition – 1st Shrawan 2070; date of incorporation – 1st Kartik, 2070 and date of
closing the books of accounts – 31st Ashadh, every year.
(ii) The sales for the year ending on 31st Ashadh, 2071 were Rs. 2,400,000 of which Rs. 480,000
goods were sold during the six months of the accounting period.

(b) (i) The accounts were made up to 31st Chaitra 2070. The company was incorporated on 1 st Bhadra
2070 to take over a business from the preceding 1st Baisakh.
(ii) Total Sales for the year were Rs. 1,200,000. It is ascertained that the sales for Falgun and
Chaitra are one and half times the average of those for the year, whilst those for Jestha and
Shrawan are only half the average.

(c) (i) Hello Ltd. was incorporated on 1st Kartik, 2070 to take the existing business of X Ltd. from 1 st
Shrawan 2070. Date of closing the books of accounts – 31st Ashadh, 2071.

RTP-CAP II –-2014-December@ICAN Page 5 of 158


(ii) Monthly sales in Shrawan 2070, Jestha 2071 and Ashadh 2071 were double the average
monthly sales for remaining months of the year.

Accounting for Partnership


18. A, B and C were partners of a firm sharing profits and losses in the ratio of 3 : 4 : 3. The Balance
Sheet of the firm, as at 31 st Ashadh, 2070 was as under:
Liabilities Rs. Assets Rs.

Capital Accounts: Fixed Assets 100,000

A 48,000 Current Assets:

B 64,000 Stock 30,000

C 48,000 160,000 Debtors 60,000

Reserve 20,000 Cash and Bank 30,000

Creditors 40,000

220,000 220,000

The firm had taken a Joint Life Policy for Rs. 100,000; the premium periodically paid was charged to
Income Statement. Partner C died on 30 th Poush, 2070. It was agreed between the remaining
partners and the legal representatives of C that:

(i) Goodwill of the firm will be taken at Rs. 60,000.


(ii) Fixed Assets will be written down by Rs. 20,000.
(iii) In lieu of profits, C should be paid at the rate of 25% per annum on his capital as on 31 st
Ashadh, 2070.
Policy money was received and the legal representatives were paid off. The profits for the year
ended 31st Ashadh, 2071, after charging depreciation of Rs. 10,000 (depreciation upto 30 th Poush
was agreed to be Rs. 6,000) were Rs. 48,000. Legal representative claimed proportionate profit for
remaining period after death.

Partners‘ Drawings Accounts showed balances as under:

A Rs. 18,000 (drawn evenly over the year)


B Rs. 24,000 (drawn evenly over the year)
C (up-to-date of death) Rs. 20,000
On the basis of the above figures, please indicate the entitlement of the legal representatives of C,
assuming that they had not been paid anything other than the share in the Joint Life Policy.

19. A, B and C are partners of the firm ‗KK Enterprises‘ for the past 5 years. The partners decided to
dissolve the firm consequent to insolvency of partner C in Magh, 2070. The Balance Sheet of the
firm as on 31.10.2070 is furnished below. They share profits and losses equally:
Liabilities Rs. Assets Rs.
Capital Accounts: Land and Building 500,000
A 450,000 Plant and Machinery 200,000
B 450,000 Furniture and Fittings 50,000
C 200,000 Stock in Trade 300,000
General Reserve 210,000 Debtors 500,000
Creditors 290,000 Cash at Hand/Bank 50,000
1,600,000 1,600,000
The partners A and B decided to form a new firm ‗RR Traders‘ and takeover all the assets and
liabilities of the firm at values given below:
Land and Building Rs. 350,000
Plant and Machinery Rs. 150,000
Furniture and Fittings Rs. 20,000
Stock in trade Rs. 200,000
Debtors include Rs. 300,000 due from SK & Co. owned by C. (Nothing is recoverable from the
said concern). Other debtors can be recovered fully.
Prepare:
(i) Realisation account, Partners‘ capital accounts in the books of KK Enterprises; and
(ii) The Balance Sheet of RR Traders (immediately after commencement).
RTP-CAP II –-2014-December@ICAN Page 6 of 158
Nepal Accounting Standards
20. a. Ram, a construction contractor, undertakes the construction of an industrial complex. He has
separate proposals raised for each unit to be constructed in the industrial complex. Since each unit
was subject to separate negotiation, he was able to identify the costs and revenues attributable to
each unit. Should Ram treat construction of each unit as a separate construction contract?

b. Indicate any three areas in respect of which different accounting policies may be adopted by different
enterprises. Also indicate the requirements with regard to disclosure of accounting policies as per the
relevant NAS.

c. What are the conditions that have to be satisfied for recognition of revenue from sale of goods?

d. Explain ‗Recognition of Assets‘ as per Framework for the Preparation and Presentation of Financial
Statements under NAS.

e. Bikash Ltd. borrowed US $ 500,000 on 31.12.2070 which will be repaid as on 30.6.2071. Bikash Ltd.
prepares its financial statements ending on 31.3.2071. Rate of exchange between reporting currency
and foreign currency on different dates are as under:
31.12.2070 1 US $ = 93
31.3.2071 1 US $ = 95
30.6.2071 1 US $ = 98
Calculate borrowings in reporting currency to be recognised in the books on above mentioned dates
and also show journal entries for the same.
Short Notes
21. (a). Sale and lease back transaction
(b). Escalation Clause in Contracts
(c). Re – Insurance:
(d). Component of Financial Statement as per NAS
(e). Features of Farm Accounting

RTP-CAP II –-2014-December@ICAN Page 7 of 158


ANSWERS

Hire Purchase and Instalment Transactions


1. Answer:
Statement for cash value of the air-conditioner acquired on hire-purchase basis
Instalment Interest @ 5% half Principal Amount (in
Amount yearly (10% p.a.) = each instalment)
5/105 = 1/21)
(in each instalment)
Rs. Rs. Rs.
5th Instalment 6,000 286 5,714
Less: Interest – 286
5,714
Add: 4th Instalment 6,000
11,714 558 5,442
Less: Interest 558 (11,156–5,714)
11,156
Add: 3rd instalment 6,000
17,156 817 5,183
Less: Interest 817 (16,339–11,156)
16,339
Add: 2nd instalment 6,000
22,339 1,063 4,937
Less: Interest 1,063 (21,276–16,339)
21,276
Add: 1st instalment 6,000
27,276 1,299 4,701
Less: Interest 1,299 (25,977–21,276)
25,977 4,023 25,977
The cash purchase price of air-conditioner is Rs. 25,977.

2. Answer:
In the books of Nepal House
Hire Purchase Trading Account
for the year ended on 31 st Chaitra, 2070
Rs. Rs.
To Hire Purchase Stock By Cash (W.N.1) 90,30,000
(30×Rs. 5,000) 1,50,000 By Stock Reserve
30,000
To Goods Sold on Hire (Rs. 150,000×20%)
Purchase (240×Rs. 50,000) 1,20,00,000
By Goods Sold on Hire Purchase 24,00,000
(Rs. 12,000,000×20%)
To Stock Reserve By Goods Repossessed (2×Rs.
(Rs. 3,000,000 × 20%) 6,00,000 40,000×50%) 40,000
To Profit & Loss A/c By Installments Due
(Transfer of Profit) 18,00,000 [(2×2+1×6)×Rs. 5,000] 50,000
By Hire Purchase
Stock[8×50+6×30+1×20)×Rs. 30,00,000
5,000]
14,550,000 14,550,000

Working Notes:
Rs.
(1) Cash collected:
Cash down payment (240 × Rs. 10,000) 2,400,000
Add: Installments collected:
Last installments on 30 computers outstanding on 1.1.2070 150,000
Installments due and collected on 240 computers sold
RTP-CAP II –-2014-December@ICAN Page 8 of 158
during the year :
Total instalments on 240 computers
(8 × 240 × Rs. 5,000) 9,600,000
Less : Instalments due but not collected
[(2 × 2 + 1 × 6 + 6 × 2) × Rs. 5,000] 110,000
Instalments not due on 31.12.99
[(8 × 50 + 6 × 30 + 1 × 20 +
1 × 2) × Rs. 5,000] 3,010,000 3,120,000
Cash collected 9,030,000

Accounting for Branches

3. Answer:
Combined Trading Account
For the year ended 31st Ashadh 2071

Particular Newroad Thimi Kapan Gwarko Total


To Opening stock at cost 54,000 12,800 10,000 8,000 84,800
(Working note iv)
To Purchases 274,000 - - -
To Goods received from HO at - 136,000 90,000 74,000 300,000
cost (balancing figure)
To Gross profit @ 20% on Sales 36,000 24,000 20,000 80,000
Total 328,000 184,800 124,000 102,000 738,800
By Sales - 180,000 120,000 100,000 400,000
By Goods sent to branch at cost 300,000 - - - 300,000
By Closing Stock at cost 28,000 4,800 4,000 2,000 38,800
(working note iv)
Total 328,000 184,800 124,000 102,000 738,800

Working Notes:
i). Calculation of cost of Stock sent to all branches
Opening stock at HO 54,000
Add: Total Purchases at HO 274,000
Less: Closing stock at HO (28,000)
Goods sent to branches at cost 300,000

ii). Calculation of invoice of goods received from HO by all branches


Closing stock at branch level (total) 13,500
Add: Total sales 400,000
Less: Opening stock at branch level (total) (38,500)
Goods received from HO (total) 375,000

iii) Calculation of profit margin on goods sent to the branches


Goods sent to branches at Invoice Price 375,000
Less: Goods sent to branches at cost price (300,000)
Profit 75,000
Rate of profit margin is 20% of Invoice price or 25% of cost price

iv). Valuation of stock at cost price


Opening Stock Closing stock
Thimi 16,000x80%= 12,800 6,000x80%=4,800
Kapan 12,500x80%=10,000 5,000x80%=4,000
Gwarko 10,000x80%=8,000 2,500x80%=2,000

Departments
4. Answer:
Profit & Loss Adjustment Account
Particular Alpha Beta Gamma Particular Alpha Beta Gamma
To Purchases 10,000 - - By Given Profit 30,000 40,000 17,400

RTP-CAP II –-2014-December@ICAN Page 9 of 158


To sales 8,400 By Purchase of 10,000
Alpha
To Expenses 2,100 By stock on Sale or 7,000
approval basis
To Goods received 7,700 24,000 14,600 By Expenses 2,100
To Stock Reserve By Goods Transfer 27,600 11,000 7,700
To Net Profit 42,900 45,500 17,000 By Closing stock 3,000 12,000 4,400
Total 60,600 80,000 31,600 Total 60,600 80,000 31,600

General Profit & Loss Account

To Stock Reserve By Net Profit


Beta 2,000 Alpha 42,900
Gamma 400 Beta 45,500
Gamma 17,000
To Manager‘s Commission 9,364

To Net profit 93,636


Total 105,400 Total 105,400

Insurance Claim

5. Answer:
a. Turnover in last financial year 450,000
Add: 25% increase 112,500
Total turnover 562,500

b. Net profit = 10% of 562,500 = 56,250


c. Total Standing Expenses = 90,000+31,250=121,250
d. Gross profit = Net profit + Standing Charges = 56,250+121,250=177,500
Hence, policy amount should be Rs. 177,500.

Investment Accounts

6. Answer:
In the books of Mrs. Sabina Thapa
Investment Account (Share)
for the year ended 31.3.2071
Dr. Cr.
Date Particulars Nominal Cost Date Nominal Cost
Value Value
(Rs.) (Rs.) (Rs.) (Rs.)
1.4.70 To Bank A/c 1,00,000 1,21,800 28.2.71 By Bank A/c 50,000 44,550

31.1.71 To Bonus shares 50,000  31.3.71 By P & L A/c - 2,200


28.2.71 To P & L A/c  3,950 31.3.71 By Balance c/d 1,00,000 79,000

1,50,000 1,25,750 1,50,000 1,25,750


Working Notes:
(i) Cost of shares purchased on 1.4.2070 = 1,000  Rs. 120 + 1% of Rs. 1,20,000 + ½% of Rs.
1,20,000 = Rs. 1,21,800
(ii) Sale proceeds of shares sold on 28.2.2071 = 500  Rs. 90 – 1% of Rs. 45,000 = Rs. 44,550.
(iii) Profit on sale of bonus shares on 28.2.2071
= Sales proceeds – Average cost
Sales proceeds = Rs. 44,550
Average cost = Rs. (1,21,800  50,000)/1,50,000
= Rs. 40,600
Profit = Rs. 44,550 – Rs. 40,600 = Rs. 3,950.
(iv) Valuation of shares on 31.3.2071
Cost = (Rs. 1,21,800 × 1,00,000)/1,50,000 = Rs. 81,200
Market Value = 1,000 shares × Rs. 79 = Rs. 79,000
Closing balance has been valued at Rs. 79,000 being the market value is lower than cost price.

Issue of Shares

RTP-CAP II –-2014-December@ICAN Page 10 of 158


7. Answer:
Books of Everest Co. Ltd.
Journal Entries
Bank A/c Dr. 6,000,000
To Share Application A/c 6,000,000
(Being application amount received)
Share Application A/c Dr. 6,000,000
To Share Capital A/c 4,000,000
To Bank A/c 1,200,000
To Share Allotment A/c 800,000
(Being application money transferred to share capital,
refunded and excess transferred to allotment )
Share Allotment a/c Dr. 10,000,000
To Share Capital A/c 6,000,000
To Share Premium A/c 4,000,000
(Being allotment amount due)
Bank A/c Dr. 9,016,000
Calls in Arrear A/c Dr. 184,000
To Share Allotment A/c 9,200,000
(Being allotment money received except from Mr. Subash)
Share First Call A/c Dr. 6,000,000
To Share Capital A/c 6,000,000
(Being first call amount due)
Bank A/c Dr. 5,700,000
Calls in Arrear A/c Dr. 300,000
To Share First Call A/c 6,000,000
(Being first call money received except from Mr. Subash and
Mr. Dhiraj)
Share Capital A/c Dr. 320,000
Share Premium a/c Dr. 80,000
To Calls in Arrear A/c 304,000
To Share Forfeiture A/c 96,000
(Being forfeiture of shares of Mr. Subash)
Share Final Call A/c Dr. 3,920,000
To Share Capital A/c 3,920,000
(Being final call amount due)
Bank A/c Dr. 3,800,000
Calls in Arrear A/c Dr. 120,000
To Share Final Call A/c 3,920,000
(Being final call money received except from Mr. Dhiraj)
Share Capital A/c Dr. 600,000
To Calls in Arrear A/c 300,000
To Share Forfeiture A/c 300,000
(Being forfeiture of shares of Mr. Dhiraj)
Bank A/c Dr. 720,000
Share Forfeiture A/c Dr. 80,000
To Share Capital A/c 800,000
(Being re-issue of shares @ 90 to Mr. Gopal as fully paid up)
Share Forfeiture a/c Dr. 268,000
To Capital Reserve A/c 268,000
(Being forfeiture amount transferred to capital Reserve A/c)

Working Note:

1. No. of Shares applied by Mr. Subash = (240,000/200,000) x 4,000 = 4,800


2. Amount paid my Mr. Subash at the time of application = 4,800 x 20 = 96,000
3. Forfeiture amount available for use = full of Mr. Dhiraj (300,000) + half of Mr. Subash (96,000/2)
=348,000
4. Amount Transferred to capital reserve = 348,000 – 80,000 = 266,000

Underwriting of Shares and Debentures

8. Answer:

RTP-CAP II –-2014-December@ICAN Page 11 of 158


(a) Statement showing the underwriters’ liability (No. of shares)
A & Co. B & Co. C & Co.
Gross Liability 120,000 120,000 120,000
Less: Firm underwriting 10,000 10,000 10,000
110,000 110,000 110,000
Less: Marked applications 72,500 84,000 131,000
37,500 26,000 (21,000)
Less: Unmarked applications distributed to A &
Co. and B & Co. in equal ratio (11,250) (11,250) Nil
26,250 14,750 (21,000)
Less: Surplus of C & Co. distributed to A & Co.
and B & Co. in equal ratio (10,500) (10,500) 21,000
Net liability (excluding firm underwriting) 15,750 4,250 Nil
Add: Firm underwriting 10,000 10,000 10,000
Total liability (No. of shares) 25,750 14,250 10,000

(b) Computation of amounts payable by underwriters


Liability towards shares to be subscribed @ 120 per 3,090,000 1,710,000 1,200,000
share
Less: Commission (5% on 1.2 lakhs shares @ 100
each) 600,000 600,000 600,000
Net amount to be paid by the underwriters 2,490,000 1,110,000 600,000

(c) In the Books of Kathmandu Ltd.


Journal Entries
Particulars Dr. Cr.
Rs. Rs.
Underwriting commission A/c Dr. 1,800,000
To A & Co. A/c 600,000
To B & Co. A/c 600,000
To C & Co. A/c 600,000
(Being underwriting commission on the shares underwritten)
A & Co. A/c Dr. 3,090,000
B & Co. A/c Dr. 1,710,000
C & Co. A/c Dr. 1,200,000
To Equity share capital A/c 5,000,000
To Share premium A/c 1,000,000
(Being shares including firm underwritten shares allotted to
underwriters)
Bank A/c Dr. 4,200,000
To A & Co. A/c 2,490,000
To B & Co. A/c 1,110,000
To C & Co. A/c 600,000
(Being the amount received towards shares allotted to
underwriters less underwriting commission due to them)

Liquidator’s Final Statement

9. Answer:
Liquidator’s Final Statement
Receipts Rs. Rs. Payments Rs. Rs.
Cash at Bank 60,000 Liquidation expenses 4,600
Assets realised:
Sundry Debtors 160,000 Liquidator‘s remuneration (W.N. 1) 30,400
Stock 120,000 Debenture holders:
Plant & Machinery 360,000 640,000 10% debentures 200,000
Surplus from Land & Interest accrued (W.N.2) 15,000 215,000
Buildings: Preferential creditors 30.000

RTP-CAP II –-2014-December@ICAN Page 12 of 158


Amount realised 340,000 Unsecured creditors 370,000
Less: Secured Preference shareholders:
Creditors 100,000 240,000 10% Preference Share
Capital 200,000
Arrear dividend 40,000 240,000
Equity Shareholders
(W.N. 3) :
Rs. 17.50 per share
on 2,000 shares 35,000
Rs. 2.50 per share
on 6,000 shares 15,000 50,000
940,000 940,000

Working Notes:
(1) Liquidator’s remuneration: Rs.
3% on Assets realised (3% of Rs. 980,000) 29,400
2% of the amounts distributed among Equity Shareholders
(2/102 × Rs. 51,000) 1,000
30,400
(2) Interest accrued on 10% debentures
Interest accrued as on 31.3.2071 10,000
Interest accrued upto the date of payment
(upto 31st Ashwin, 2071) 5,000
15,000
(3) Amount payable to Equity Shareholders
Equity Share Capital 510,000
Less: Surplus available for Equity Shareholders 50,000
Loss to be borne by them 460,000
Loss per Equity share (Rs. 460,000/8,000) 57.50
Amount payable to Equity shareholders:
Each Equity share of Rs. 75 paid up 17.50
Each Equity share of Rs. 60 paid up 2.50

Business Combination

10. Answer:
Books of Blue Star Limited
Realization Account
Rs. Rs.
To Building 3,40,000 By Creditors 3,20,000
To Machinery 6,40,000 By B Ltd. 12,10,000
To Stock 2,20,000 By Equity Shareholders A/c 60,000
(Loss)
To Debtors 2,60,000
To Goodwill 1,30,000
1,590,000 1,590,000

Bank Account
To Balance b/d 136,000 By 10% debentures 400,000
To Big Star Ltd. 600,000 By Loan from A 160,000
By Equity shareholders A/c 176,000
736,000 736,000

Big Star Ltd. Account


To Realisation A/c 1,210,000 By Bank A/c 600,000
By Equity Share holders A/c
(4,880 shares at Rs.125 each in Big
Star Ltd.) 610,000
1,210,000 1,210,000

Equity Share Holders Account


To Realisation A/c 60,000 By Equity Share Capital 800,000
RTP-CAP II –-2014-December@ICAN Page 13 of 158
To Deferred Revenue Exp. 34,000 By General Reserve 80,000
To Equity shares in Big Star Ltd. 610,000
To Bank A/c 176,000
880,000 880,000

Big Star Ltd.


Balance Sheet as on 1 st Shrawan, 2071 (An extract)
Liabilities Rs. Assets Rs.
4880 Equity shares of Rs.100 each 488,000 Goodwill 232,000
(Shares have been issued for Building 306,000
consideration other than cash)
Securities Premium 122,000 Machine 576,000
Profit and Loss A/c ….
Less: unrealized profit 15,000 …..
Creditors (320,000 - 40,000) 280,000 Stock (198,000 -15,000) 183,000
Bank Overdraft 616,000 Debtors (260,000 – 40,000) 220,000
Less: Provision for bad debts 26,000 194,000

Working Notes:
1. Valuation of Goodwill Rs.
Average profit 124,400
Less: 8% of Rs. 880,000 70,400
Super profit 54,000
Value of Goodwill = 54,000 x 4 216,000
2. Net Assets for purchase consideration
Goodwill as valued in W.N.1 216,000
Building 306,000
Machinery 576,000
Stock 198,000
Debtors 260,000
Total Assets 1,556,000
Less: Creditors 320,000
Provision for bad debts 26,000 346,000
Net Assets 1,210,000

Out of this Rs. 600,000 is to be paid in cash and remaining i.e., (1,210,000–600,000) Rs. 610,000 in
shares of Rs. 125/-. Thus, the number of shares to be allotted 610,000/125 = 4,880 shares.
3. Unrealized Profit on Stock Rs.
The stock of Blue Star Ltd. includes goods worth Rs. 100,000 which was sold by
40,000
Big Star Ltd. on profit. Unrealized profit on this stock will be 1,00,000 25,000
1,60,000
As Big Star Ltd. purchased assets of Blue Star Ltd. at a price 10% less than the (10,000)
book value, 10% need to be adjusted from the stock i.e., 10% of Rs.100,000.
Amount of unrealized profit 15,000

4. Liquidation expenses borne by the Big Star Ltd. so that should be debited to Goodwill Account.

Internal Reconstruction

11. Answer:

Journal Entries
in the books of Pokhara Light Ltd.

RTP-CAP II –-2014-December@ICAN Page 14 of 158


Rs.
Rs.
(i) Equity Share Capital (Rs.100) A/c Dr. 1,00,00,000
To Equity Share Capital (Rs.40) A/c 40,00,000
To Reconstruction A/c 60,00,000
(Being conversion of equity share capital of Rs.100 each into Rs.40
each as per reconstruction scheme)
(ii) 12% Cumulative Preference Share capital (Rs.100) A/c Dr. 50,00,000
To 12% Cumulative Preference Share Capital (Rs.60) A/c 30,00,000
To Reconstruction A/c 20,00,000
(Being conversion of 12% cumulative preference share capital of
Rs.100 each into Rs.60 each as per reconstruction scheme)
(iii) 10% Debentures A/c Dr. 40,00,000
To 12% Debentures A/c 28,00,000
To Reconstruction A/c 12,00,000
(Being 12% debentures issued to 10% debenture-holders for 70% of
their claims. The balance transferred to capital reduction account as per
reconstruction scheme)
(iv) Sundry Creditors A/c Dr. 20,00,000
To Equity Share Capital A/c 12,00,000
To Reconstruction A/c 8,00,000
(Being a creditor of Rs.20,00,000 agreed to surrender his claim by 40%
and was allotted 30,000 equity shares of Rs.40 each in full settlement
of his dues as per reconstruction scheme)
(v) Provision for Taxation A/c Dr. 1,00,000
Reconstruction A/c Dr. 50,000
To Current Assets (Bank A/c) 1,50,000
(Being conversion of the provision for taxation into liability for taxation
for settlement of the amount due)
(vi) Reconstruction A/c Dr. 99,50,000
To P & L A/c 4,00,000
To Preliminary Expenses A/c 2,00,000
To Fixed Assets A/c 37,50,000
To Current Assets A/c 55,00,000
To Investments A/c 50,000
To Capital Reserve A/c 50,000
(Being amount of Reconstruction utilized in writing off P & L A/c (Dr.)
Balance, Preliminary Expenses, Fixed Assets, Current Assets,
Investments and the Balance transferred to Capital Reserve)

Balance Sheet of Pokhara Light Ltd. (and reduced)


as on 31.3.2071
Liabilities Rs. Assets Rs.
Issued, subscribed and paid up capital: Fixed Assets 87,50,000
1,30,000 equity shares of Rs.40 each 52,00,000 (1,25,00,000 – 37,50,000)
12% Cumulative Preference Shares of Investments 9,50,000
Rs. 60 each 30,00,000 (10,00,000 – 50,000)

RTP-CAP II –-2014-December@ICAN Page 15 of 158


Reserves & Surplus: Current Assets 43,50,000
Capital Reserve 50,000 (45,00,000 – 1,50,000)
Secured Loan:
12% Debentures 28,00,000
Current Liabilities and Provisions:
Sundry Creditors:
(50,00,000 – 20,00,000) 30,00,000

1,40,50,000 1,40,50,000

Working Note:
Reconstruction Account
Rs. Rs.
To Liability for taxation A/c 50,000 By Equity share capital 60,00,000
To P & L A/c 4,00,000 By 12% Cum. preference share 20,00,000
To Preliminary expenses 2,00,000 By 10% Debentures 12,00,000
To Fixed assets 37,50,000 By Sundry creditors 8,00,000
To Current assets 55,00,000
To Investment 50,000
To Capital Reserve 50,000
(balancing figure) _________ _________
1,00,00,000 1,00,00,000

Incomplete Records

12. Answer:
Projected Balance Sheet
as on 31st Ashadh, 2071
Liabilities Rs. Assets Rs.
Capital 1,000,000 Fixed Assets 400,000
Profit & Loss Account as on Additions 100,000
1st Shrawan, 2070 60,000 500,000
Add: Profit for the year 374,000 434,000 Less: Depreciation (50,000) 450,000
Creditors (Trade) 110,000 Stock in trade 336,000
Sundry Debtors 200,000
Cash & Bank Balances 558,000
1,544,000 1,544,000
Working Notes:
Projected Trading and Profit and Loss Account
for the year ended 31st Ashadh, 2071
Rs. Rs.
To Opening Stock 300,000 By Sales 2,120,000
To Purchases 1,520,000 By Closing Stock (balancing figure) 336,000
To Gross Profit (30% on sales) 636,000
2,456,000 2,456,000
To Sundry Expenses (10% on sales) 212,000 By Gross Profit b/d 636,000
To Depreciation 50,000
To Net Profit 374,000
636,000 636,000

Cash and Bank Account for the period


1st Shrawan, 2070 to 31st Ashadh, 2071
Rs. Rs.
To Balance b/d 350,000 By Sundry Creditors 1,550,000
To Sundry Debtors 2,070,000 (Rs. 140,000 + Rs. 1,410,000)
(Rs. 150,000 + Rs. 1,920,000) By Expenses 212,000

RTP-CAP II –-2014-December@ICAN Page 16 of 158


By Fixed Assets 100,000
By Balance c/d 558,000
2,420,000 2,420,000

Note: It is assumed that the entire sales and purchases are on credit basis.

Accounting for Banks

13. Answer:
Nepal Bank Ltd.
Journal Entries
(Rupees in crores)
Dr. Cr.
Discount Charge Received in Advance A/c Dr. 9.00
To Discount on bills A/c 9.00
(Being the transfer of opening balance in Discount Charge
Received in Advance account to discount on bills account)

Bills purchased and discounted A/c Dr. 4000.00


To Discount on bills A/c 144.00
 18 73 
Rs. 4,000 crores  100  365 
To Clients A/c 3,856.00
(Being the discounting of bills of exchange during the year)

Discount on bills A/c Dr. 10.80


To Discount Charge Received in Advance A/c 10.80
(Being the unexpired portion of discount in respect
of the discounted bills of exchange carried forward)

Discount on bills A/c Dr. 142.20


To Profit and loss A/c 142.20
(Being the amount of income for the year from
discounting of bills of exchange transferred to
Profit and Loss A/c)

Ledger Accounts
(i) Discount on bills A/c
2071 Rs. 2070 Rs.
Ashadh 31 To Discount Charge Shrawan 1 By Discount Charge 9.00
Received in Advance A/c 10.80 Received in Advance A/c
To Profit and loss A/c 142.20 2070-71 By Bills purchased and
discounted A/c 144.00
153.00 153.00

(ii) Discount Charge Received in Advance A/c


2070 Rs. 2070 Rs.
Shrawan 1 To Discount on bills A/c 9.00 Shrawan 1 By Balance b/d 9.00
2071 2071
Ashadh 31 To Balance c/d 10.80 Ashadh 31 By Discount on bills A/c 10.80
19.80 19.80

14. Answer:
a. Calculation of Total Risk Weighted Assets = 213,546 + 4,235 + 1,618 = 219,399

b. Calculation of Core Capital


Particular Amount
Paid up Capital 20,000
General Reserve Fund 377
Retained Earnings 308
Profit for current year 1,945

RTP-CAP II –-2014-December@ICAN Page 17 of 158


Less: Loan Given to Relatives of Staffs (37)
Total core capital 22,593

c. Supplementary Capital
Particular Amount
General Loan Loss Provision 1,215
Investment Adjustment Reserve 22
Total Supplementary Capital 1,237

d. Core Capital Ratio = 22,593 X 100/ 219,399 = 10.29%


e. Total Capital Adequacy Ratio = (22,593 + 1,237) X 100/ 219,399 = 10.86%
f. Compliance with NRB Directives
Particular NRB Actual
Requirement
Core Capital Ratio 6% 10.29%
Total Capital Adequacy Ratio 10% 10.86%

Accounting for Electricity Company

15. Answer:
Books of Butwal Power Co. Ltd.
Journal Entries
Replacement A/c Dr. 1,380,000
To Bank A/c 1,380,000
(current cost of replacement of ¾ of Transmission Line
charged to replacement account)
New Transmission Line A/c Dr. 600,000
To Bank A/c 540,000
To Replacement A/c 60,000
(cost incurred on laying auxiliary Transmission Line including
old material worth Rs. 60,000)
New Transmission Line A/c Dr. 420,000
To Bank A/c 380,000
To Replacement A/c 40,000
(Additional cost of new Transmission Line capitalised including
cost of old material used Rs. 40,000)
Bank A/c Dr. 40,000
To Replacement A/c 40,000
(sale of old materials)
Profit & Loss A/c Dr. 1,240,000
To Replacement A/c 1,240,000
(net current cost of replacement transferred to PL a/c)

Working note:
Amount of additional cost to be capitalised
Cost of ¾ of old Transmission Line 1,200,000
Add: Increase in cost by 15% 180,000
1,380,000
Cost of replacement 1,800,000
Additional cost of new Transmission Line (to be capitalised) 420,000
Less: cost of old material (40,000)
Additional cash cost of replacement 380,000

Cash Flow Statement

16. Answer:
Cash Flow Statement
Particular Amount Amount
Cash Flow from Operating Activities
Net Profit before Taxation (given) 229,500
Adjustment for Depreciation (WN 2) 83,700
Debenture Interest (150,000x8%x6/12) 6,000
Provision for Doubtful Debts 9,900

RTP-CAP II –-2014-December@ICAN Page 18 of 158


Profit/gain on sale of plant (WN 1) (7,500) 92,100
Operating profit before working capital changes 321,600
Increase in Inventory (115,500)
Increase in Trade Receivables (150,000)
Increase in Trade Payables 35,400 (230,100)
Net cash flow from Operating Activities (A) 91,500
Cash Flow from Investing Activities
Purchase of Plant & Machinery (WN 3) (234,000)
Purchase of Trade Investments (141,000)
Sale of machinery 21,000
Net cash flow from Investing Activities (B) (354,000)
Cash Flow from Financing Activities
Proceeds from issue of 8% Debentures (net) 147,000
Interest paid on 8% Debentures (6,000)
Dividends paid in respect of earlier years (90,000)
Net Cash flow from Financing Activities (C) 51,000
Net Increase/(Decrease) in Cash and Cash Equivalents (A+B+C) (211,500)

Working Notes:
1. Profit on sale of Plant = WDV at disposal – sale value
= (54,000-40,500) – 21,000
= 7,500

2. Depreciation for current year = 43,200 + 40,500 = 83,700

3. Cash flow towards assets purchase = Increase in Plant & machinery at cost + cost of plant sold
= 180,000 + 54,000 = 234,000

Profit or Loss Pre and Post Incorporation

17. Answer:

(a) Sales of first 6 months = Rs. 480,000.


Average sale of first 6 months = 480,000/6 = 80,000 per month
Pre-incorporation period consist of 3 months = Shrawan, Bhadra and Ashwin
Sales for 3 months = 80,000 x 3 = 240,000
Sales for remaining 9 months = 2,400,000 – 240,000 = 2,160,000
Therefore, the ratio of sale = pre : post = 240,000 : 2,160,000 = 1: 9

(b) Let the average of monthly sales = 1. The sales of different months can be shown as follows:

Month Bai. Jes. Ash. Shr. Bha. Ash. Kar. Mang. Poush Mag. Fal. Chai. Total
Sales 1 0.5 1 0.5 1 1 1 1 1 1 1.5 1.5 12
Date of incorporation = 1st Bhadra, 2070
Pre- incorporation period is from Baisakh to Shrawan = 1+0.5+1+0.5 = 3
Post – incorporation period is from Bhadra to Chaitra =9
The ratio of sales = 3:9 = 1: 3

(c) Let the average of monthly sales = 1. The sales of different months can be shown as follows:

Month Shr. Bha. Ash. Kar. Mang. Poush Mag. Fal. Chai. Bai. Jes. Ash. Total
Sales 2 1 1 1 1 1 1 1 1 1 2 2 15
Date of incorporation = 1st Kartik, 2070
Pre- incorporation period is from Shrawan to Ashwin = 2+1+1 = 4
Post – incorporation period is from Kartik to Ashadh = 11
The ratio of sales = 4 : 11

Accounting for Partnership

18. Answer:
Computation of entitlement of legal representatives of C
(1) Profits for the half year ended 31 st Ashadh, 2071
Rs.

RTP-CAP II –-2014-December@ICAN Page 19 of 158


Profits for the year ended 31 st Ashadh, 2071 (after depreciation) 48,000
Add: Depreciation 10,000
Profits before depreciation 58,000
Profits for the first half (assumed: evenly spread) 29,000

Less: Depreciation for the first half 6,000

Profits for the first half year (after depreciation) 23,000

Profits for the second half (i.e., 1 st Magh, 2070 to 31 st Ashadh, 2071) 29,000

Less: Depreciation for the second half 4,000

Profits for the second half year (after depreciation) 25,000


th
(2) Capital Accounts of Partners as on 30 Poush, 2070

Dr. Cr.
A B C A B C
Rs. Rs. Rs. Rs. Rs. Rs.
To Fixed Assets By Balance b/d 48,000 64,000 48,000
(loss on revaluation) 6,000 8,000 6,000 By Reserve 6,000 8,000 6,000
By Goodwill 18,000 24,000 18,000
To Drawings 9,000 12,000 20,000 By Interest
To C Executor‘s A/c 52,000 (Rs. 48,000 @ 25%
To Balance c/d 57,000 76,000 – for 6 months) — — 6,000

72,000 96,000 78,000 72,000 96,000 78,000

(3) Calculation of Proportionate profit to Legal Representative of C

Profit for the second half = 25,000


Proportionate profit to C‘s legal representative = 25,000 x 52,000/(57,000+76,000+52,000)
= 7,027

(4) Amount due to legal representative of C Rs.

Balance in C‘s Executor‘s account 52,000


Amount of profit earned out of unsettled capital [calculated in (3)] 7,027
Total Amount due 59,027

19. Answer:
In the Books of KK Enterprises
Realisation Account
Rs. Rs.
To Sundry assets: By Creditors 290,000
Land and building 500,000 By RR Traders 970,000
Plant and machinery 200,000 (W.N. 1)
Furniture and fittings 50,000 By Loss transferred to
Stock 300,000 partners‘ capital accounts
Debtors 200,000 A 110,000
Cash at hand/bank 50,000 B 110,000
To RR Traders 290,000 C 110,000 330,000
(liability taken over) ________ ________
1,590,000 1,590,000

Partners’ Capital Accounts


A B C A B C
Rs. Rs. Rs. Rs. Rs. Rs.
To Sundry 300,000 By Balance 450,000 450,000 200,000
debtors b/d
To Realisation 110,000 110,000 110,000 By General 70,000 70,000 70,000

RTP-CAP II –-2014-December@ICAN Page 20 of 158


account reserve
To C‘s account 70,000 70,000 By Cash 110,000* 110,000*
(deficiency By A and B
borne by (deficiency
solvent borne by
partners) solvent
To Balance c/d 450,000 450,000 _______ partners) -______ -_____ 140,000
630,000 630,000 410,000 630,000 630,000 410,000
* Solvent partners bring cash to the extent of loss arising upon realisation of assets of the firm as per
Garner vs Murray Rule.

Balance Sheet of RR Traders


As on 31.10.2070
(Immediately after commencement)
Liabilities Rs. Assets Rs.
Capital Accounts: Land and building 350,000
A 450,000 Plant and machinery 150,000
B 450,000 Furniture and fittings 20,000
Creditors 290,000 Stock in trade 200,000
Debtors 200,000
Cash at hand/bank 270,000
________ (W.N. 2) ________
1,190,000 1,190,000
Working Notes:
1. Agreed value of assets taken over by RR Traders
Rs.
Land and building 350,000
Plant and machinery 150,000
Furniture and fittings 20,000
Stock in trade 200,000
Debtors (500,000 – 300,000) 200,000
Cash at hand/bank 50,000
970,000
2. Cash in hand/bank balance of RR Traders as on 31.10.2070
Rs.
Opening Balance 50,000
Add: A‘s and B‘s contribution 220,000
(Rs.110,000 + Rs.110,000) _______
270,000

Nepal Accounting Standards

20. Answers:

(a) The provision of Nepal Accounting Standard 13 on ―Construction Contracts‖ Para 8 and 9 explains the
situations where accounting segmentation and combination of construction contracts can be applied.
As per para 8 of the standard, when separate proposals have been submitted, and separate
negotiations have been concluded, costs and revenues identified separately, the contract for
construction of a number of assets shall be treated as separate construction contracts. Therefore,
Ram has to treat construction of each unit as a separate construction contract.

(b). Areas in which different accounting policies may be adopted: The following are three areas in which
different accounting policies may be adopted by different enterprises:
(i) Methods of depreciation, depletion and amortization.
(ii) Valuation of inventories.
(iii) Valuation of Fixed Assets.
(The above three areas are not exhaustive. There are other areas also)
Disclosure requirements of accounting policies :The disclosure requirements as prescribed in
Accounting Standard 1 (NAS 1) ‗Disclosure of Accounting Policies‘ are as follows :
(i) All significant accounting policies adopted in the preparation and presentation of financial
statements should be disclosed at one place and they should form part of the financial statements.

RTP-CAP II –-2014-December@ICAN Page 21 of 158


(ii) Any change in the accounting policies which has a material effect in the current period should be
disclosed along with the amount, to the extent ascertainable, by which any item in the financial
statement is affected. Where such amount is not ascertainable, wholly or in part, the fact should be
indicated. However, if a change in accounting policies is reasonably expected to have a material effect
in later periods, the fact of such change should be appropriately disclosed in the period in which the
change is made.
(iii) If the fundamental accounting assumptions viz .Going concern, Consistency and Accrual are
followed in the preparation of financial statements, specific disclosure is not required. If a
fundamental accounting assumption is not followed, the fact should be disclosed.
(c). As per NAS 7, Revenue from Sale of goods shall be recognized when all the following conditions have
been satisfied:
(i) The entity has transferred to the buyer the significant risks and rewards of ownership of goods;
(ii) The entity retains neither continuing managerial involvement to the degree usually associated
with ownership nor effective control over the goods sold;
(iii) The amount of revenue can be measured reliably;
(iv) It is probable that the economic benefits associated with the transaction will flow to the entity;
and
(v) The cost incurred or to be incurred in respect of the transaction can be measured reliably.

(d). An asset is recognized in the balance sheet when it is probable that the future economic benefits will
flow to the enterprise and the asset has a cost or value that can be measured reliably.
An asset is not recognized in the balance sheet when expenditure has been incurred for which it is
considered improbable that economic benefits will flow to the enterprise beyond the current
accounting period. Instead such a transaction results in the recognition of an expense in the income
statement. This treatment does not imply either that the intention of management in incurring
expenditure was other than to generate future economic benefits for the enterprise or that
management was misguided. The only implication is that the degree of certainty that economic
benefits will flow to the enterprise beyond the current accounting period is insufficient to warrant the
recognition of an asset.

(e). As per provision of NAS, a foreign currency transaction should be recorded, on initial recognition in
the reporting currency by applying to the foreign currency amount, the exchange rate between the
reporting currency and the foreign currency at the date of transaction. Accordingly, on 31.12.2070,
borrowings will be recorded at Rs. 46,500,000 (500,000 x 93).
As per the standard, at each balance sheet date, foreign currency monetary items should be reported
using the closing rate. Accordingly, on 31.3.2071, borrowings will be recorded at Rs. 47,500,000
(500,000 x 95).
In the books of Bikash Ltd.
Journal Entries
Date Particular Debit Credit
31.12.2070 Bank A/c Dr. 46,500,000
To Borrowings A/c 46,500,000
31.3.2071 PL A/c Dr. (difference in exchange rate) 1000,000
To Borrowings A/c 1,000,000
30.6.2071 Borrowings A/c Dr. 47,500,000
PL A/c Dr. (Difference in exchange rate) 1,500,000
To Bank A/c 49,000,000

Short Notes
21. Answers:
(a). Sale and Lease Back Transaction
A sale and leaseback transaction involves the sale of an asset by the vendor and the leasing of the
same asset back to the vendor. The lease payments and the sale price are usually interdependent as
they are negotiated as a package. The accounting treatment of a sale and leaseback transaction
depends upon the type of lease involved.

(b). Escalation Clause in Contracts

RTP-CAP II –-2014-December@ICAN Page 22 of 158


This is a clause provided in the contracts to cover up any changes in the price of the contract due to
changes in the price of raw materials and labour or change in utilization of factor of production. The
object of this clause is to safeguard the interest on both sides against unfavourable change in prices.
The basis, on which the factor prices are based, is laid down in the contract. For e.g.: in a contract with
transport undertaking, it may be provided that the price per ton km will increase or decrease for each
rise or fall in price of diesel by 10% of the prevailing prices.

(c). Re – Insurance
If an insurer does not wish to bear the whole risk of policy written by him, he may reinsure a part of
the risk with some other insurer. In such a case the insurer is said to have ceded a part of his business
to other insurer. The reinsurance transaction may thus be defined as an agreement between a ‗ceding
company‘ and ‗reinsurer‘ whereby the former agreed to ‗cede‘ and the latter agrees to accept a certain
specified share of risk or liability upon terms as set out in the agreement. A ‗ceding company‘ is the
original insurance company which has accepted the risk and has agreed to ‗cede‘ or pass on that risk to
another insurance company or a reinsurance company. It may however be emphasized that the original
insured does not acquire any right under a reinsurance contract against the reinsurer. In the event of
loss, therefore, the insured‘s claim for full amount is against the original insurer. The original insurer
has to claim the proportionate amount from the reinsurer. There are two types of reinsurance
contracts, namely, facultative reinsurance and treaty reinsurance. Under facultative reinsurance each
transaction has to be negotiated individually and each party to the transaction has a free choice, i.e.,
for the ceding company to offer and the reinsurer to accept. Under treaty reinsurance a treaty
agreement is entered into between ceding company and the reinsurer whereby the volume of the
reinsurance transactions remain within the limits of the treaty.

(d). Component of Financial Statement as per NAS


A complete set of financial statement includes the following components:
1. A Balance Sheet: Statement showing financial position of entity as on date.
2. An income statement: Statement showing performance of the entity during the period.
3. A Statement of changes in equity showing either:
i) All changes in equity, or
ii) Changes in equity other than those arising from transactions with equity holders acting in their
capacity as equity holder
4. A cash flow statement: Summary of cash inflow and outflow from operating, financing and
investing activities.
5. Notes to accounts: Comprising a summary of significant accounting policies and other explanatory
notes.

(e). Features of Farm Accounting


(i) Agricultural sector is unorganized and dominated by small farmers. Most agricultural farms are
family oriented and part of the farms produce is consumed by the family memb ers.
(ii) The family takes part in management. Besides it provides labor for the farm. Farmers cannot
afford the additional expenses involved in hiring a person to maintain accounts.
(iii) Agriculture is in some cases a seasonal occupation and many farmers have other occupations
also. Farming operations are uncertain due to natural calamities.
(iv) There are many divisions in farm accounting; finished product of one can become the raw
material for another.
(v) Tax authorities do not insist on maintenance of books of account. Collection of statistics by the
government is also not adequate

RTP-CAP II –-2014-December@ICAN Page 23 of 158


AUDIT AND ASSURANCE

Questions
Principle and Concept
1) Write short notes on Audit versus Investigation.

Planning and Assurance Engagement


2) What are the disadvantages of the use of an audit programme?

3) An NGO operating in Kathmandu had collected large scale donations for flood victims. The donations so
collected were sent to different NGOs operating in Nepalgunj for relief operations. This NGO operating in
Kathmandu has appointed you to audit its accounts for the year in which it collected and remitted donations for
flood victims. Draft audit programme for audit of receipts of donations and remittance of the collected amount to
different NGOs. Mention six points each, peculiar to the situation, which you will like to incorporate in your audit
programme for audit of said receipts and remittances of donations.

Audit Risk and Internal Control


4) Explain the Relationship between materiality and audit risk.

5) Why is review of internal control system is regarded as indispensable part of the overall audit programme?

6) In a medium size trading organisation the accountant was given additional responsibility of making recoveries
from the debtors. On one occasion, when an insurance claim of Rs. 25,000 was received, he credited the same to
the account of a debtor and misappropriated the cash which he had recovered from the said debtor. Analyze the
weaknesses in the internal control system which led to this situation.

7) During the audit of a company you observed that, No entry is passed for cheques received by them on the last day
of the year, but not yet deposited with the bank. Give your comments.

8) Mention any six points to be considered for good internal control for collection of tuition fees from students of
college.

Vouching and Verification


9) Your article is of the opinion that, while vouching payments, the auditor merely check proof that money has been
paid away.‟ Clarify him.

10) The management of Urgent Limited suggested for quick completion of the statutory audit that it would give its
RTP-CAP II –-2014-December@ICAN Page 24 of 158
representation about the receivables in terms of their recoverability. The management also acknowledged to the
auditors that the management would give their representation after scrutinizing all accounts diligently and they
own responsibility for any errors in these respects. It wanted auditors to complete the audit checking all other
important areas except receivables. The auditor certified the account clearly indicating in his report the fact of
reliance he placed on representation of the management. Comment.

11) Write internal control questionnaire for verification of Fixed Assets.

Computerized Environment
12) “Installation of Computer Operating System has created both benefits and problems for auditors”. Explain the
Statement?

13) Comment on the overall objective and scope of an audit does not change in an EDP environment.

Analytical Procedures
14) In the course of audit of a trade, you noticed that although there is no change in either selling or purchase price of
the goods, there is considerable increase in Gross Profit Ratio in comparison to previous year. What matters
would you examine to assess the reason for such increase?

15) As an auditor of a Limited Company, you observe that during the month of Ashad, 2071, sales invoices were not
recorded in books of accounts. You also observe that payment of wages was much higher compared to last year.
Keeping in mind the above, analyse possible ways of manipulation of accounts

Audit of Special Sectors


16) Mention the special points in the case of an audit of the entity from Incomplete Records.

17) Draft an audit programme to audit the receipts of a cinema theatre owned by a partnership firm.

Cases related to Auditing and Accounting Standards


18) State with reason your views on the following :
a) Birat Ltd. Purchased a factory building for Rs.2 crores at Biratnagar. The company paid Rs.2 lakhs professional
fees to its solicitors for providing consultancy in connection with purchase of factory. The accountant debited Rs.2
lakhs as legal expenses in the account books of the company.

RTP-CAP II –-2014-December@ICAN Page 25 of 158


b) During audit it was revealed that, as a result of an improper cut –off of inventory shipments at the end of the year,
approximately Rs. 500,000 of sales pertaining to the subsequent year was recorded in the current year by ABC (P)
Ltd for the year ended 31st Ashad 2070. The management of ABC however refuses to adjust the financial
statements for the Rs. 50,000 over booking of sales.

c) Mr. LM Bista has been doing audit of PQR Co. Ltd. for the past few years. His experience as auditor of the
company for the past few years shows that accounts of the company are always in order. The managing director of
PQR Co. Ltd. requested him to issue an unqualified audit report urgently without conducting the audit for the year
since the unqualified audit report is required immediately in order to obtain a contract.

d) Big Ones Limited at its general meeting authorized its Board of Directors to fill up the vacancy in the office of
auditor, which was caused by the resignation of an auditor.

e) During the year 2011-12, a company has developed blocks and dies for Rs.30 Lakhs. The company did not provide
any depreciation on the said blocks and dies. State the auditor‟s duties.

19) As an Auditor, comment on the ENT Enterprises entered into a contract for sale of its goods worth Rs.50 lacs with
EN Ltd. The goods were inspected, approved and finalised by the inspection team of EN Ltd. EN Ltd. made the
whole payment of Rs. 50 lacs. However, it requested ENT Enterprises to dispatch the goods in six equal monthly
instalments from Magh, 2070 to Ashad, 2071. In the month of Baishak, 2071, due to natural calamity, EN Ltd.
informed ENT Enterprises to stop dispatches of the remaining three instalments until further notice. At the time of
finalising its accounts for the financial year 2070-71, ENT Enterprises booked sales amounting to Rs.25 lacs and
showed remaining Rs.25 lacs as advance against Sales.

20) Give your comments on “The Nepca Ltd., a Pharmaceutical Company, while valuing its finished stock at the year
end wants to include interest on Bank Overdraft as an element of cost, for the reason that overdraft has been taken
specifically for the purpose of financing current assets like inventory and for meeting day to day working
expenses”.
21) As an Auditor, comment on the Tyre Ltd. is engaged in manufacturing and supply of tyres to KTM Motors Ltd.
As per terms of supply, full price of the goods are not released by KTM Motors Ltd. but 10% thereof is retained
and paid after one year, if there is satisfactory performance of the parts supplied. Tyre Ltd. accounts for only 90%
of the invoice value as sale at the time of supply and balance 10% is accounted as sale in the year of receipt of
payment.
According to NAS 07 on Revenue recognition, revenue from sale of goods should be recognised when
the seller has transferred to the buyer, the property in the goods for a price or when the seller has
transferred all significant risk and rewards and the seller repairs no effective control over goods and no
significant uncertainty exists regarding the amount of consideration and its collectibility.
RTP-CAP II –-2014-December@ICAN Page 26 of 158
22) M/s Left & Right , a partnership firm, running a nursing home have decided to discontinue you as an auditor for
the next year and requests you to handover all the relevant workingpapers of the previous year.

23) M, Statutory Auditor of ABC Ltd wants to verify cash on hand as on 31st Ashad, 2071. The Management informs
Mr. M. that it is not possible to cooperate, as cashier has been hospitalized. Advise Mr. M. on how to deal with the
situation.

Accounting Standards
24) List out the procedures to be performed by the auditor in expressing opinion on 'going concern' assumption

Auditing Standards
25) Can the External Auditor rely upon the work of an Internal Auditor?

Audit Preperation
26) Your client is requesting you for starting the audit of the financial year 2070/71. Describe a set of instructions,
which you give to your client before the start of actual audit.

27) How will you vouch the following?


a) Assets acquired on Hire Purchase Basis
b) Salaries and Wages
c) Advertisement Expenses
d) Sale of Assets
e) Insurance Claims

28) Write short note on following:


a) Doing a statutory audit is full of risk‟
b) Contents of Audit Note book:
c) Precautions by an auditor while applying test check techniques
d) Auditing through the Computer
e) Peer Review
f) Propriety audit

RTP-CAP II –-2014-December@ICAN Page 27 of 158


SUGGESTED ANSWERS
1) Answer
Auditing versus Investigation: Audit is generally objected to find out whether the accounts show true
& fair view. It is a critical examination of books of accounts.
Investigation on the other hand is critical examination of the accounts with a special purpose. For
example if fraud is suspected and an accountant is called upon to check the accounts to whether fraud
really exists and if so, the amount involved, the character of the enquiry changes into investigation.
Investigation may be undertaken in numerous areas of accounts, e.g., the extent of waste and loss,
profitability, cost of production etc. It extends scope beyond books of accounts.
For auditing on the other hand, the general objective is to find out whether the accounts show a true and
fair view. The auditor seeks to report what he finds in the normal course of examination of the accounts
adopting generally followed techniques unless circumstances call for a special probe. Fraud, error,
irregularity, whatever comes to the auditor‟s notice in the usual course of checking, are all looked into in
depth and sometimes investigation results from the prima facie findings of the auditor.

2) Answer.
Disadvantages of the use of an Audit Programme
(i) The work may become mechanical and particular parts of the programme may be carried out without any
understanding of the object of such parts in the whole audit scheme.
(ii) The programme often tends to become rigid and inflexible following set grooves; the business may change in
its operation of conduct, but the old programme may still be carried on.
(iii) Inefficient assistants may take shelter behind the programme i.e. defend deficiencies in their work on the
ground that no instruction in the matter is contained therein.
(iv) A hard and fast programme may kill the initiative and innovation of efficient and enterprising assistants.

3) Answer
Receipt of Donations
(i) Internal Control System: Existence of internal control system particularly with reference to division of
responsibilities in respect of authorised collection of donations, custody of receipt books and safe custody of
money.
(ii) Custody of Receipt Books: Existence of system regarding issue of receipt books, whether unused receipt
books are returned and the same are verified physically including checking of number of receipt books and
sequence of numbering therein.
(iii) Receipt of Cheques: Receipt Book should have carbon copy for duplicate receipt and signed by a responsible
official. All details relating to date of cheque, bank‟s name, date, amount, etc. should be clearly stated.
(iv) Bank Reconciliation: Reconciliation of bank statements with reference to all cash deposits not only with
RTP-CAP II –-2014-December@ICAN Page 28 of 158
reference to date and amount but also with reference to receipt book.
(v) Cash Receipts: Register of cash donations to be vouched more extensively. If addresses are available of donors
who had given cash, the same may be cross-checked by asking entity to post thank you letters mentioning
amount, date and receipt number.
(vi) Foreign Contributions, if any, to receive special attention to compliance with applicable laws and regulations.
Remittance of donations to different NGOs
(i) Mode of Sending Remittance: All remittances are through account payee cheques. Remittances through
Demand Draft would also need to be scrutinised thoroughly with reference to recipient.
(ii) Confirming Receipt of Remittance: All remittances are supported by receipts and acknowledgements.
(iii) Identity: Recipient NGO is a genuine entity. Verify address, Registration Number, etc.
(iv) Direct Confirmation Procedure: Send confirmation letters to entities to whom donations have been paid.
(v) Donation Utilisation: Utilisation of donations for providing relief to Tsunami victims and not for any other
purpose.
(vi) System of NGOs‟ Selection: System for selecting NGO to whom donations have been sent.

4) Answer
Relationship between materiality and audit risk: NSA 320 on „Materiality in Planning and
Performing an Audit‟ requires that the auditor should consider materiality and its relationship with audit
risk when conducting an audit. Materiality depends on the size and the nature of the items judged in the
particular circumstances of its misstatement.
The audit should be planned so that audit risk is kept at an acceptably low level. There is an inverse
relationship between Materiality and the degree of audit risk. Higher the materiality level the lower the
audit risk and vice-versa. After the auditor has assessed the inherent and control risks, he should consider
the level of detection risk that he is prepared to accept and, based upon his judgment, select appropriate
substantive audit procedures. If the auditor does not perform any substantive procedures, detection risk,
that is, the risk that the auditor will fail to detect a misstatement, will be high.
The auditor‟s assessment of audit risk may change during the course of an audit according to the need
and development of the circumstances.

5) Answer
So far as the auditor is concerned, the examination and evaluation of the internal control system is an
indispensable part of the overall audit programme. The auditor needs reasonable assurance that the
accounting system is adequate and that all the accounting information which should be recorded has in
fact been recorded. Internal control normally contributes to such assurance. The auditor should gain an
understanding of the accounting system and related internal controls and should study and evaluate the
operations of these internal controls upon which he wishes to rely in determining the nature, timing and

RTP-CAP II –-2014-December@ICAN Page 29 of 158


extent of other audit procedures. The review of internal controls will enable the auditor to know:
(i) whether errors and frauds are likely to be located in the ordinary course of operations of the business;
(ii) whether an adequate internal control system is in use and operating as planned by the management;
(iii) whether an effective internal auditing department is operating;
(iv) whether any administrative control has a bearing on his work (for example, if the control over worker
recruitment and enrolment is weak, there is a likelihood of dummy names being included in the wages sheet
and this is relevant for the auditor);
(v) whether the controls adequately safeguard the assets;
(vi) how far and how adequately the management is discharging its function in so far as correct recording of
transactions is concerned;
(vii) how reliable the reports, records and the certificates to the management can be;
(viii) the extent and the depth of the examination that he needs to carry out in the different areas of accounting;
(ix) what would be appropriate audit technique and the audit procedure in the given circumstances;
(x) what are the areas where control is weak and where it is excessive; and
(xi) whether some worthwhile suggestions can be given to improve the control system.
The auditor can formulate his entire audit programme only after he has had a satisfactory understanding
of the internal control systems and their actual operation. If he does not care to study this aspect, it is
very likely that his audit programme may become unwieldy and unnecessarily heavy and the object of
the audit may be altogether lost in the mass of entries and vouchers. Often, after installation of a system,
no proper follow up is there by the management to ensure compliance so , it is also important for him to
know whether the system is actually in operation..
It would be better if the auditor can undertake the review of the internal control system of client which
will enable him to be more objective in the framing of the audit programmes. He will also be in a
position to bring to the notice of the management the weaknesses of the system and to suggest measures
for improvement.

Hence it can be concluded that the extent and the nature of the audit programme is substantially
influenced by the internal control system in operation. A proper understanding of the internal control
system in its content and working also enables an auditor to decide upon the appropriate audit procedure
to be applied in different areas to be covered in the audit programme. In a situation where the internal
controls are considered weak in some areas, the auditor might choose an auditing procedure or test that
otherwise might not be required; he might extend certain tests to cover a large number of transactions or
other items than he otherwise would examine and at times he may perform additional tests to bring him
the necessary satisfaction.

6) Answer.

RTP-CAP II –-2014-December@ICAN Page 30 of 158


Following two essential features of internal control are relevant here:
(i) Breaking the chain of the work in a manner so that no single person can handle a transaction from the
beginning to the end and
(ii) Segregation of accounting and custodial functions.
Weakness in internal control system in the instant case:
a) The accountant is receiving cash and also passing the entries in the books. The accountant should not have
been allowed to effect recoveries.
b) It also appears that system for issuing receipts for amount received - whether cash or cheque is also lacking.
c) In a small and to some extent medium size organization, the supervision of the owner offsets the deficiencies
in internal control system. But in this case, it appears, that supervision and personal control is also lacking.
Thus, in the given case, the main weakness of the system is that it is ignoring the basic requirements of a
good internal control system.

7) Answer.
Cheques Received on the last day of Accounting Year: It is a quite normal that in any on going business entity
many a time‟s cheques are received from the customers on the last day of the accounting year. It is also quite
likely, that cheques received on the last day of the accounting year could not be deposited in the bank. Though
normally speaking, it is expected that all cheques should be deposited in the bank daily. But there may be a
possibility that such cheques which are received particularly during the late hours could not be deposited in the
bank. Therefore, it is quite important to ensure that the system of internal control is effective and such cheques
should be properly accounted for to avoid any frauds and that the financial statements reflect a true and fair view.
As far as internal control system is concerned, it should be ensured that a list of such cheques is prepared
in duplicate and a copy of the same has been sent to person controlling the debtors‟ ledger and a second
copy is handed over to cashier along with the cheques received. The person who is controlling the
debtors‟ ledger should ensure that proper accounting entries have been passed by crediting respective
debtors‟ accounts. The balance of cheques-in-hand should also be disclosed along with the cash and
bank balances in the financial statements.
8) Answer.
Internal control points for collection of tuition fees:
(i) There must be a clear cut tuition fee structure approved by the college council.
(ii) The challan or paying in slip should contain necessary fields for identifying the roll number of the student,
class, and period for which fees is paid etc. The slips should have such number of counterfoils to cross check
the remittance.
(iii) The paying in slip when filled by the students, should be checked for its correctness as to applicable amount
etc by one clerk and the amount should be entered in a scroll. He must initial the slip which authorises the
cashier to accept the fees as per slip.
(iv) The cashier scroll and the authorising officer/s scroll should be checked by an office daily.

RTP-CAP II –-2014-December@ICAN Page 31 of 158


(v) All remittance should be banked each day. No amount should be allowed to be spared for meeting any type of
expense.
(vi) Alternatively, the fees may be directly remitted into bank and banker‟s daily remittance slip should be
scrutinised by college officers.
(vii) Arrears list should be periodically prepared from the students rolls. Any concession, remission of tuition fees
should have approval of competent authority.
(viii) Delayed remittance should carry fines or compensating charges for delay.
(ix) When students are readmitted after removal for non-payment of fees, the admission should carry the
permission of competent authority.

9) Answer.
Vouching is a substantive audit procedure which aims at verifying the genuineness and validity of a transaction
contained in the accounting records. It involves examination of documentary evidence to support the genuineness
of transaction. Thus the object of vouching is not merely to ascertain that money has been paid away; but the
auditor aims to obtain reasonable assurance in respect of following assertions in regard to transactions recorded in
the books of account that –
(i) a transaction is recorded in the proper account and revenue or expense is properly allocated to the accounting
period;
(ii) a transaction pertains to entity and took place during the relevant period;
(iii) all transactions which have actually occurred have been recorded;
(iv) all transactions were properly authorised; and
(v) transactions have been classified and disclosed in accordance with recognised accounting policies and
practices.
Thus, it is through vouching that the auditor comes to know the genuineness of transactions recorded in
the client‟s books of account wherefrom the financial statements are drawn up. Apart from genuineness,
vouching also helps the auditor to know the regularity and validity of the transaction in the context of the
client‟s business, nature of the organisation and organisational rules.
Thus, the auditor‟s basic duty is to examine the accounts, not merely to see its arithmetical accuracy but
also to see its substantial accuracy and then to make a report thereon.
This substantial accuracy of the accounts and emerging financial statements can be known principally by
examination of vouchers which are the primary documents relating to the transactions. If the primary
document is wrong or irregular, the whole accounting statement would, in turn, become wrong and
irregular. Precisely auditor‟s role is to see whether or not the financial statements are wrong or irregular,
and for this, vouching is simply imperative.
Thus, vouching which has traditionally been the backbone of auditing does not merely involve checking
arithmetical accuracy but goes much beyond and aims to check the genuineness as well as validity of
transactions contained in accounting records
RTP-CAP II –-2014-December@ICAN Page 32 of 158
10) Answer
Management Representation: The management of Urgent Limited wants the auditor to carry out audit
on all areas except on area of receivables. There cannot be any restriction on scope of audit in case of
statutory audit.
The management representation, according to NSA 580 - Written Representations by management,
cannot substitute other audit evidence that the auditor could reasonably expect to be available to the
auditor. The audit evidences available for checking receivables- say, invoices, debt acknowledgement
documents, receipts, statement of accounts, confirmations etc., are available evidences which auditor is
duty bound to verify.
Just because management had owned responsibility for the correctness of its evaluation of receivables,
the auditor cannot shirk his responsibility.
This is negligence on his part if he relies on the management representation without assessing the
corroborative available evidences.

11) Answer
(i) Are fixed assets verified periodically?
(ii) Is there a written procedure for such verification?
(iii) Does the procedure provide for verification/confirmation of fixed assets with third parties?
(iv) Does the procedure provide for verification of compliance with the warranties and conditions in the relevant
insurance policies?
(v) Are reports prepared on such verification?
(vi) Do such reports indicate damaged/obsolete items of fixed assets?
(vii) Are discrepancies disclosed by such reports investigated?
(viii) Are the records and financial accounts corrected, with proper authority?
(ix) Are damaged/obsolete items disclosed by such reports, removed from the records and financial accounts
with proper authority?

12) Answer
Computer Operating Systems and the Auditor: The installation of computer operating system is an
integral and absolutely essential part of a computer even in a stand alone PC- based environment. In fact
it is difficult to visualize a computer to be operational without installation of the operating system. With
the advancement of technology, the operating systems are part of the server or hard disc and provide lots
of options and flexibility to the user. The provision of all these built-in-features is quite beneficial to user
and the auditor alike. The data stored in the system can be extracted depending upon the requirement,

RTP-CAP II –-2014-December@ICAN Page 33 of 158


e.g., records relating to students can be region-wise, city-wise, examination centre-wise, etc to compare
the performance. At the same time, these advanced features of operating systems have given rise to
several general hazards associated with it. In these circumstances, it becomes essential to restrict the
access to data by ensuring proper security system such as passwords and other access controls, etc.
However, such system at time can be hacked and then the entire data base is vulnerable to manipulation.
Thus, from the auditor‟s point of view installation of operating system have created both benefits and
problems. The major benefits flow from the fact of examination of execution of transactions, taking
samples, etc. while problems might arise to potential manipulation of the data. It may however, be noted
that benefits from the operating system for outweigh the problems associated with it.

13) Answer
The principal objective of an audit of financial statements, prepared within a framework of recognised accounting
policies and practices and relevant statutory requirements, if any, is to ensure that the financial statements reflect a
true and fair view. The scope of an audit of financial statements is determined by the auditor having regard to the
terms of the engagement, the requirements of relevant legislation and the pronouncements of the Institute. This
would involve assessment of reliability and sufficiency of the information contained in the accounting records and
other source data by study and evaluation of accounting system and internal controls in operation.
The overall objective and scope of an audit does not change in an EDP environment but the use of a
computer changes the processing and storage of financial information and may affect the organisation
and procedures employed by the entity to achieve adequate internal control. Accordingly, the procedures
followed by the auditor in his study and evaluation of the accounting system and related internal controls
and nature, timing and extent of his other audit procedures may be affected by an EDP environment. The
computerisation of accounts would also have an impact on the increase in fraud and errors. Thus when
auditing in an EDP environment, the auditor should have sufficient understanding of computer hardware,
software and processing systems to plan the engagement and to understand how EDP affects the study
and evaluation of internal control and application of auditing procedures including computer-assisted
audit techniques. The auditor should also have sufficient knowledge of EDP to implement the auditing
procedures, depending on the particular audit approach adopted.
Thus, it is clear from the above that overall objective and scope of audit does not change irrespective
of fact that whether the accounting information is generated manually or through EDP.
Fraudulent Financial Reporting:
It involves intentional misstatements or omissions of amounts or disclosures in financial statements to
deceive financial statement users.
Fraudulent financial reporting may involve:
Deception such as manipulation, falsification, or alteration of accounting records or supporting
RTP-CAP II –-2014-December@ICAN Page 34 of 158
documents from which the financial statements are prepared. For example, in a period of rising prices,
sales contract documents may be ante-dated to record sales at prices lower than the prices at which sales
have actually taken place.
Misrepresentation in, or intentional omission from, the financial statements of events, transactions or
other significant information. For example, goods sold may not be recorded as sales but included in
inventories Intentional misapplication of accounting principles relating to measurement, recognition,
classification, presentation, or disclosure. For example, where a contracting firm follows the „completed
contract‟ method of accounting but does not provide for a known loss on incomplete contracts.

14) Answer
In assessing the reason for considerable increase in Gross Profit Ratio, the auditor should examine the
followings:
(i) Under valuation of opening stock due to non-inclusion of certain items or applying lower rate to one or more
items of stock.
(ii) Over valuation of closing stock either by the inclusion of fictitious items or over valuing some of the items of
Stock.
(iii) Change in the method of valuation of opening and closing stock. For example, opening stock was valued at
lower of cost and market value, whereas closing stock has been valued at market price which is higher than
cost.
(iv) Goods sold but not delivered are included in stock.
(v) Goods delivered last year but sales invoice raised in current year.
(vi) Recording of fictitious sales to boost up profits.
(vii) Goods returned to supplier awarding dispatch and included in closing stock.
(viii) Goods returned by customers but not debited to sales return and included in closing stock.
(ix) Inclusion in the closing stock of goods received for sale on approval or consignment basis.
(x) Treatment of goods sent on consignment basis as regular sales.
(xi) Non-provision of outstanding expenses like wages, carriage inward etc.
(xii) Wrong allocation of expenses in Trading and Profit & Loss Account.

15) Answer
Manipulation of Accounts: Accounts are falsified in order to conceal the true position of the business for
some purpose. They are always intentional, for a predetermined purpose and are generally committed
either by the owners or top management personnel or senior officers of the business. This type of fraud is
generally committed:
(i) to avoid incidence of income-tax or other taxes by showing profits at a lower figure.
(ii) for delaying a dividend when there are insufficient profits by showing profits at inflated figures.
RTP-CAP II –-2014-December@ICAN Page 35 of 158
(iii) to withhold declaration of dividend even there is adequate profit.
(iv) for receiving higher remuneration where managerial remuneration is payable by reference to profits.
Such frauds are difficult to be detected as they are committed by persons holding position of trust and
use carefully guarded by them. Such frauds are generally of the following nature:
(i) Recording fictitious sales or omission of sales
(ii) Recording fictitious purchases or suppression of purchases
(iii) Over valuation or under valuation of stock.
(iv) Recording fictitious expenses or omission of expenses
(v) Taking credit for accrued income not likely to be received or omission of income.
(vi) Revenue expenses changed to capital and vice-versa. NSA 240 “The Auditor‟s responsibilities relating to
Fraud in an Audit of Financial Statements” deals with the auditor‟s responsibilities for the detection of
material misstatement resulting from fraud and error. It requires a considerable skill and vigilance on the part
of an auditor. In doubtful cases he may refuse to believe the information supplied to him by any officer of the
concern.
An auditor, who uses adequate skill and reasonable care, is legally exempt from liability if he fails to
discover a well concealed detection. But an auditor by a skilled auditor should rarely permit such a
failure.
All possible opportunities for dishonesty and manipulation of the accounts must be considered and
guarded against and the degree of checking and investigation should be determined by the circumstances
surrounding the transactions and the effectiveness of the system of intended check in operation.

16) Answer
The examination of records and documents is one of the most important techniques through which an
auditor collects evidence. Therefore, in case the records and documents maintained by an enterprise are
incomplete, it would prove to be a great handicap to the auditor. An auditor may face the situation of
incomplete records under the following circumstances:
(i) Where records are kept on single entry basis; or
(ii) Where records are kept on double entry basis, but some of the records are destroyed accidentally, or are seized
by authorities, or are otherwise not available for the auditor‟s examination due to similar reasons.
Under the second circumstance, an ideal approach for carrying out audit would be that the auditor may
direct the management of the enterprise to complete or reconstruct the accounting records, e.g., if
vouchers are available but the cash book, journal and the ledger are not maintained, then the cash book,
journal and ledger should be written up. However, if vouchers are also not available, then cash
book/journal/ledger will have to be prepared by correlating the evidence available, e.g., memoranda
records, bank statements, statements from outside parties, etc. Even though such books which are
prepared may not be complete, but may still contain useful information for the auditor.

RTP-CAP II –-2014-December@ICAN Page 36 of 158


On the other hand, when books are maintained on single entry basis, then the management of the
enterprise would be asked to write up the books, to the extent possible, as they would have been written
up under double entry system.
In any case, the following steps would be required to conduct an audit:
(i) Ascertain that the balance sheet or statement of affairs as at the beginning of the year should be prepared and
all the relevant accounts should be opened in the ledger. Normally, under the single entry system, cash, bank,
and personal accounts are maintained.
(ii) Confirming that all entries on receipt side of the cash book are posted in the ledger, even by opening new
account(s) wherever necessary.
(iii) Check that all entries on the payment side of cash book are posted in the ledger.
(iv) Confirming that all entries appearing in bank account are posted in the ledger.
(v) Analyse personal accounts of debtors. This will provide vital information regarding credit sales, sales
returns, discounts allowed, bills received, bills dishonored, etc. It would be necessary to post such items to
relevant accounts, to complete the double entry from the debtor‟s accounts.
Similarly, it would be necessary to analyse the creditors' accounts and post entries relating to credit
purchase made, discounts earned, purchases returns, bills payable issued to suppliers, bills payable
dishonored, etc., to relevant accounts.
From an auditor's view point, the supervisory controls exercised by the owners are generally less reliable
and hence while auditing incomplete records, auditor will largely depend on extensive substantive
procedures and obtain external evidence, physical examination/ observation, management representation
and perform analytical procedures.

17) Answer
The role of audit programme in audit plan and performance: The audit programme is helpful both in planning and
performance stages of audit:
(i) The audit programme lists down areas of audit before commencement.
(ii) The audit timing is built therein; thereby it becomes a schedule of audit plan.
(iii) The staff who are entrusted with the audit assignment is also specified. It is a plan of resource allocation of
the firm.
(iv) It specifies the procedures to be checked during the audit.
(v) As the audit work is split into various elements of procedures to be performed, the audit programme acts as a
guiding chart or check list during the performance of audit.
(vi) Since the staff in charge of each work is specified and they sign the programme, it extracts the
responsibility from the audit assistants.
(vii) The working papers of the audit staff can be reviewed against the audit programme which helps a base of
reference for evaluation of the performance before reporting on the financial statements.
(viii) It also helps in preparing a diary of the performance and plan and also base for billing the clients for the time

RTP-CAP II –-2014-December@ICAN Page 37 of 158


and manpower involved in the audit.
Programme for receipts of cinema theatre of a firm: Audit programme for checking the receipts of a
cinema theatre of a partnership firm.
(i) The partnership deed should be first scrutinized.
(ii) The receipts of the cash from partners on capital and current accounts should be vouched with reference to
the relative terms in the deed.
(iii) The internal control for collections from sale of tickets should be checked.
(iv) See that the tickets are serially numbered and effective custody of un-issued tickets are in existence.
(v) Check the rough cash book and reconcile from the stock of ticket books issued, the cash to be collected each
day.
(vi) Check that the cash balance and ticket sales from stock is daily checked by the manager.
(vii) Check that the collections are banked daily, the very next day.
(viii) See rates for each class and the ticket rates are as per current prices.
(ix) The entertainment tax collected should be separately accounted for its subsequent payment to the
government agencies.
(x) Check the relation between the amounts of tax collected and sales.
(xi) The collections from the advertising and publicity materials should be checked with reference to the terms of
agreement.
(xii) Income from canteen, stalls, parking facilities should also be checked and see that the income are fairly
booked without any seepage.
(xiii) The cash collections should not be used for meeting petty cash expenses. There should be separate impressed
system.
(xiv) Do surprise checking of cash balances.
(xv) See that cash collections are insured and the policy is in force.

18) State with reason your views on the following :


f) Answer
A capital expenditure is that which is incurred for the under mentioned purposes:
(a) Acquiring fixed assets, i.e., assets of a permanent or a semi-permanent nature, which are held not for
resale but for use with a view to earning profits.
(b) Making additions to the existing fixed assets.
(c) Increasing earning capacity of the business.
(d) Reducing the cost of production.
(e) Acquiring a benefit of enduring nature of a valuable right.
Expenses which are essentially of a revenue nature, if incurred for creating assets or adding to its value
or achieving higher productivity, are also regarded as expenditure of a capital nature. For Example:
Material and wages-capital expenditure when expended on the construction of a building or erection of
RTP-CAP II –-2014-December@ICAN Page 38 of 158
machinery.
In view of the above, Rs. 2, 00,000 has to be added in the cost of Land and Building and cannot be
shown as revenue expenditure.

g) Answer
The profit and Loss account of the business enterprise should include only income and expenses relating to the
relevant financial year. Inclusion of any income and /or expenses pertaining to the previous or the subsequent year
in the profit and loss account of the current year will inhibit presentation of a true and fair view of the working
result of the enterprise. In the books of ABC (P) Ltd. the auditor founds appx. Rs.500,000 of sales applicable to the
subsequent year recorded as sales in the current year due to improper cut- off of inventory shipments at the year-
end. Having detected overstatement of sales will artificially inflate the net profit, thereby by Rs.500,000, not
presenting a true and fair view of the operating results of the company. Should the management refuse to take heed
of suggestions; the auditor should express qualified opinion in his audit report.

h) Answer.
Mr. LM Bista should conduct the audit of PQR Co. Ltd. in accordance with Nepalese Standards on Auditing
having regard to the requirements of NSA, relevant professional bodies, Legislation, regulations and where
appropriate, the terms of the audit engagement and reporting requirements. For this he needs to obtain sufficient
evidence and information to verify the transaction of PQR Co. Ltd. for the year. The auditor should not only rely
on the past tract records of the company and agree to request of his client to issue unqualified audit report.
Hence the request of management to issue unqualified report based on the past track record is not appropriate.

i) Answer.
“Depreciation is a measure of the wearing out, consumption or other loss of value of a depreciable asset arising
from use, effluxion of time, and obsolescence through technology and market changes. Depreciation is allocated
so as to charge a fair proportion of the depreciable amount in each accounting period during the expected useful
life of the asset. Depreciation includes amortization of assets whose usefulness is predetermined”.
The accounting standard recommends that the depreciable amount of a depreciable asset should be
allocated on a systematic basis to each accounting period during the useful life of the asset. The
depreciation has to be provided on semi-permanent assets, e.g., patents, trademarks, blocks and dies, etc
apart from fixed assets. Since the auditor is not in a position to estimate the working life of a majority of
them, for this he has to rely on the opinion of persons who have a technical knowledge of the assets. He
must, however satisfy himself that an honest attempt has been made to estimate the working life of each
asset, that the total provision for depreciation is adequate and that the method adopted for determining
that amount to be written off appears to be fair and reasonable. If he is of the opinion that the provision

RTP-CAP II –-2014-December@ICAN Page 39 of 158


for depreciation is not adequate, he should report to the appropriate authority. He must also see that
depreciation written off is properly disclosed in the Profit and Loss Account and the Balance Sheet.

j) Answer.
Casual vacancy arising in the office of auditor on account of resignation of an auditor cannot be filled up by the
Board of Directors. Only the company in general meeting shall fill up the vacancy caused by resignation of the
auditor. The power cannot be delegated to the Board of Directors. The appointment by the Director is not valid.

19) Answer
NAS 07 requires that revenue from sale of goods should be recognized when the following conditions
are followed:
(i) The entity has transferred to the buyer the significant risks and rewards of ownership of the goods.
(ii) The entity does not retain continuing managerial involvement and effective control over goods sold
(iii) The amount of revenue can be measured reliable
(iv) It is probable that the economic benefits associated with the transaction will flow to the entity and
(v) The costs incurred or to be incurred in respect of the transaction can be measured reliable.
In the given case, the sale transaction carried by ENT Enterprise fulfills all the condition stated above.
Hence, the accounting entry passed by ENT Enterprise recognising partial revenue to the extent of goods
dispatched is not correct. The entire amount of Rs.50 lakhs should be recognised as sales.

20) Answer
Cost of Inventories: As per Nepal Accounting Standard 10 on Inventories, cost of inventories
comprises all costs of purchase, costs of conversion and other costs incurred in bringing the inventories
to their present location and condition. However, it makes clear that interest and other borrowing costs
are usually not included in the cost of inventories because generally such costs are not related in bringing
the inventories to their present location and condition.
Therefore, the intention of Nepca Ltd. to include interest on bank overdraft as an element of cost is not
acceptable because it does not form part of cost of production.

21) Answer
According to NAS 07 on Revenue recognition, revenue from sale of goods should be recognised when the seller
has transferred to the buyer, the property in the goods for a price or when the seller has transferred all significant
risk and rewards and the seller repairs no effective control over goods and no significant uncertainty exists
regarding the amount of consideration and its collectibility.
In the given case the goods as well as the risk and ownership has been transferred by Tyre Ltd., to Indian
Automobile Ltd., on the basis of invoice and delivery of material. In the instant case, therefore, Gear

RTP-CAP II –-2014-December@ICAN Page 40 of 158


Ltd., should recognise sale at full 100% of the invoice value in spite of the fact that 10% payment will
be released after one year. However,depending upon the past experience regarding collectibility of 10%
amount, they can make a provision for the amount that is not likely to be realised. Hence, the treatment
given by the company is not correct and if they do not correct it, the auditor should qualify his report.

22) Answer
As per NSA 230 on “Audit Documentations” the working papers are the property of the auditor and the
auditor has right to retain them. He may at his discretion can make available working papers to his client.
The auditor should retain them long enough to meet the needs of his practice and legal or professional
requirement. Working papers are the important records of the auditor. They serve as evidence of the
auditor‟s exercise of due care and conclusion reached regarding significant matters. The client does not
have a right to access the working papers and it is up to the discretion of the auditor to make them
available or not to others including the client.
Hence in the instant case, management of M/s Left & Right can‟t insist upon the auditor to handover the
working papers of the previous year.

23) Answer
The scope of audit may be limited for varied reasons, (i) the entity may impose restriction on scope of
audit, (ii) the limitation may be imposed by circumstances. When the audit is carried out under and as
per statute, the auditor should not accept the assignment when his duties are curtailed by agreement,
unless required by any Law. When audit is carried out in accordance with the entity‟s terms voluntarily,
the auditor may indicate his scope in his audit report.
Sometimes, the circumstances may impose restrictions on audit scope. For example, if the auditor is
appointed after the year end, he may not be able to participate in inventory checking. Or sometimes, the
records required may not be available so that the auditor may not be able to check details in the manner
he liked. Such limitations in scope may warrant an auditor to express disclaimer of opinion or qualified
opinion in his audit report depending upon the circumstances.
The non co-operation of ABC Limited will amount to limitation on scope of auditors.

RTP-CAP II –-2014-December@ICAN Page 41 of 158


24) Answer
Financial
a) Net liability or net current liability position
b) Fixed term borrowings approaching maturity without realist prospects of renewal or repayment, or excessive
reliable on short term borrowings to finance long term assets.
c) Indications of withdrawal of financial support by debtors and other creditors
d) Negative operation cash flows indicated by historical or prospective financial statements.
e) Adverse key financial ratios
f) Substantial operating losses or significant deterioration in the value of assets used to generate cash flows.
g) Arrears or discontinuance of dividends
h) Inability to pay creditors on due dates
i) Inability to comply with the terms of loan agreements
j) Change from credit to cash-on-delivery transaction with suppliers
k) Inability to obtain financing for essential new product development or other essential investments.
Operating
a) Loss of key management with replacement
b) Los of major market, franchise, license or principal supplier
c) Labor difficulties or shortage of important supplies
Other
a) Non compliance with capital and other statutory requirements
b) Pending legal or regularity proceedings against the entity that may, if successful, result in claims that are
unlikely to be satisfied
c) Changes in legislation or governmental policy expected to adversely affect the entity

25) Answer
Reliance on the Work of Internal Auditor: The external auditor should as a part of his audit, carryout general
evaluation of the internal audit function to determine the extent to which he can place reliance upon the work of
the internal auditor. As per NSA 610 “Using the Work of Internal Auditors ", factors that may affect the external
auditor‟s determination of whether the work of the internal auditors is likely to be adequate for the purposes of the
audit include:
Objectivity
 The status of the internal audit function within the entity and the effect such status has on the ability of the
internal auditors to be objective.
 Whether the internal audit function reports to those charged with governance or an officer with appropriate
authority, and whether the internal auditors have direct access to those charged with governance.
 Whether the internal auditors are free of any conflicting responsibilities.
RTP-CAP II –-2014-December@ICAN Page 42 of 158
 Whether those charged with governance oversee employment decisions related to the internal audit function.
 Whether there are any constraints or restrictions placed on the internal audit function by management or those
charged with governance.
 Whether, and to what extent, management acts on the recommendations of the internal audit function, and
how such action is evidenced.
Technical competence
 Whether the internal auditors are members of relevant professional bodies.
 Whether the internal auditors have adequate technical training and proficiency as internal auditors.
 Whether there are established policies for hiring and training internal auditors.
Due professional care
 Whether activities of the internal audit function are properly planned, supervised, reviewed and documented.
 The existence and adequacy of audit manuals or other similar documents, work programs and internal audit
documentation.
Communication: Communication between the external auditor and the internal auditors may be most
effective when the internal auditors are free to communicate openly with the external auditors, and:
Meetings are held at appropriate intervals throughout the period;
 The external auditor is advised of and has access to relevant internal audit reports and is informed of any
significant matters that come to the attention of the internal auditors when such matters may affect the work
of the external auditor; and
 The external auditor informs the internal auditors of any significant matters that may affect the internal audit
function
The degree of reliance that a statutory auditor can place on the work done by the internal auditor is also a
matter of individual judgement in a given set of circumstances. The ultimate responsibility for reporting
on the financial statements is that of the statutory auditor. It must be clearly understood that the statutory
auditor‟s responsibility is absolute and any reliance he places upon the internal audit system is part of his
audit approach or technique and does not reduce his sole responsibility.

26) Answer
Following instructions are given by the auditor to the client before the start of audit:
(i) The accounts should be total up and trial balance and final accounts to be kept ready.
(ii) Vouchers should be serially arranged.
(iii) Schedule of debtors and creditors should be prepared.
(iv) Schedule of outstanding expenses, prepaid expenses and accrued income to be kept ready.
(v) A list of bad and doubtful debts should be prepared.
(vi) Schedule of investments should be prepared.
(vii) Certified list of goods returned to be prepared.
(viii) Statement of permanent capital expenditure to be prepared.

RTP-CAP II –-2014-December@ICAN Page 43 of 158


(ix) Schedule of deferred revenue expenditures to be prepared.
(x) Names and addresses of managers and other officers should be kept ready.

27) How will you vouch the following?


a) Answer
(i) Examine the Board‟s Minute Book approving the purchase on hire-purchase terms.
(ii) Examine the hire-purchase agreement carefully and note the description of the machinery, cost of the
machinery, hire purchase charges, terms of payment and rate of purchase.
(iii) Assets acquired under Hire Purchase System should be recorded at the full cash value with corresponding
liability of the same amount. In case cash value is not readily available, it should be calculated presuming
an appropriate rate of interest.
(iv) Hire purchased assets are shown in the balance sheet with an appropriate narration to indicate that the
enterprise does not have full ownership thereof. The interest payable along with each installments, whether
separately or included therein should be debited to the interest account and not to the asset account.

b) Answer
Payments on account of salaries and wages need to be vouched carefully, since amounts which were
either not due or in excess of those due may have been paid by the client. The evidence in support of
such payments generally is internal. It can, therefore, be relied upon only if it has been produced in the
normal course of business and there exists an efficient
system of internal control which could be expected to prevent it from being fabricated.
Therefore, before proceeding to verify payment made on account of salaries and wages, the auditor
should examine the internal control procedure as regards the following:
a) Appointment, promotion, transfer and discharge of employees.
b) Recording attendance of workers engaged on the time basis, as well as particulars of jobs performed by piece
workers.
c) Arrangement for the preparation of wages and salaries bills and their analysis.
d) Sanctioning the disbursement of wages and salaries.
e) Arrangement for disbursement of wages and salaries for workers and employees not present on the pay day.
f) Custody of the wages records.

c) Answer
The advertisement expenses will be vouched in the following manner :
a) Ascertain the nature of advertisement expenses to ensure that the same have been charged properly.

RTP-CAP II –-2014-December@ICAN Page 44 of 158


b) Obtain the complete list of advertisement, media wise, i.e., newspapers, slides, hoardings, magazines,
television, radio, etc. showing the dates, exact location, timings, etc., along with the amounts paid in respect
of each category.
c) See that advertisement expenses relate to the client‟s business.
d) Ascertain whether there is a regular contract with an advertising agency. See that regular statements are
obtained from the agency showing the advertising media and amounts debited to the client. Discounts, if any,
should be properly adjusted and disclosed in the bills.
e) Check the receipts for amounts paid for the advertising expenses incurred.
f) See that outstanding advertising expenses have been properly disclosed on the liabilities side of the balance
sheet.

d) Answer
In this case also, as in the case of sale of investments, the authority for sale is most important. It is,
therefore, a matter which should receive the attention of the auditor. Another important aspect which
requires consideration is the basis of sale, whether by auction or by negotiation, for determining that the
asset was sold at the maximum price that could be contained for it and that the sale proceeds of the asset
have been fully accounted for. It should further be confirmed that sale proceeds have been credited to an
appropriate head of account and the amount of profit arising out of it has been segregated between
revenue profits and capital profits, if any, accordingly appropriate accounts are credited, where there is a
loss, the same should be written off.

e) Insurance
Insurance claims may be in respect of fixed assets or current assets. While vouching the receipts of
insurance claims, the auditor should examine a copy of the insurance claim lodged with the insurance
company. Correspondence with the insurance company and the insurance agent should also be seen.
Counterfoils of the receipts issued to the insurance company should also be seen. The auditor should
also determine the adjustment of the amount received in excess or short of the value of the actual loss as
per the insurance policy. The copy of certificate/report containing full particulars of the amount of loss
should also be verified. The accounting treatment of the amount received should be seen particularly to
ensure that revenue is credited with the appropriate amount and that in respect of claim against an asset,
the profit and loss account is debited with the shortfall of the claim admitted against the book value. If
the claim was lodged in the previous year but no entries were passed, entries in the profit and loss
account should be appropriately described.

RTP-CAP II –-2014-December@ICAN Page 45 of 158


28) Write short note on following:
a) Answer
An independent audit whether performed in terms of relevant statutory legislation or in terms of the engagement,
the auditor has to be reasonably satisfied as to whether the information contained in the underlying accounting
records and other source data is reliable for the preparation of financial statements. Since the entire process of
auditing is based on the assessment of judgments made by the management of the entity as well as evaluation of
internal controls, the audit suffers certain inherent risks. Factors which may cause such risk in conducting an audit
are discussed below:
(i) Exercising judgment on the part of the auditor: The auditor‟s work involves exercise of judgment, for
example, in deciding the extent of audit procedures and in assessing the reasonableness of the judgments and
estimates made by management in preparing the financial statements.
(ii) Nature of audit evidence: The auditor normally relies upon persuasive evidence rather than conclusive
evidence. Even in circumstances where conclusive evidence is available, the cost of obtaining such evidence
may far exceed the benefits.
(iii) Inherent limitations of internal control: Internal control can provide only reasonable, but not absolute,
assurance on account of several inherent limitations such as potential for human error, possibility of
circumstances of control through collusion, etc.
On account of above, it is quite nature that an audit suffers from control risk on account of inherent limitations
of internal control risk and detection risk on account of test nature of audit and judgment and estimates
involved in formulating accounting policies.

b) Answer
Audit note book contains large variety of matters observed during the course of audit. Significant matters
observed during audit which should be recorded in audit note book are normally the following:
(i) Audit queries not cleared immediately.
(ii) Mistakes or irregularities observed during the course of audit.
(iii) Unsatisfactory book-keeping arrangements, costing method.
(iv) Important information about the company which is not apparent from the accounts.
(v) Special points requiring consideration at the time of verification of annual accounts.
(vi) Important matters for future reference

c) Answer

While adopting test check technique, an auditor should take following precautions:-
i. The transactions of the concern should be classified under appropriate heads and may be stratified in case of
wide variations between the transactions of the same kind.
ii. Authorizations, documentations, recording of the transactions should be studied right from the beginning to
end.

RTP-CAP II –-2014-December@ICAN Page 46 of 158


iii. Evaluating the system of internal control for its efficiency, soundness and capability to produce reliable
accounting and financial data.
iv. Preparation of test check plan with clear audit objective understood by the audit staff.
v. Un-biased selection of the transactions with reference to the random number tables or other statistical
methods.
vi. Identification of the areas where test check may not be done.
vii. Based on degree of reliance and the confidence level required in the audit, the number of transactions to be
selected for each test plan should be pre-determined.
viii. Setting up criteria to judge what constitute material or immaterial errors. Further investigation of only
material errors be carried out and all immaterial errors may be avoided.

d) Answer
The auditor can use the computer to test: (a) the logic and controls existing within the system and (b) the
records produced by the system. Depending upon the complexity of the application system being audited,
the approach may be fairly simple or require extensive technical competence on the part of the auditor.
There are several circumstances where auditing through the computer must be used:
1. The application system processes large volumes of input and produces large volumes of output that
make extensive direct examination of the validity of input and output difficult.
2. Significant parts of the internal control system are embodied in the computer system. For example, in
an online banking system a computer program may batch transactions for individual tellers to provide
control totals for reconciliation at the end of the day‟s processing.
3. The logic of the system is complex and there are large portions that facilitate use of the system or
efficient processing.
Because of cost-benefit considerations, there are substantial gaps in the visible audit trail. The primary
advantage of this approach is that the auditor has increased power to effectively test a computer system.
The range and capability of tests that can be performed increases and the auditor acquires greater
confidence that data processing is correct. By examining the system‟s processing the auditor also can
assess the system‟s ability to cope with environment change.
The primary disadvantages of the approach are the high costs sometimes involved and the need for
extensive technical expertise when systems are complex. However, these disadvantages are really
spurious if auditing through the computer is the only viable method of carrying out the audit

e) Answer
Peer Review means an examination and review of the system and procedures to determine whether they
have been put in place by the practice unit for ensuring the quality of attestation service as envisaged and

RTP-CAP II –-2014-December@ICAN Page 47 of 158


implied/mandated by the Technical Standards and whether these were effective or not during the period
under review.
Peer Review is directed towards maintenance as well as enhancement of quality of attestation services
and to provide guidance to members to improve their performance and adhere to various statutory and
other regulatory requirements. Essentially, through a review of attestation services engagement records.
Peer review identifies the areas where a practicing member may require guidance in improving the
quality of his/her performance and adherence to various requirements as per applicable Technical
Standards.
The main objective of Peer Review is to ensure that in carrying out their attestation services assignments;
the members of the Institute (a) comply with the Technical Standards made mandatory for application by
the Institute and (b) have in place proper systems (including documentation systems) for maintaining the
quality of the attestation services work they perform. The Council has specified in this statement on Peer
Review does not seek to redefine the scope and authority of the Technical Standards promulgated by the
respective Standards Boards but seeks to ensure their implementation both in letter and spirit.

f) Answer
Propriety audit stands for verification of transaction on the test of public interest, commonly accepted customs and
standards of conduct. The propriety audit is concerned with scrutiny of executive actions and decisions bearing on
financial and profit and loss situation of the company with special regard to public interest and commonly
accepted customs, and standards of conduct. It is also seen whether every officer has exercised the same vigilance
in respect of expenditure of his own money under similar circumstances. Propriety requires the transaction, and
more particularly expenditure, to confirm to certain general principles. These principles are:

1. That the expenditure is not prima facie more than the occasion demands and that every official exercise the
same degree of vigilance in respect of expenditure as a person of ordinary prudence would exercise in respect
of his own money.
2. That the authority exercise its power of sanctioning expenditure to pass an order which will not directly
indirectly accrue to its own advantage;
3. That funds are not utilized for the benefit of a particular person or group of persons and
4. That, apart from the agreed remuneration or reward, no other avenue is kept open to indirectly benefit the
management personnel, employees and others.

g) Answer
The concept of independence is equally relevant for internal auditor also. Internal auditing is an
independent, objective assurance and consulting activity designed to add value and improve an
organisation‟s operations. Internal auditor is part of the management but he evaluates the functioning of
RTP-CAP II –-2014-December@ICAN Page 48 of 158
the management at different levels.
Therefore, to be efficient and effective, the internal auditor must have adequate independence. It may
be noted that by its very nature, the internal audit function cannot be expected to have the same degree of
independence as is essential when the external auditor expresses his opinion on the financial information.
To ensure his independence he is made
responsible directly to the Board of Directors through audit committee. Such a channel of
communication provides an independent mode whereby an internal auditor can communicate and share
his views on the scope of internal audit, findings, etc. If internal auditor is made subordinate to lower
level, his independence will be effected which will affect his functioning and effectiveness. An outsider,
like a firm of chartered accountants, if acting as internal auditor, is likely to be more independent than an
employee of the organization.

Corporate and Other Laws


Revision Test Paper CAP-II
Question and Answer

Contract Act, 2056


1. A lost his bag with important document in public place. When it came to his knowledge, A by
publishing an advertisement in Kantipur National Daily advances a proposal in public to the effect
that he will pay Rs. 50,000 to any person who finds out the bag within seven days. Mr. X, Y, Z
while walking together found the beg and delivered it to A and having knowledge of the
advertisement they all claimed with A for prize. Advice Mr. A how to settle the matter referring the
provision of Contract Act, 2056.

Answer:

Section 10 of the Contract Act, 2056 deals with the issue mentioned in the question.

Section 10 of the Contract Act


Contract according to proposal presented before the public: (1) In case any person, by means of an
advertisement, advances a proposal in public to the effect that he/she will pay a specific prize or any
person for performing any work specified in the advertisement, and in case any person performs the work
as mentioned in the advertisement, the advertiser shall pay the specific remuneration to that person.
(2) In case the work mentioned in Sub-section (2) is performed by one or more persons, only the person
who has performed the work first shall be paid prize.
Provided that, in case two or more persons have performed the work mentioned in the proposal at the
same time, all of them may share the prize among themselves. In case the prize can not be shared, the
proceeds of the sale of the same shall be equally distributed among them.
(3) In case a specific period has been prescribed for performing the work as per the advertisement
published under Sub-section (1), the proposal, mentioned in the advertisement shall be deemed to have
been cancelled immediately after the expiry of that period.

RTP-CAP II –-2014-December@ICAN Page 49 of 158


(4) A proposal advanced under Sub-section (2) may be cancelled through the medium through which it
was published.
(5) Notwithstanding anything contained in Sub-section (4), in case anyone has, performed the work
mentioned in the advertisement published under Sub - section (1) before the publication of a notice of
cancellation of the proposal, he/she shall be paid the prize mentioned in the advertisement.
Provided that, the person who has performed the work as mentioned in the advertisement must have
notified the advertiser about the completion of the work by the quickest possible means.
(6) In case any person has begun the work according to the advertisement published under Sub-section
(1) and furnished a notice thereof to the advertise, he/she shall be paid an appropriate remuneration for
the work, performed by him/her before the cancellation of the advertisement.

As per above provision in case two or more persons have performed the work mentioned in the proposal
at the same time, all of them may share the prize among themselves. So, Mr. X, Y, Z may claim the
prize and share among themselves.

Negotiable Instruments Act, 2034


2.
When does notice of dishonour unnecessary under Negotiable Instruments Act, 2034.

Answer

As per section 70 of the Negotiable Instruments Act, 2034, following are the cases in which the Notice of
Dishonour is unnecessary

(a) When it is dispensed with by the party entitled thereto,


(b) When the Drawer has countermanded payment,
(c) When the party has not suffered damage for want of notice,
(d) When the acceptor is also a Drawer,
(e) In the case of a Promissory Note which is not negotiable.
(f) When the party entitled to notice cannot after due search be found, or the party bound to give notice,
is for any other reason, unable without any fault of his/her own to give it,
(g) When the party entitled to notice, knowing the facts, promises unconditionally to pay the amount due
on the Negotiable Instrument.

Negotiable Instruments Act, 2034.


3.
A cheque payable to bearer is crossed generally and marked “not negotiable”. The cheque is lost or
stolen and comes into possession of B who takes it in good faith and gives value for it. B deposits the
cheque into his own bank and his banker presents it and obtains payment for his customer from the bank
upon which it is drawn. The true owner of the cheque claims refund of the amount of the cheque from B.
Discuss Fully under Negotiable Instruments Act, 2034
Answers
The cheque in the given case was crossed generally and marked „Not Negotiable‟. Thereafter, the
cheque was lost or stolen and came into the possession of B, who takes it in good faith and gives
value for it.

Section 89 of the Negotiable Instruments Act, 2034 provides that a person taking a cheque crossed
generally or specially, bearing in either case the words „not negotiable‟, shall not have, and shall not
RTP-CAP II –-2014-December@ICAN Page 50 of 158
be capable of giving a better title to the cheque than that which the person from whom he took it had.
In view of these provisions, B, even though he was a holder in due course, did not acquire any title to
the cheque as against its true owner. The addition of the words „not negotiabl e‟ entirely takes away
the main feature of negotiability, which is, that a holder with a defective title can give a good title to
a subsequent holder in due course. B did not obtain any better title than his immediate transferor,
who had either stolen or found the cheque and was not the true owner of the cheque. Therefore, as
regards the true owner, B was in no better position than the transferor. B is also liable to repay the
amount of the cheque to the true owner. He can, however, proceed against the p erson from whom he
took the cheque.

In the given case, both the collecting banker and the paying bankers would be exonerated. Since the
collecting banker, in good faith and without negligence, had received payment for B, who was its
customer of the cheque which was crossed generally, the banker would not be liable, in case the title
proved to be defective, to the true owner by reason only of having received the payment of the
cheque for his customer (Section 90 of the Act).

Companies Act, 2063


4. XYZ Company decided to have election for constituting new Board of Directors of the company.
The articles of association of the company is silent about number of shares required to be held by
a person for his appointment as director of the company. Mr. A having 90 number of shares of
the company filed his candidacy for appointment of director of the company. The election officer
refused his application. Do you think the decision of election officer valid? Is there any exception
where share qualification is not required for being appointed in the post of director of the
company? Answer with reference to the Companies Act 2063.

Answer:
Section 88 of Companies Act deals with share qualification of directors. Under the section i f the
articles of association of a company specify any number of shares required to be held by a person for
his appointment as director of the company, the person who becomes director shall hold such number
of shares. Failing any provision specifying such number of shares, any such person shall hold at least
one hundred shares.
As per above provision the refusal of election officer is valid, as Mr. A does not hold one hundred
shares.

Exceptions
The section further says -provided, however, that any director who is appointed pursuant to Sub-section
(3) of Section 86 and Sub-section (2) of Section 87 shall not be required to hold such shares.

Section 86(2) and (3)- (2) Every public company shall have a board of directors consisting of a
minimum of three and a maximum of eleven directors.
(3) In forming the board of directors pursuant to Sub-section (2), at least one independent director, in
the case of the number of directors not exceeding seven, and at least two independent directors, in
the case of the number of directors exceeding seven, shall be appointed from amongst the persons
who have the knowledge as prescribed in the articles of association of the company and gained
knowledge and experience in the subject related with the business of the company concerned.

Section 87(2) If the office of any director appointed by the annual general meeting is vacated for any
reason, the board of directors shall appoint another director in that vacancy.
RTP-CAP II –-2014-December@ICAN Page 51 of 158
Companies Act, 2063
5. Miss Richa, chartered accountant licensed by ICAN has been appointed the auditor of XYZ Trading
Pvt. Ltd. for the fiscal year of 2071/2072. It has been revealed that he was appointed by the general
meeting of the fiscal year of 2070/071 on the basis of recommendation of her Maternal Uncle's Son
who has been working in the company as General Manager since last several years. The appointment
was questioned later that the close relative of employee cannot be appointed as auditor in the
company. Discuss referring to Companies Act, 2063 the validity of his appointment with
disqualification of the auditor along with its consequences.

Answer:
Under section 111 of the Companies Act, the auditor of accompany shall be appointed, from amongst the
auditors licensed to carry out audit under the prevailing law, by the general meeting, in the case of a
public company. The auditor appointed shall hold office only until the next annual general meeting.
Section 112 specifies the disqualifications of auditor as follows: (1) None of the following persons or the
firms or companies in which such persons are partners shall be qualified for appointment as auditor and
shall, despite appointment as auditor, continue to hold office:
(a) A director, advisor appointed with entitlement to regular remuneration or cash benefit, a person or
employee or worker involved in the management of the company or a partner of any of them or and
employee of any of such partners or a close relative of a director or partner, out of them, or and employee
of such relative;
(b) A debtor who has borrowed moneys from the company in any manner, or a person who has failed to
pay any dues payable to the company within the time limit and is in such arrears or close relative of such
person;
(c) A person who has been sentenced to punishment for an offense pertaining to audit and a period of
three years has not elapsed thereafter;
(d) A person who has been declared insolvent;
(e) A substantial shareholder of the company or a shareholder holding one percent or more of the paid up
capital of the company or his close relative;
(f) A person who has been sentenced to punishment for an offense of corruption, fraud or a criminal
offense involving moral turpitude and a period of five years has not elapsed thereafter;
(g) A person referred to in Sub-section (3) of Section 111;
(h) In the case of a public company , any person who works, whether full time or part time , for any
governmental body or any body owned fully or partly by the Government of Nepal or any other company
or a partner of such person or a person who is working as an employee of such partner or a person who is
authorized to sign any documents or reports to be prepared by the management of the company;
(i) A company or corporate body with limited liability;
(j) A person having interest in any transaction with the company or his/her close relative or a director,
officer or substantial shareholder of another company having any interest in any transaction with the
company.

Section 2(z9) defines the term close relative- “Close relative” means a partition shareholder in joint
family or husband, wife, father, mother, mother-in -law, father-in- law, elder brother, younger brother,
elder sister, younger sister, sisterin –law ,(elder or younger brother‟s wife), brother-in– law , sister–in–
law, brother-in- law, (husband of elder sister), uncle, aunt, maternal uncle, maternal aunt, son, daughter,
daughter-in-law ,grand–son, grand-daughter, grand-daughter-in– law or son-in– law .

On basis of above legal provision Maternal Uncle's Son does not contain under the definition of close
relative, Miss Richa is not disqualified to be appointed as director.

RTP-CAP II –-2014-December@ICAN Page 52 of 158


Consequences
Section 112 further states that the auditor shall, prior to his/her appointment, give information in writing
to the company that he/she is not disqualified. Where any auditor becomes disqualified to audit the
accounts of a company or there arises a situation where he/she becomes disqualified for appointment or
can no longer continue to act as an auditor of the company, he/she shall immediately stop performing
audit which is required to be performed or is being performed by him/her and give information thereof to
the company in writing. The audit performed by an auditor who has been appointed in contravention of
this Section shall be invalid.
Under Section 160(k) An auditor who knowingly carries out auditing of the concerned company even
after that he/she is not qualified to carry out auditing of any company; shall be punished with a fine from
twenty thousand rupees to fifty thousand rupees or with imprisonment for a term not exceeding two years
or with both punishments.

Insurance Act, 2049


6
The Insurance Board cancelled the registration of XYZ Insurance Company as it was felt that the liability
of the company exceeded its assets within Nepal. The employees of the company claimed their
remuneration and other outstanding amounts. Other creditors of the company also claimed their due and
argued that they should be paid before employees. Advice the employee and creditor about the payment
of their claims under Insurance Act, 2049.

Answer
Under section 13(1)(b) of Insurance Act, the Board may cancel the registration of an Insurer by
providing a written notice with effect from the date prescribed in the same notice if it is felt that the
liability of the Insurer exceeds its assets within Nepal,
Under section 41B of the Act, If any Insurer is dissolved due to the cancellation of its registration
pursuant to Section 13, the liabilities shall be settled in the following order of priority :-
(a) The expenses incurred for the dissolution,
(b) The amount to be paid against the insurance claims to the Insured pursuant to Section 16,
(c) The remuneration and other outstanding amounts to be obtained by the employees of the Insurer,
(d) Loan amounts,
(e) The amount to be paid to the Board,
(f) The amount to be paid to the Government of Nepal.

So, in case of dissolution of insurer due to cancellation of registration, the payment shall be made on
basis of above order of priority. As per the order of priority, the payment of loan amount is made only
after payment is made to the employee of the insurer. So, the creditors shall be paid after payment is
made to employees. So Creditor cannot argue that they should be paid before employees.

Labour Act, 2048


7
a. Who are not entitled to participate on strike? How can the dispute relating to such employees
or workers be settled under Labour Act, 2048?

Answer

Under section 78 of the Labour Act, 2048, following workers or employees are not entitled to
participate on strike

RTP-CAP II –-2014-December@ICAN Page 53 of 158


a) if any existing law has prohibited the strike to be done by workers or employees the
workers or employees of such Enterprise.

b) Any employee appointed or deputed on the duty of control, security and guard of any
Enterprise

Dispute Settlement
Under sub section (3) of the Section, the employees prohibited to strike may submit their
genuine demands to the Proprietor. If such demands are not fulfilled and a dispute has
been created Government of Nepal shall constitute a tribunal for solving it. The decision
of the Tribunal shall be final and binding upon both of the parties.

Companies Act, 2063


8. Discuss Civil and Criminal Liability to be imposed on the Directors under Companies Act, 2063 in
respect of containing false statement in the prospectus and also discuss defense if any in this regard.

Answer:

Under Section 24 of Companies Act, It shall be the duty and obligation of the concerned company to
abide by the matters contained in the prospectus. The directors who have signed the prospectus shall be
liable for the matters mentioned in that prospectus .

Civil Liability

The section further says- If any published prospectus contains false statements made maliciously or
deliberately and any person sustains any loss or damage by reason of his/her subscription of securities on
the faith of that prospectus, the directors who have signed that prospectus shall be personally liable to
pay compensation for the actual loss or damage so sustained.

Defense

A director who resigns before the decision made by the company to publish the prospectus or whom on
becoming aware of any false statement in the prospectus, publishes a notice of that matter to the
information of the general public prior to the sale or allotment of securities or who proves that he/she did
not know that the prospectus contained any false statement shall not be liable to bear such compensation.

Criminal Liability
Section 160 of the Act deals with criminal liability in this regard. Any director or officer who issues the
prospectus of a company prior to its being registered with the Office or who gives false details in the
prospectus shall be punished with a fine from twenty thousand rupees to fifty thousand rupees or with
imprisonment for a term not exceeding two years or with both punishments.

Companies Act, 2063

9. What are the grounds in which auditor of a company may be removed before completion of audit of
accounts of any fiscal year for which s/he was appointed as the auditor. Discuss referring the
Companies Act, 2063.

Answer:
RTP-CAP II –-2014-December@ICAN Page 54 of 158
Under Section 119 of Companies Following are the grounds in which auditor of a company may be
removed before completion of audit of accounts of any fiscal year for which s/he was appointed as the
auditor.
1) if any auditor breaches the code of conduct of auditors or
2) If s/he does any act against the interest of the company which has appointed him as the auditor or
3) If s/he commits any act contrary to the prevailing law,

Companies Act, 2063

10. Who are disqualified to be a promoter of the company under Companies Act, 2063. A, a minor who
wants to purchase the share of XYZ Company, advice the minor under Companies Act, 2063.

Answer:
Under Section 66 of the Companies Act
A minor who has not attained the age of sixteen years and a person who is not qualified to make contract
under the law shall not be eligible to become the promoter of a company. Provided, however, that this
Section shall not be deemed to prevent a minor or a person who is disqualified under the law to make
contract from acquiring the title to the shares of a company subscribed by the promoter, by virtue of
succession or operation of law.

Where any minor or any person who is disqualified under the law to make contract is to purchase or sell
any shares or debentures of any company, the father, mother or husband or wife or legally appointed
protector or guardian of such person shall do so.

11.
Write Short Notes on following.

Insurance Act, 2049


a) Principle of Proximate Cause

Answer:
This is the principle of insurance. This rule is that immediate and not the remote cause is to be regarded. The real
cause must be seen while payment of the loss. If the real cause of loss is insured, the insurer is liable to
compensate the loss, otherwise the insurer may not be responsible for loss.If the loss or damage is created by more
than one cause, it should be found out which cause is the nearest cause for that event included in the policy.If it is
the risk out of the policy, certainly the insurer is not compelled to provide the compensation.If there are many
causes of the event, insurer should pay attention to the proximity cause.i.e.Nearest cause of happening the event.
Determination of proximate Cause
1) If there is a single cause of the loss, the cause will be the proximate cause and further if the peril(cause of
loss) was insured, insurer will have to indemnify the loss.
2) If there are concurrent causes, the insured perils and excepted perils have to be segregated.
3) If the causes occurred in form of chain, they have to be observed seriously.

RTP-CAP II –-2014-December@ICAN Page 55 of 158


Industrial Enterprise Act 2049,
b) Export Promotion Industry
Answer :
Under Section 2 of Industrial Enterprise Act 2049, Export Promotion Industry means any prescribed
industry established with the objective of exporting eighty or more than eighty percent of its product.
Sec 15: no tax fee or charge of any kind shall be levied on the machine, tools equipment, machinery
and raw material to be employed by an export promotion industry as well as on the product of such
industry.
Provided that if, for any reason, any product of such industry is required to be sold within the
country, tax fee or charge thereof shall be required to be paid on the basis of the quantity of sale.
Under Sec 16: Nepal government may by notification published in the Nepal Gazette, grant
additional facilities to the Export promotion industry.

Securities Act, 2063.

12. Discuss the conditions where any information or notice shall be deemed to have been made public
under Securities Act, 2063.

Answer
Under Section 13 of the Securities Act, 2063, On any of the following conditions, any information or
notice shall be deemed to have been made public:
(a) If any matter has been published with intent to inform the investors and their business advisers in
accordance with the Bye-laws of a stock exchange,
(b) If there is a provision made under the law that the general public can see an information or a notice
contained in any records,
(c) If there is a provision that any person desiring to deal in securities is escorted to the business room of
a stock exchange so that such a person can know such an information or a notice,
(d) If there is a provision that any person desiring to obtain or see such information or notice or get a
copy thereof can obtain and see the same,
(e) If it has been communicated to any specific class, out of the general public,
(f) If there is a provision that information can be obtained only upon payment of fees or if such an
information or a notice has been published outside Nepal.

Nepal Rastra Bank Act, 2058


13.
How does the Government of Nepal appoint Governor of Nepal Rastra Bank ?

Answer
Section 15 of Nepal Rastra Bank Act, 2058
(1) Government of Nepal, the Council of Ministers shall appoint Governor on the basis of the
recommendation of the Recommendation Committee formed pursuant to Sub-section (2).
(2) Government of Nepal, the Council of Ministers shall, for the appointment of Governor, form a
Recommendation Committee as follows:-
(a) Minister of Finance -Chairperson
(b) One person from among the former Governors - Member
RTP-CAP II –-2014-December@ICAN Page 56 of 158
(c) One person designated by Government of Nepal from amongst the persons renowned in the fields of
Economic, Monitory, Banking, Finance and Commercial Law. -Member
(3) While making recommendation for the appointment of Governor, the committee formed pursuant to
Sub-section (2), shall recommend to Government of Nepal, the Council of Ministers the names of three
persons renowned in the field of economic, monetary, banking, finance, commerce, management,
commercial law and from among the Deputy Governors.
(4) Government of Nepal, the Council of Ministers shall, out of the names recommended pursuant to
Sub-section (3), appoint one person to the Office of Governor.

Nepal Rastra Bank Act, 2058


14.
Nepal Rastra Bank takes XYZ Bank under its control after suspending the board of directors as it
failed to honour liability and it has not been properly operated. It was later on found that the XYZ
Bank was able to honour its commitment and was in position to be operated properly. What action
may be taken by NRB under Nepal Rastra Bank Act, 2058?

Answer
Under Section 86 of Nepal Rastra Bank Act, 2058, the Bank may take such commercial bank or
financial institution under its control after suspending the Board of Directors of such commercial bank or
financial institution where any commercial bank or financial institution has failed to honour its liability
or there are probability of such failure or it has not been properly operated.
The Rastra Bank may take the management of such commercial bank or financial institution under its
control and operate its business by itself or appoint a person, firm, company or institution to operate or
manage the business of such commercial bank or financial institution.
The Rastra Bank shall, within one year of operating the business on its own or through any other person,
firm, company or institution audit or cause to audit the accounts of the concerned commercial bank or
institution and publish the report thereof.
The Rastra Bank may take following actions if the Bank is convinced from the auditing report that the
concerned commercial bank or financial institution is able to honour its commitment or such commercial
bank or financial institution is in position to be operated properly.
(a) To release the suspension of the Board of Directors of the commercial bank or financial institution
and to order the same Board of Directors to operate the business; or
(b) To dismiss the Board of Directors of the commercial bank or financial institute suspended and to
have the business conducted by a new Board of Directors formed from amongst the shareholders of the
commercial bank or financial institution;
(c) To get the new Board of Directors elected after convening the general meeting of the shareholder of
commercial bank or financial institution and to have the business conducted by it; or
(d) To take any other appropriate action as the Bank may deem fit.
The Rastra Bank shall, before taking the commercial bank or financial institution under its control,
provide an opportunity to defend within fifteen days to such commercial bank or financial institution as
per the situation.
The concerned commercial bank or financial institution shall bear all expenses incurred while operating
the commercial bank or financial institution after the Bank has taken over pursuant to this section.

Banks and Financial Institutions Act, 2063


15. Discuss the circumstances where the secrecy shall not be deemed to have been violated under
BAFIA, 2063.

RTP-CAP II –-2014-December@ICAN Page 57 of 158


Answer

Under Section 79 of BAFIA Notwithstanding anything contained in the laws in force, the relationship
between a licensed institution and its customers and information pertaining to its accounts, records,
books, ledgers and statements shall not be disclosed to any person other than the concerned person.
However, if such information is provided in any of the following circumstances, the secrecy shall not be
deemed to have been violated:
(a) If it is provided to the Rastra Bank pursuant to the Nepal
Rastra Bank Act, 2058 (2002) or this Act or the Rules or Bye-laws framed thereunder or the orders or
directives issued thereunder;
(b) If it is provided to a court in connection with any lawsuit or any other legal action;
(c) If it is provided to the legally authorized official making inquiries and investigations or prosecution,
in connection with such inquiries and investigations or prosecution;
(d) If it is provided to the auditor in connection with audit;
(e) If it is mentioned in the audit report by the auditor;
(f) If it is provided to the enquiry committee or the investigation officer authorized by the laws in force,
in the course of enquiries;
(g) If banking information is exchanged between two or more countries on the basis of a bilateral or
multilateral treaty;
(h) If it is provided in the course of exchanging credit between the licensed institutions;
(i) If, in the course of investigation of any case of specific nature, the Government of Nepal, Ministry of
Finance, has made a request, accompanied by the reason therefor, for providing the details of the account
of the licensed institution or any details of the account of any person, firm, company or institution
maintained with the licensed institution, and if the Rastra Bank has given direction for the same.
(3) The Government of Nepal, court, Rastra Bank, enquiry committee, investigating or prosecuting
official, auditor or inspector or any other person appointed under this Act shall not divulge the secrecy of
any information or particulars received under Sub-section (2).
(4) The directors, chief executive, employees, auditor and advisors of a licensed institution shall not
divulge the secrecy of the accounts, records, books, ledgers and any other information of the licensed
institution the secrecy of which is required to be maintained in such a manner as to undermine the
relationship between the licensed institution and its customers.

Industrial Enterprises Act, 2049


16.
When and How does Government of Nepal take action any person for violating any provision of
Industrial Enterprises Act.

Answer:
Under Section 25 of Industrial Enterprises Act Government of Nepal shall take any of the following
actions against any person for establishing any industry without obtaining permission required to be
obtained under this Act or for noncompliance with the terms and conditions set forth in the license or
certificate of registration or for violating any other provision of this Act:
(a) To impose a fine in an amount not exceeding five hundred thousand rupees,
(b) To cancel the registration or permission of the industry,
(c) To cause to close down the industry;

Before taking any action the concerned industry shall be given a reasonable time either to submit the
explanation or to correct the mistake.
If any industry is not satisfied with the decision make by Government of Nepal, it may file an appeal to
an Appellate Court within thirty five days of the notification thereof.
RTP-CAP II –-2014-December@ICAN Page 58 of 158
Foreign Investment and Technology Transfer Act, 2049

17. List out the industry where permission for making foreign investment shall not be granted under
Foreign Investment and Technology Transfer Act, 2049

Answer
Under Annex of Foreign Investment and Technology Transfer Act, 2049, following Industries are not to
be granted permission for making foreign investment.

Part (A)
1. Cottage Industries.
2. Personal Service Business (Business such as Hair Cutting, Beauty Parlour, Tailoring, Driving Training
etc.)
3. Arms and Ammunition Industries.
4. Explosives, Gunpowder
5. Industries related to Radio-Active Materials.
6. Real Estate Business (Excluding Construction Industries)
7. Motion Pictures Business (Produced in national languages and the language of the nation)
8. Security Printing
9. Currencies and Coinage Business

Part (B)
1. Retail Business
2. Travel Agency
3. Trekking Agency
4. Water Rafting
5. Pony Trekking
6. Horse Riding
7. Cigarette, Bidi (Tobacco), Alcohol (excluding those exporting more that 90%)
8. Internal Courier Service
9. Atomic Energy
10. Tourist Lodging
11. Poultry Farming
12. Fisheries
13. Bee-Keeping
14. Consultancy Services such as Management, Accounting, Engineering and Legal Services.
15. Beauty parlour
16. Domestic food processing methods in rent .
17. Local Catering Service
18. Rural tourism

Labour Act, 2048?


18. Where does the Labour Act, 2048 apply? Can a person be appointed in contractual service under
Labour Act, 2048?

Answer

RTP-CAP II –-2014-December@ICAN Page 59 of 158


The Labour Act is applicable in Enterprises(Establishment). Section 2(b) of the Act defines
Enterprises(Establishment) as under:
"Enterprise" means any factory, company, organisation, association, firm, or group thereof, established
under the prevailing laws for the purpose of operating any industry, profession or service, where ten or
more workers or employees are engaged and this expression also includes:
(1) Tea estates, established under the law for commercial purpose;
(2) Enterprise operating within the industrial districts established by Government of Nepal where less
than ten workers or employees are engaged.

Yes, under section 7 of the Labour Act, any establishment may appoint in contractual service.
If an Enterprise needs to increase in production or service immediately, any person may be appointed in
contract, by fixing a time period and setting forth the remuneration to be received by him/her and the
terms of service, for any specified work other than the works of the Enterprise which are permanent in
nature.

Bonus Act, 2030.


19. ABC Company has recently declared to pay bonus to its employee. A an employee of ABC
Company dies without getting bonus. His step mother living in an undivided family claimed the bonus to
be paid to him. Advice referring the Bonus Act, 2030.

Answer
Under section 10 of Bonus Act,
1. Any employee who is unable to present himself to take payment of bonus may depute
another person to accept the bonus due to him along with his consent in writing.
2. In case any employee dies, the bonus due to him shall be paid to any person who has been
designated so designated, or if the person so designated also dies, it shall be paid to the
surviving relatives of the deceased employee according to the following order:-
a) Husband or wife living in an undivided family.
b) Son living in an undifided family.
c. Unmarried daughter living in a undivided family.
d. Father, mother or grand-children on the son's side,
e. Husband or wife who is living separately,
f. Son who is living separately.
g. Unmarried daughter, father and mother who are living separately,
h. Grand-father or grand-mother on the father's side, elder brother, younger brother,
widowed daughter-in-low, and unmarried grand-daughter on the son's side living in an
undivided family,
i. Unmarried elder or younger sister, and married daughter living in an undivided family,
j. Nephew, niece and step-mother living in an undivided family, grand-son on the son's
side who is living separately, and unmarried grand-daughter,
k. Uncle or widowed aunt, or wife of elder brother, younger brother, or grand-son living in
an undivided family.
l. Elder or younger brothers who are living separately.
m. Married elder or younger sister, grand-father or grand-mother, widowed daughter-in-law,
grand-daughter, daughter-in-law, or nephew who are living separately.

In case several relatives have been placed in the same order, bonus shall be apportioned equally
among all or them.
The step mother as mentioned in the given case can claim as above.

20. Distinguish between


RTP-CAP II –-2014-December@ICAN Page 60 of 158
Insurance Act, 2049
a) Reinsurance and Double insurance.

Every insurance company has a limit to the risk it is willing to undertake in respect of an individual policy.
Thus, if an insurance company finds that it has entered into an insurance contract which is an expensive
proposition for it or if it wishes to minimize the chances of any possible loss, it will reinsure a portion of
the risk with some other insurance company or companies. This is known as re insurance.

When the same subject matter is insured with two or more insurers and the total sum insured exceeds the
actual value of the subject matter, it is known as double insurance and it amounts to over insurance. In case
of loss, the assured may claim payment from the insurers in such order as he thinks fit, but he will not get
more than his actual loss, as each contract of insurance is a contract of indemnity. The advantage of double
insurance is that it protects him against loss in the event of one or more of the insurers becoming insolvent,
he can recover up to the value of the policy from the solvent insurer. There is no double insurance in case
of life insurance.

Reinsurance Vs. Double insurance.


1. Re insurance business entered into by the original insurer with other insurers. But in double insurance
the insured gets the same subject matter insured with more than one insurer or under more than one
policy with the same insurer.
2. In reinsurance the insured cannot claim any part of his loss from the insurer. But in double insurance
the insured can claim only his actual loss from each of the insurers up to the amount insured with
them.
3. In reinsurance the reinsured will claim a part of the loss proportionate to the risk reinsured by him
with the reinsurers. But in double insurance each insurer is liable to contribute on prorate basis
towards loss suffered by the insured.

Audit Act, 2048


b) Corporate body wholly owned by government Between Corporate body substantially owned by
government
Corporate body wholly owned by government Corporate body substantially owned by
government
1 Sec 2(d) of Audit Act: "Corporate body wholly Sec2(e) of Audit Act: "Corporate body
owned by government of Nepal" means a substantially owned by government of Nepal"
corporate body whose all shares of assets are means a corporate body whose more than fifty
owned by Government of Nepal, or a corporate percent shares or assets are owned by
body whose all shares or assets are owned by the Government of Nepal.
aforesaid corporate body or by such corporate
body and Government of Nepal and this
expression shall also include such corporate body
for whom Government of Nepal is required to
bear full responsibility.
2 The audit of the corporate bodies wholly owned The audit of the corporate bodies substantially
by Government of Nepal shall be audited by the owned by Government of Nepal shall be done in
Auditor General. accordance with the prevailing laws relating to
such body.

RTP-CAP II –-2014-December@ICAN Page 61 of 158


3. If the Auditor General is constrained by time and The Auditor General shall be consulted while
resources to audit the corporate bodies wholly appointing an auditor for auditing of the
owned by Government he may appoint license corporate bodies substantially owned by
holder auditors an assistant. Government of Nepal. The procedures to be
The auditor appointed shall act under the followed while consulting the Auditor General
direction, supervision and control of the Auditor for appointing auditors and on matters to
General. The powers, functions, duties and principles of audit to be followed by the
responsibilities of the auditors appointed and the auditors during their audit shall be as prescribed
procedures to be followed in course audit and by the Auditor General.
provisions relating to their report shall be as
prescribed by the Auditor General.

4 The remuneration to be paid by the concerned The remuneration to be paid by the concerned
organization to the auditors appointed shall be organization to the auditors appointed shall be
fixed by the Auditor General keeping in view the fixed by the concerned organization.
volume of financial transactions, status of
accounts, number of branches and sub-branches,
work load and work progress of the concerned
organization.

Nepal Chartered Accountants, Act, 2053?


21. What are functions, duties and powers of council of ICAN under Nepal Chartered
Accountants, Act, 2053?

Answer:
Under section 11 of Nepal Chartered Accountants, Act, 2053, the functions, duties and powers of the
Council shall be as follows:-
(a) To conduct professional examinations for new entrants in accounting profession.
(b) To fix up procedures relating to the registration of members and members holding Certificate
of Practice.
(c) To provide membership of appropriate class to applicants having acquired qualifications
pursuant to Section 16.
(d) To optimum utilise available resources and means for the cause of development of accounting
profession.
(e) To determine appropriate qualifications for manpower engaged in the accounting profession.
(f) To fix curriculum and practical training period required for obtaining membership of the
Institute.
(g) To organize academic classes and training programmes by itself or in collaboration with any
university or other academic institutions.
(h) To issue Certificates of Practice to members willing to carry on accounting profession.
(i) To offer career development opportunities in order to enhance efficiency of members.
(j) To monitor as to whether or not the members and Members holding Certificate of Practice
have acted in conformity with the prescribed professional code of conduct.
(k) To initiate actions, in accordance with the recommendation of the Disciplinary Committee,
against members holding Certificate of Practice for their acts and actions done in
contravention of the professional code of conduct.
(l) To issue theoretical or practical guidance and guidelines in various aspects of accounting and
auditing and to carry out other professional development activities as and when needed.

RTP-CAP II –-2014-December@ICAN Page 62 of 158


(m) To monitor and regulate so as to ensure the compliance of Accounting Standards and
Standards on Auditing developed or recommended by Accounting Standards Board and
Standards on Auditing Board".
(n) To safeguard rights and interests of the members and to protect and promote their reputation.
(o) To render advice and suggestions to Government of Nepal for improvement in prevailing
laws related with industry, commerce, finance, revenue and accounting profession.
(p) To acquire membership of the International Federation of Accountants and Regional and Sub-
regional Federations, and establish contact with other foreign professional institutes.
(q) To recommend appropriate educational standards for account education, in consultation with
university or any other educational institutions.
(r) To organize, as and when necessary, training, symposia and seminars to enhance professional
efficiency of Registered Auditors.
(s) To conduct short-term or long-term training, workshops or symposia for the benefit of
manpower involved accounting profession.
(t) To publish materials related with accounting profession.
(u) To establish a library containing materials related to profession.
(v) To determine procedures to be followed by Committees constituted by the Council.
(w) To approve budget of the Institute and arrange for the fund.
(x) To recruit staff, as required, for the Institute and fix their remuneration and other perks.
(x1) To install the mechanism of providing Continued Professional Education (CPE) to members.
(x2) To develop education system to produce Accounting Technicians and undertake other
necessary actions in this regard.
(y) To carry out such other functions as prescribed by this Act or Regulations and Byelaws
framed under this Act.
(z) To carry out any other functions that deem to be necessary to attain the objectives laid down
by this Act.

FINANCIAL MANAGEMENT

1. SPG Ltd. has the following balances as on 1st of Shrawan 2071:

NRs.
Particulars Amount
Property, Plant & Equipment
Gross Block 1,140,000
Less: Accumulated 399,000
Depreciation
Net Block 741,000

Inventory 225,000
Receivables 250,000
Cash & Bank Balance 66,500
Payables 190,000
Share Capital 570,000

a. The company made the following estimates for the financial year 2071-72:
b. The company will pay a tax-free dividend of 10% the rate of tax being 5%.
c. The company will acquire PPE costing NRs. 190,000 after selling one machine for NRs. 38,000
costing NRs. 95,000 and on which depreciation provided amounted to NRs. 66,500.
d. At the end of the year, the company will have the following balances

RTP-CAP II –-2014-December@ICAN Page 63 of 158


NRs.
Particulars Amount
Inventory 210,500
Receivables 350,000
Payables 247,000
Profit after depreciation- NRs. 104,500
114,000

Prepare the projected cash flow statement and ascertain the bank balance of SPG Ltd. as on
32 Ashad 2072

2. The Board of OAP Co has decided to limit investment funds to NRs.10 million for the next year and
is preparing its capital budget. The company is considering five projects, as follows:

NRs.
Projects Initial investment Net present value

Project A 2,500,000 1,000,000


Project B 2,200,000 1,550,000
Project C 2,600,000 1,350,000
Project D 1,900,000 1,500,000
Project E 5,000,000 To be calculated

All five projects have a project life of four years. Projects A, B, C and D are divisible, and
Projects B and D are mutually exclusive. All net present values are in nominal, after-tax
terms.

Project E
This is a strategically important project, which the Board of OAP Co has decided must be
undertaken for the company to remain competitive, regardless of its financial acceptability.
Information relating to the future cash flows of this project is as follows:

Year 1 2 3 4
Sales volume (units) 12,000 13,000 10,000 10,000
Selling price (NRs./unit) 450 475 500 570
Variable cost (NRs./unit) 260 280 295 320
Fixed costs (NRs.’000) 750 750 750 750

These forecasts are before taking account of selling price inflation of 5% per year, variable
cost inflation of 6% per year and fixed cost inflation of 3·5% per year. The fixed costs are
incremental fixed costs, which are associated with Project E. At the end of four years,
machinery from the project will be sold for scrap with a value of NRs. 400,000. Tax
allowable depreciation on the initial investment cost of Project E is available on a 25%

RTP-CAP II –-2014-December@ICAN Page 64 of 158


reducing balance basis and OAP Co pays corporate tax of 28% per year, one year in arrears.
A balancing charge or allowance is available at the end of the fourth year of operation.
OAP Co has a nominal after-tax cost of capital of 13% per year.

Required:
a) Calculate the nominal after-tax net present value of Project E and comment on the financial
acceptability of this project.
b) Calculate the maximum net present value which can be obtained from investing the
fund of NRs. 10 million, assuming here that the nominal after-tax NPV of Project E is zero.

3. The current assets and current liabilities of CSZ Co at the end of March 2014 are as follows:

NRs.’000 NRs.’000

Inventory 5,700

Trade receivables 6,575 12,275

Trade payables 2,137

Overdraft 4,682 6,819

Net current assets 5,456

For the year to end of March 2014 CSZ Co had domestic and foreign sales of NRs. 40 million, all on
credit, while cost of sales was NRs.26 million. Trade payables related to both domestic and foreign
suppliers.
For the year to end of March 2015, CSZ Co has forecast that credit sales will remain at NRs.40 million
while cost of sales will fall to 60% of sales. The company expects current assets to consist of
inventory and trade receivables, and current liabilities to consist of trade payables and the
company’s overdraft.

CSZ Co also plans to achieve the following target working capital ratio values for the year to the end
of March 2015:

Inventory days 60 days


Trade receivables days 75 days
Trade payables days 55 days
Current ratio 1.4 times

Required:
a) Calculate the working capital cycle (cash collection cycle) of CSZ Co at the end of March 2014
and discuss whether a working capital cycle should be positive or negative.
b) Calculate the target quick ratio (acid test ratio) and the target ratio of sales to net working
capital of CSZ Co at the end of March 2015.

4. The equity beta of Fence Co is 0.9 and the company has issued 10 million ordinary shares. The
market value of each ordinary share is NRs.75. The company is also financed by 7% bonds with a
nominal value of NRs. 1000 per bond, which will be redeemed in seven years’ time at nominal value.
The bonds have a total nominal value of NRs. 140 million. Interest on the bonds has just been paid
and the current market value of each bond is NRs. 1071.4.
The risk-free rate of return is 4% per year and the average return on the stock market is 11% per year.
The corporate tax rate is 20% per year.
RTP-CAP II –-2014-December@ICAN Page 65 of 158
Required:
a) Calculate the current weighted average cost of capital of Fence Co.

5. Spot Co is considering how to finance the acquisition of a machine costing NRs. 750,000 with an
operating life of five years. There are two financing options.
Option 1
The machine could be leased for an annual lease payment of NRs. 155,000 per year, payable at the
start of each year.
Option 2
The machine could be bought for NRs. 750,000 using a bank loan charging interest at an annual rate
of 7% per year. At the end of five years, the machine would have a scrap value of 10% of the purchase
price. If the machine is bought, maintenance costs of NRs. 20,000 per year would be incurred.
Taxation must be ignored.

Required:
Evaluate whether Spot Co should use leasing or borrowing as a source of finance, explaining
the evaluation method which you use.

6. XYZ Co has annual sales revenue of NRs. 6 million and all sales are on 30 days’ credit, although
customers on average take ten days more than this to pay. Contribution represents 60% of sales and
the company currently has no bad debts. Accounts receivable are financed by an overdraft at an
annual interest rate of 7%.
XYZ Co plans to offer an early settlement discount of 1.5% for payment within 15 days and to extend
the maximum credit offered to 60 days. The company expects that these changes will increase
annual credit sales by 5%, while also leading to additional incremental costs equal to 0.5% of
turnover. The discount is expected to be taken by 30% of customers, with the remaining customers
taking an average of 60 days to pay.

Required:
Evaluate whether the proposed changes in credit policy will increase the profitability of XYZ Co.

7. The following financial information relates to VBBS Co.

Income statement extracts


NRs.’000
Particulars 2011 2010
Revenue 14,525 10,375
Cost of sales 10,458 6,640
Profit before interest and
tax 4,067 3,735
Interest 355 292
Profit before tax 3,712 3,443
Taxation 1,485 1,278
Distributable profit 2,227 2,165

Statement of financial position extracts


NRs.’000
2011 2010
Non-current assets 15,284 14,602
Current assets
RTP-CAP II –-2014-December@ICAN Page 66 of 158
Inventory 2,149 1,092
Trade receivables 3,200 5,349 1,734 2,826
Total assets 20,633 17,428

Current liabilities
Trade payables 2,865 1,637
Overdraft 1,500 4,365 250 1,887
Equity
Ordinary shares 8,000 8,000
Reserves 4,268 12,268 3,541 11,541
Long-term liabilities
7% Bonds 4,000 4,000

Total liabilities 20,633.00 7,428.00

Average ratios for the last two years for companies with similar business operations to VBBS
Co are as follows:

Current ratio 1·7 times


Quick ratio 1·1 times
Inventory days 55 days
Trade receivables days 60 days
Trade payables days 85 days
Sales revenue/net working capital 10 times

Using suitable working capital ratios and analysis of the financial information provided,
evaluate whether VBBS Co can be described as overtrading (undercapitalized).

8. PS Ltd. is a company that is listed on a major stock exchange. The company has struggled to
maintain profitability in the last two years due to poor economic conditions in its home country and
consequently it has decided not to pay a dividend in the current year. However, there are now clear
signs of economic recovery and PS Ltd. is optimistic that payment of dividends can be resumed in
the future. Forecast financial information relating to the company is as follows:

Year 1 2 3
Earnings (NRs.’000) 3,000 3,600 4,300
Dividends (NRs.’
000) nil 500 1,000

The company is optimistic that earnings and dividends will increase after Year 3 at a
constant annual rate of 3% per year.
PS Ltd. currently has a before-tax cost of debt of 5% per year and an equity beta of 1.6. On a
market value basis, the company is currently financed 75% by equity and 25% by debt.
During the course of the last two years, the company acted to reduce its gearing and was
able to redeem a large amount of debt. Since there are now clear signs of economic
recovery, PS Ltd. plans to raise further debt in order to modernize some of its non-current
assets and to support the expected growth in earnings. This additional debt would mean
that the capital structure of the company would change and it would be financed 60% by

RTP-CAP II –-2014-December@ICAN Page 67 of 158


equity and 40% by debt on a market value basis. The before-tax cost of debt of PS Ltd.
would increase to 6% per year and the equity beta of PS Ltd. would increase to 2.
The risk-free rate of return is 4% per year and the equity risk premium is 5% per year. In
order to stimulate economic activity the government has reduced profit tax rate for all large
companies to 20% per year.
The current average price/earnings ratio of listed companies similar to PS Ltd. is 5 times.

Required
a) Estimate the value of PS Ltd. using the price/earnings ratio method and discuss the usefulness
of the variables that you have used.
b) Calculate the current cost of equity of PS Ltd. and, using this value, calculate the value of the
company using the dividend valuation model.
c) Calculate the current weighted average after-tax cost of capital of PS Ltd. and the weighted
average after-tax cost of capital following the new debt issue, and comment on the difference
between the two values.

9. Entity C has monthly sales of NRs. 100,000. A factor has offered to take over the administration of
entity C’s trade receivables, on a non-recourse basis. It would charge a fee of 4% of the value of
invoices processed. If the factor takes over this work, entity C would save monthly administration
costs of NRs. 2,000 and would avoid bad debts, which are 0.75% of sales. Entity C has been informed
by the factor that the average collection period will be reduced from 2 months to 1 month.
The factor will also provide finance by lending 80% of the value of unpaid invoices, charging at an
annual rate of 8% on the cash that it lends. At the moment, entity C finances its trade receivables
with bank overdraft finance at 9% per year interest.

Required
Calculate the net effect on annual profits of Entity C if the factor took over the
administration of the trade receivables and provided finance on the terms described above.

10. PGSS produces and sells a single product. The company has issued share capital of 8,000 equity
shares of NRs. 100 each. For the year ended 16th July 2014, the company sold 60,000 units of product
at a price of NRs. 30 each.
The income statement for the year to 16th July 2014 is as follows:

NRs.’ 000
Sales 1,800
Variable costs 720
Fixed costs 360
Net profit before interest and tax 720
Interest payable 190
Net profit before tax 530
Tax@ 35% 186
Net profit after Tax 344

The company has decided to introduce a new automated production process, in order to
improve efficiency. The new process will increase annual fixed costs by NRs. 120,000
(including depreciation) but will reduce variable costs by NRs. 7 per unit. There will be no
increase in annual sales volume.

RTP-CAP II –-2014-December@ICAN Page 68 of 158


The new production process will be financed by the issue of NRs. 2,000,000 debentures @
12.5%.

Required
a) Calculate the change in earnings per share if the company introduces the new production
process.
b) Assume that the company introduces the new production process immediately on 17th July
2014. Calculate for the year to 16th July 2015:
i) The degree of operating gearing
ii) The degree of financial gearing
iii) The combined gearing effect

11. The following figures are collected from the annual report of ABC Ltd.

Net Profit NRs. 3o lakh


Outstanding 12% preference shares NRs. 100 lakhs
No. of equity shares 3 lakhs
Return on Investment 20%
Cost of Capital (Ke) 16%

What would be the approximate dividend pay out ratio so as to keep the share price at NRs. 42 by
using walter model?

12. SKL ltd. is forced to choose between two machines X & Y. the two machines are designed differently
but have identical capacity and do exactly the same job. Machine X costs NRs. 150,000 and will last
for 3 years with annual operating cost of NRs. 40,000. Machine Y is an economy model costing only
NRs. 100,000, but will last only for 2 years, and costs NRs. 60,000 p.a. to operate. Opportunity cost of
capital is 10%.
Advise which machine SKL ltd. should buy?

13. Following is data regarding six securities

A B C D E F

Return % 10 10 15 5 11 10

Risk (SD) % 5 6 13 5 6 7

i) Which of three securities will be selected?


ii) Assuming perfect correlation, analyse whether it is preferable to invest 80* in security A &
20% in security C or to invest 100% in E.

14. SS Ltd. provides you with following figures


NRs.
Profit before interest & Tax 300,000
Less: interest on Debentures @ 12% 60,000
240,000
Income Tax @ 50% 120,000
EAT 120,000
No. of Equity Shares (NRs. 10 each) 40,000
EPS 3
Ruling price in market 30
RTP-CAP II –-2014-December@ICAN Page 69 of 158
P/E ratio 10

The company has undistributed reserve of NRs. 600,000. The company needs NRs. 200,000
for expansion. This amount will earn at the same rate as funds already employed. You are
informed that a debt equity ratio higher than 35% will push the P/E ratio down to 8. The
interest rate on additional amount borrowed will be 14%.
(a) You are required to ascertain the probable price of the share
i) If the additional funds are raised as debt
ii) If the amount is raised by issuing shares
(b) Define optimum capital structure

15.
(a) Discuss the attractions of leasing as a source of both short-term and long-term finance.
(b) Discuss the relationship between the cash operating cycle and the level of investment in working
capital.
(c) Discuss the weaknesses of the dividend growth model as a way of valuing a company and its shares.

16. Distinguish between:


(a) Money market and Capital Market
(b) Factoring and Forfaiting

17. Define the following terms


(a) Venture capital financing
(b) Global depositary receipt
(c) Speculative motive of holding cash
(d) Hedging approach (to financing decision)

18. Define the following terms


a) Sensitivity Analysis
b) Risk & Uncertainty
c) Acid test ratio
d) Residual theory of dividend policy

19. A shareholder of QSX Co is concerned about the recent performance of the company and has
collected the following financial information
NRs.
Year to 31 May 2009 2008 2007
Turnover 6.8m 6.8m 6.6m
Earnings per share 58.9 64.2 61.7
Dividend per share 40 38.5 37
Closing ex dividend share price 648 835 740
Return on equity predicted by
CAPM 8% 12%

One of the items discussed at a recent board meeting of QSX Co was the dividend payment
for 2010. The finance director proposed that, in order to conserve cash within the company,
no dividend would be paid in 2010, 2011 and 2012. It was expected that improved economic
conditions at the end of this three-year period would make it possible to pay a dividend of
NRs. 70 per share in 2013. The finance director expects that an annual dividend increase of

RTP-CAP II –-2014-December@ICAN Page 70 of 158


3% per year in subsequent years could be maintained. The current cost of equity of QSX Co
is 10% per year. Assume that dividends are paid at the end of each year.

Required
Calculate the dividend yield, capital gain and total shareholder return for 2008 and 2009

20. Explain the following

a) Explain the difference between systematic and unsystematic risk in relation to portfolio theory and
the capital asset pricing model
b) Discuss whether changing the capital structure of a company can lead to a reduction in its cost of
capital and hence to an increase in the value of the company.
c) Identify the objectives of working capital management and discuss the central role of
working capital management in financial management.

SUGGESTED ANSWERS/HINTS

1. Projected cash flow statement of SPG Ltd. as on 32 Ashad 2072:


NRs.
Net Profit After Depreciation 104,500
Adjustment
Depreciation 114,000
Gain on Sale of PPE (9,500) 104,500
209,000
Adjustment for changes in Working
Capital
(Increase)/Decrease in Inventory 14,500
(Increase)/Decrease in Receivables (100,000)
Increase/(Decrease) in Payables 57,000 (28,500)
Cash Flow From Operating Activities 180,500

Cash Flow From Investing Activities


Purchase of an item of PPE (190,000)
Disposal of an item of PPE 38,000
Cash Flow From Investing Activities (152,000)

Cash Flow From Financing Activities


Increase/(Decrease) in Share Capital -
Dividend Paid (60,000)
Cash Flow From Financing Activities (60,000)
Net Cash flow During the Year (31,500)
Opening Balance 66,500
Closing Balance of Cash & Cash
Equivalent 35,000

2.

RTP-CAP II –-2014-December@ICAN Page 71 of 158


a) Calculation of NPV
NRs.’000
Year 1 2 3 4 5

Sales income 5,670 6,808 5,788 6,928

Variable cost (3,307) (4,090) (3,514) (4,040)

Contribution 2,363 2,718 2,274 2,888

Fixed cost (776) (803) (832) (861)

Cash flow before tax 1,587 1,915 1,442 2,027

Tax at 28% (444) (536) (404) (568)

Depreciation tax benefit 350 263 197 479

Cash flow after tax 1,587 1,821 1,169 1,820 (89)

Scrap value 400

Net cash flow 1,587 1,821 1,169 2,220 (89)

Discount at 13% 0·885 0·783 0·693 0·613 0·543

Present values 1,405 1,426 810 1,361 (48)

Sum of present values 4,954

Initial investment 5,000

Net present value (46)

Although the NPV of the project is negative and so financially it is not acceptable, the Board of OAP
Co has decided that it must be undertaken as it strategically important.

Workings
Year 1 2 3 4

Selling price (NRs./unit) 450 475 500 570

Inflated selling price (NRs./unit) 472·50 523·69 578·81 692·84

Sales volume (units/year) 12,000 13,000 10,000 10,000

Sales income (NRs.’000/year) 5,670 6,808 5,788 6,928

Variable cost (NRs./unit) 260 280 295 320

RTP-CAP II –-2014-December@ICAN Page 72 of 158


Inflated variable cost (NRs./unit) 275·60 314·61 351·35 403·99

Sales volume (units/year) 12,000 13,000 10,000 10,000

Variable cost (NRs.’000/year) 3,307 4,090 3,514 4,040

NRs.
Year Tax allowable depreciation Tax benefit

1 5,000,000 x 0·25 = 1,250,000 1,250,000 x 0·28 = 350,000

2 3,750,000 x 0·25 = 937,500 937,500 x 0·28 = 262,500

3 2,812,500 x 0·25 = 703,125 703,125 x 0·28 = 196,875

4 1,709,375* 1,709,375 x 0·28 = 478,625

*5,000,000 – 1,250,000 – 937,500 – 703,125 – 400,000 = 1,709,375

b) Calculation of maximum NPV

Project A B C D E

Investment (NRs.’000) 2,500 2,200 2,600 1,900 5,000

NPV (NRs.’000) 1,000 1,550 1,350 1,500 nil

PV of future cash flows (NRs.’000) 3,500 3,750 3,950 3,400 5,000

Profitability index 1.4 1.705 1.519 1.789 1

Ranking 4 3 2 1

Project E has been ranked first as it must be undertaken. Project B cannot be undertaken if Project D
is undertaken, as the two projects are mutually exclusive.

Calculation of maximum NPV


NRs.’000
Investment NPV

Project E 5,000 nil

Project D 1,900 1,500

Project C 2,600 1,350

Project A 500 200

10,000 3,050

As Project A is divisible and only NRs. 500,000 (20%) of its NRs. 2,500,000 initial cost is available
after cumulative investment in Projects E, D and C, the NPV from the project is NRs. 200,000 (20%
of NRs. 1,000,000).
RTP-CAP II –-2014-December@ICAN Page 73 of 158
3.
a)
80 days
Inventory days
{365 x (5,700/26,000)}

60 days
Trade receivables days
{365 x (6,575/40,000)}

30 days
Trade payables days
{365 x (2,137/26,000)}

110 days
Working capital cycle of CSZ Co
(80 + 60 – 30)

The working cycle of CSZ Co is positive and the company pays its trade suppliers 110 days (on
average) before it receives cash from its customers. This represents a financing need as far as CSZ Co
is concerned, which could be funded from a short-term or long-term source.
If the working capital cycle had been negative, CSZ Co would have been receiving cash from its
customers before it needed to pay its trade suppliers. A company, which does not give credit to its
customers, such as a supermarket chain, can have a negative working capital cycle.

b) At the end of March 2015:


Cost of sales = 40,000,000 x 0.6 = NRs. 24,000,000
Inventory using target inventory days = 24,000,000 x 60/365 = NRs. 3,945,206
Trade receivables using target trade receivables days = 40,000,000 x 75/365 = NRs. 8,219,178
Current assets = 3,945,206 + 8,219,178 = NRs. 12,164,384

If the target current ratio is 1.4 times, current liabilities = 12,164,384/1·4 = NRs. 8,688,846
The target quick ratio (acid test ratio) = 8,219,178/8,688,846 = 0.95 times
Net current assets at the end of March 2015 = 12,164,384 – 8,688,846 = NRs. 3,475,538
Target sales/net working capital ratio = 40,000,000/3,475,538 = 11.5 times

4.
a) Cost of equity
The current cost of equity can be calculated using the capital asset pricing model.
Equity or market risk premium = 11 – 4 = 7%
Cost of equity = 4 + (0.9 x 7) = 4 + 6.3 = 10.3%

After-tax cost of debt


After-tax interest payment = 1000 x 0.07 x (1 – 0.2) = NRs. 56 per bond

Year Cash flow (NRs.) 5% discount PV 4% discount PV (NRs.)

0 market value (1,071.4) 1 (1,071.4) 1 (1,071.4)

RTP-CAP II –-2014-December@ICAN Page 74 of 158


1-7 interest 56 5.786 324.02 6.002 336.11

7 redemption 1000 0.711 711 0.760 760

(36.38) 24.71

After-tax cost of debt = IRR = 4 + ((5 – 4) x 24.71)/ (24.71 + 36.38) = 4 + 0·4 = 4·4%
Market value of equity = 10,000,000 x 750 =NRs. 750 million
Market value of Fence Co debt = 140 million x 1071.4/1000 = NRs.150 million
Total market value of company = 750 + 150 = NRs. 900 million
WACC = {(10·3 x 750) + (4·4 x 150)}/900 = 9·3%

5.
a) In order to evaluate whether Spot Co should use leasing or borrowing, the present value of the cost
of leasing is compared with the present value of the cost of borrowing.

Leasing
The lease payments should be discounted using the cost of borrowing of Spot Co. Since
taxation must be ignored, the before-tax cost of borrowing must be used. The 7% interest rate of
the bank loan can be used here.
The five lease payments will begin at year 0 and the last lease payment will be at the start of year 5,
i.e. at the end of year 4. The appropriate annuity factor to use will therefore be 4.387 (1 + 3.387).
Present value of cost of leasing = 155,000 x 4.387= NRs. 679,985

Borrowing
The purchase cost and the present value of maintenance payments will be offset by the present
value of the future scrap value. The appropriate discount rate is again the before-tax cost of
borrowing of 7%.

7% Discount Present
Year Cash flow (NRs.)
factor value (NRs.)

0 Purchase (750,000) 1 (750,000)

1-5 Maintenance (20,000) 4.1 (82,000)

5 Scrap value 75,000 0.713 53,475

Present value of cost of borrowing = 750,000 + 82,000 – 53,475 = NRs. 778,525


The cheaper source of financing is leasing, since the present value of the cost of leasing is NRs.
98,540 less than the present value of the cost of borrowing.

6.
Current average collection period = 30 + 10 = 40 days
Current accounts receivable = 6m x 40/ 365 = NRs. 657,534
Average collection period under new policy = (0.3 x 15) + (0.7 x 60) = 46.5 days
New level of credit sales = NRs. 6.3 million
Accounts receivable after policy change = 6.3 x 46.5/ 365 = NRs. 802,603
Increase in financing cost = (802,603 – 657,534) x 0.07 = NRs. 10,155

RTP-CAP II –-2014-December@ICAN Page 75 of 158


Amount (NRs.)

Increase in financing cost 10,155

31,500
Incremental costs
(6.3m x 0.005)

28,350
Cost of discount
(6.3m x 0.015 x 0.3)

Increase in costs 70,005

180,000
Contribution from increased sales
(6m x 0.05 x 0.6)

Net benefit of policy change 109,995

The proposed policy change will increase the profitability of XYZ Co.

7.
Overtrading arises when a company does not have enough long-term finance to support its
level of trading activity. There are a number of signs of overtrading, which are referred to in
the following discussion.
Rapid increase in revenue or turnover compared to long-term finance
Revenue has increased by 40%, from NRs. 10,375,000 to NRs. 14,525,000, while long-term
finance has increased by only 4.7% (NRs. 16,268,000/NRs. 15,541,000).
Increase in trade receivables days
A rapid increase in revenue may be due to offering more generous credit terms to
customers, in which case the trade receivables ratio would be expected to increase. Trade
receivables days have in fact increased from 61 days to 80 days, an increase of 31%. In 2010
trade receivables days were close to the average value for similar companies of 60 days, but
they are now 33% more than this. While revenue has increased by 40%, trade receivables
have increased by 85% (NRs. 3,200,000/NRs. 1,734,000). It appears that VBBS Co has offered
more generous credit terms to its customers, although another explanation could be that
the company’s customers are struggling to settle their accounts on time due a downturn in
economic activity, for example a recession, leading to an increase in overdue payments and
outstanding invoices.
Decrease in profitability
A rapid increase in revenue may also be due to offering lower prices on products sold,
affecting gross profit margin or net profit margin. The net profit margin of VBBS Co has
decreased from 36% in 2010 to 28% in 2011. While revenue increased by 40%, profit before
interest and tax increased by only 8.9% (NRs. 4,067,000/NRs. 3,735,000). While this
decrease in profitability supports the possibility that VBBS Co has decreased selling prices in
order to increase sales volume, such a decrease in profitability may also be caused by an
increase in cost of sales or other operating costs.

Rapid increase in current assets

RTP-CAP II –-2014-December@ICAN Page 76 of 158


The increase in trade receivables has already been discussed. Inventory increased by 97%
(NRs. 2,149,000/NRs. 1,092,000) compared to the revenue increase of 40%, indicating
perhaps that further increases in sales volume are being planned by VBBS Co. Inventory
days also increased from 60 days in 2010 to 75 days in 2011, well above the average value for
similar companies of 55 days. There has therefore been a rapid increase in current assets of
89% (NRs. 5,349,000/NRs. 2,826,000), compared to the increase in long-term finance of
only 4.7%.
An increased dependence on short-term finance
VBBS Co has certainly increased its dependence on short-term finance and this can be
shown in several ways. The sales revenue/net working capital ratio has increased from 11
times in 2010 to 15 times in 2011, compared to the average value for similar companies of 10
times. There has been a 500% increase in the company’s overdraft (NRs. 1,500,000/NRs.
250,000) and a 75% increase in trade payables (NRs. 2,865,000/NRs. 1,637,000).
Furthermore, trade payables days rose from 90 days in 2010 to 100 days in 2011, higher than
the average value for similar companies of 85 days. Short-term debt as a proportion of total
debt increased from 6% in 2010 (NRs. 250,000/NRs. 4,250,000) to 27% in 2011 (NRs.
1,500,000/NRs. 5,500,000). This analysis supports the view that VBBS Co is more dependent
on short-term finance in 2011 than in 2010.
A decrease in liquidity
A key problem arising from overtrading is a decrease in liquidity and a shortage of cash. The
current ratio of VBBS Co has fallen from 1.5 times in 2010 to 1.2 times in 2011, compared to an
average value for similar companies of 1.7 times. The quick ratio or acid test ratio, which is a
more sensitive measure of liquidity, has fallen from 0.9 times in 2010 to 0.7 times in 2011,
compared to an average value for similar companies of 1.1 times. There are therefore clear
indications that liquidity has fallen over the period and that VBBS Co has a weaker liquidity
position than similar companies on an average basis. However, the current assets of the
company do still exceed its current liabilities, so it does not yet have a liquid deficit.
Conclusion
Overall, it can be concluded that there are several indications that VBBS Co is moving, or
has moved, into an overtrading (under capitalization) position.

Workings
Increase in revenue = 100 x (14,525 – 10,375)/10,375 = 40%
Increase in long-term finance = 100 x (16,268 – 15,541)/15,541 = 4·7%
2011 2010
Net profit margin 100 x 4,067/14,525 = 28% 100 x 3,735/10,375 = 36%
Current ratio 5,349/4,365 = 1.2 times 2,826/1,887 = 1.5 times
Quick ratio 3,200/4,365 = 0.7 times 1,734/1,887 = 0.9 times
Inventory days 365 x 2,149/10,458 = 75 days 365 x 1,092/6,640 = 60 days
Receivables days 365 x 3,200/14,525 = 80 days 365 x 1,734/10,375 = 61 days
Payables days 365 x 2,865/10,458 = 100 days 365 x 1,637/6,640 = 90 days
Net working capital 5,349 – 4,365 = NRs.984,000 2,826 – 1,887 = NRs.939,000
Sales/net working
capital 14,525/984 = 15 times 10,375/939 = 11 times

8.
a) Price/earnings ratio valuation

RTP-CAP II –-2014-December@ICAN Page 77 of 158


The value of the company using this valuation method is found by multiplying future
earnings by a price/earnings ratio. Using the earnings of PS Ltd. in Year 1 and the
price/earnings ratio of similar listed companies gives a value of 3,000,000 x 5= NRs.
15,000,000.
Using the current average price/earnings ratio of similar listed companies as the basis for
the valuation rests on two questionable assumptions. First, in terms of similarity, the
valuation assumes similar business operations, similar capital structures, similar earnings
growth prospects, and so on. In reality, no two companies are identical. Second, in terms of
using an average price/earnings ratio, this may derive from companies that are large and
small, successful and failing, low-geared and high-geared, and domestic or international in
terms of markets served. The calculated company value therefore has a large degree of
uncertainty attached to it.
b) Value of company using the dividend valuation model

The current cost of equity using the capital asset pricing model = 4 + (1.6 x 5) = 12%
Since a dividend will not be paid in Year 1, the dividend growth model cannot be applied
straight away. However, dividends after Year 3 are expected to grow at a constant annual
rate of 3% per year and so the dividend growth model can be applied to these dividends.
The present value of these dividends is a Year 3 present value, which will need discounting
back to year 0. The market value of the company can then be found by adding this to the
present value of the forecast dividends in Years2 and 3.
PV of year 2 dividend = 500,000/1.122 = NRs. 398,597
PV of year 3 dividend = 1,000,000/1.123 = NRs. 711,780
Year 3 PV of dividends after year 3 = (1,000,000 x 1.03)/ (0.12 – 0.03) = NRs. 11,444,444
Year 0 PV of these dividends = 11,444,444/1.123 = NRs. 8,145,929
Market value from dividend valuation model = 398,597 + 711,780 + 8,145,929 = NRs.
9,256,306 or approximately NRs. 9.3 million

c) Current weighted average after-tax cost of capital

Current cost of equity using the capital asset pricing model = 12%
After-tax cost of debt = 5 x (1 – 0·2) = 5 x 0·8 = 4%
Current after-tax WACC = (12 x 0·75) + (4 x 0·25) = 10% per year
Weighted average after-tax cost of capital after new debt issue
Revised cost of equity = Ke = 4 + (2·0 x 5) = 14%
Revised after-tax cost of debt = 6 x (1 – 0·2) = 6 x 0·8 = 4·8%
Revised after-tax WACC = (14 x 0·6) + (4·8 x 0·4) = 10·32% per year

Comment
The after-tax WACC has increased slightly from 10% to 10·32%. This change is a result of the
increases in the cost of equity and the after-tax cost of debt, coupled with the change in
gearing. Although the cost of equity has increased, the effect of the increase has been
reduced because the proportion of equity finance has fallen from 75% to 60% of the long-
term capital employed. Although the after-tax cost of debt has increased, the cost of debt is
less than the cost of equity and the proportion of cheaper debt finance has increased from
25% to 40% of the long-term capital employed.

9.
Annual sales= NRs. 100,000 x 12 months= NRs. 1,200,000.
Average trade receivables without the factor = NRs. 1,200,000 x 2 months/12 months
RTP-CAP II –-2014-December@ICAN Page 78 of 158
= NRs. 200,000
Average trade receivables with the factor = NRs. 1,200,000 x 1 months/12 months
= NRs. 100,000

Annual Cost Amount (NRs.)


Without Factor
Administration (12 x NRs. 2000) 24,000
Bad debts (0.75 x NRs. 1,200,000) 9,000
Interest Cost of Finance (9% x NRs. 18,000 51,000
200,000)
With Factor
Fees (4% x NRs. 1,200,000) 48,000
Interest cost of finance
Factor finance (8% x 80% x NRs. 100,000) 6,400
Overdraft finance (9% x 20% x NRs. 1,800 56,200
100,000)
Net extra cost of the factor per year 5,200

10.
a) Existing earnings per share = = NRs. 43

Earnings per share with new production process:


NRs.’ 000
Sales 1,800
Variable costs (60,000 x NRs. 5) 300
Fixed costs (360+120) 480
Net profit before interest and tax 1,020
Interest payable {190+ (12.5% x NRs. 2 440
Million)
Net profit before tax 580
Tax@ 35% 203
Net profit after Tax 377
EPS 47.13

There is increase in EPS by NRs. 4.13

b)
i) The degree of operating gearing = 1.47 times

ii) The degree of operating gearing

= 1.76 times

iii) The combined gearing effect = 1.47 x 1.76 = 2.59 times

RTP-CAP II –-2014-December@ICAN Page 79 of 158


11.
We have,
Net profit 30 laks

Less: Preference Dividend 12 laks

Earning for Equity 18 Laks

No. of equity 3 laks

EPS Rs 6

Amount of Dividend = 0
As per Walters Model,

42
6.72 = 7.50 - 0 .25 D
0.25 D = 0.78
0.25D = 0.78

Therefore dividend per share (D) = = 3.12

Dividend payout ratio = = = 52%


12.
We have,

Machine A Machine
B
Life 3 years 2 years
Initial outflows (purchase 150000 100000
cost)
Annual Running Cost 40000 p.a 60000 p.a

Pv of outflows for Machine A


= 150,000
= 150,000
= 249,440

Pv of Outflows for Machine B


= 100,000
= 204,100

RTP-CAP II –-2014-December@ICAN Page 80 of 158


Machine A Machine B
Life 3 Years 2 Years
Pv of Outflows 249,440 204,100
Sum of PV 2.486 1.735
EAC (equal annualized year end NRs 100,338 NRs 117,637
cost) (249,440/2.486) (204,100/1.735)

Advice: SKL Ltd. shall go for machine A since it has lower equal annualized cost.

13.
Arranging the securities in the increasing order of risk;

Risk Return Security Invest


(%) (%)
5 10 A Yes
5 5 D No
6 10 B No
6 11 E Yes
7 10 F No
13 15 C Yes

A, E & C shall be selected

When Investment proportion in A & C is 80% & 20% respectively


Overall Return = ReturnA X WeightA +Returnc X WeightC
= 10%*0.80+15%*0.20
= 11%
Overall Variance = (SDAX WA)2 + (SDC XWC)2+ 2 X (SDAX WA) X (SDC X WC) x rAC
(Overall SD)2 = (5 X0.80) + (13 X 0.20) + 2 (5 X0.80) (13 X 0.20) (-1)
2
(Overall SD) = 1.96

Therefore, Overall SD = 1.4


Summary

Alternative 1 Alternative 2
(80% A & 20% C) (100% in E)
Standard Deviation (SD0 1.4% 6%
Overall Return 11% 11%

Conclusion
As we can see that though the return for both alternatives are same, alternative 1 has
priority over alternative 2 because it has a lower standard deviation (Risk).

14.
Existing Capital Employed
Equity capital =400,000
Reserves =600,000
Total Equity =1,000,000
Debts =500,000
Capital Employed =1,500,000
RTP-CAP II –-2014-December@ICAN Page 81 of 158
EBIT = 300,000

Now, Rate of Return on Investment (On Closing capital) = (EBIT/Closing Capital


Employed)*100
= (300,000/1,500,000)*100
=20%
As specified in the question, this rate of return i.e. 20% will be available on additional
investments

Proposed position
Existing capital Employed = 1,500,000
Additional Investment =200,000
Total Capital after addition =1,700,000

Expected EBIT = 1,700,000 X 20%


= 340,000
The additional investment can be raised from either equity or debt issue

Option 1: If fund is raised through debt issue


Total Capital =1,700,000
Equity =1,000,000
Debt =500,000+200,000 = 700,000

Debt- Equity Ratio = Debt/(Debt+Equity)


=700,000/ (1,700,000)
=41.18%
Since the D/E ratio is more than 35%, applicable P/E ratio will be 8 times

Option 2: If Fund is raised through equity issue


New shares may be issued at the current market price by the company or at a slightly lower
price. Let us assume that the company issues its shares at Rs. 28 then,
Number of new shares to be issued =Fund Required/Issue price per share
=200,000/28
=7,143 (appx.)

Total shares outstanding under this option= 40,000+7,143 = 47,143


Calculation of probable price per share

Option 1 (Debt Option 2 (Equity


Particulars
Issue) Issue)
EBIT 340,000 340,000
Less: Interest on Existing bonds 60,000 60,000
Interest on additional debt 28,000 0
Earnings Before Tax 252,000 280,000
Less: Tax @ 50% 126,000 140,000
Earnings After Tax (EAT) 126,000 140,000
Outstanding No. of Shares (N) 40,000 47,143
Earnings per share (EPS) = 3.15 2.97
RTP-CAP II –-2014-December@ICAN Page 82 of 158
EAT/N
P/E Ratio (Times) 8 10
Market price per share (EPS X 25.20 29.70
P/E ratio)

Conclusion:
Since market price per share is higher in case of equity issue, the company shall
raise required funds from equity.

Working Notes
Amount of interest = Debt X Interest Rate
Therefore, Debt amount = Interest amount/interest rate
= 60,000/12
= 500,000

Existing Equity Capital =No. of shares X paid up value per share


=40,000 X 10
=400,000

15.
a) Operating leasing can act as a source of short-term finance, while finance leasing can act as a source
of long-term finance.
Operating leasing offers a solution to the obsolescence problem, whereby rapidly aging assets
can decrease competitive advantage. Where keeping up-to-date with the latest technology is
essential for business operations, operating leasing provides equipment on short-term contracts
which can usually be cancelled without penalty to the lessee. Operating leasing can also provide
access to skilled maintenance, which might otherwise need to be bought in by the lessee, although
there will be a charge for this service.
Both operating leasing and finance leasing provide access to non-current assets in cases where
borrowing may be difficult or even not possible for a company. For example, the company may lack
assets to offer as security, or it may be seen as too risky to lend to. Since ownership of the leased
asset remains with the lessor, it can be retrieved if lease rental payments are not forthcoming.

b) The cash operating cycle is the average length of time between paying trade payables and receiving
cash from trade receivables. It is the sum of the average inventory holding period, the average
production period and the average trade receivables credit period, less the average trade payables
credit period. Using working capital ratios, the cash operating cycle
is the sum of the inventory turnover period and the accounts receivable days, less the accounts
payable days.
The relationship between the cash operating cycle and the level of investment in working capital is
that an increase in the length of the cash operating cycle will increase the level of investment in
working capital. The length of the cash operating cycle depends on working capital policy in relation
to the level of investment in working capital, and on the nature of the business operations of a
company.
Working capital policy
Companies with the same business operations may have different levels of investment in working
capital as a result of adopting different working capital policies. An aggressive policy uses lower
levels of inventory and trade receivables than a conservative policy, and so will lead to a shorter cash
operating cycle. A conservative policy on the level of investment in working capital, in contrast, with
higher levels of inventory and trade receivables, will lead to a longer cash operating cycle. The
higher cost of the longer cash operating cycle will lead to a decrease in profitability while also
decreasing risk, for example the risk of running out of inventory.
RTP-CAP II –-2014-December@ICAN Page 83 of 158
Nature of business operations
Companies with different business operations will have different cash operating cycles. There may
be little need for inventory, for example, in a company supplying business services, while a company
selling consumer goods may have very high levels of inventory. Some companies may operate
primarily with cash sales, especially if they sell direct to the consumer, while other companies may
have substantial levels of trade receivables as a result of offering trade credit to other companies.

c) The dividend growth model (DGM) is used widely in valuing ordinary shares and hence in valuing
companies, but there are a number of weaknesses associated with its use.
The future dividend growth rate
The DGM is based on the assumption that the future dividend growth rate is constant, but
experience shows that a constant dividend growth rate is, in reality, very rare. This may be seen as
less of a problem if the future dividend growth rate is regarded as an average growth rate.
Estimating the future dividend growth rate is very difficult in practice and the DGM is very sensitive
to small changes in this key variable. It is common practice to estimate the future dividend growth
rate by calculating the historical dividend growth, but the assumption that the future will reflect the
past is an easy one to challenge.
The cost of equity
The DGM assumes that the future cost of equity is constant, when in reality it changes quite
frequently. The cost of equity can be calculated using the capital asset pricing model, but this model
usually employs historical information, which may not reflect accurately expectations about the
future.
Zero dividends
It is sometimes claimed that the DGM cannot be used when no dividends are paid, but this depends
on whether dividends are expected in the future. If dividends are forecast to be paid from a future
date, the dividend growth model can be applied at that point to calculate a share price, which can
then be discounted to give the current ex dividend share price. Only in the case where no dividends
are paid and no dividends are expected to be paid will the DGM have no application.

16.
a) Money Market & Capital Market
The capital Market deals in financial assets. Financial Assets comprises of shares, debentures, mutual
fund etc. The capital market is also known as stock market.
Stock market and money market are two basic components of Financial system. Capital market-deals
with long and medium term instruments of financing while money market deals with short term
instruments. Some of the points of distinction between capital market and money market are as
follows:

SN Money Market Capital Market


1. There is no classification between There is a classification between primary
primary market and secondary market. market and secondary market.
2. It deals for funds of short-term It deals with funds of long-term
requirement. requirement.
3. Money market instruments include Capital Market instruments are shares and
interbank call money, notice money up debt instruments.
to three months, commercial paper, 91
days treasury bills.
4. Money market participants are banks, Capital Market participants include retail
financial institution, Central Bank and investors, institutional investors like
Government. Mutual Funds, Financial Institution,
corporate and banks.

b) Factoring & Forfaiting


RTP-CAP II –-2014-December@ICAN Page 84 of 158
Basis of Difference Factoring Forfaiting
Extent of Finance Usually 80%of the value of he 100% Financing
invoice is considered for advance
Creditworthiness Factor does the credit rating of the The forfeiting bank relies on
counterparty in case of a non- the credibility of the availing
recourse factoring transaction. bank.
Service Provided Day to day administration of sales No services are provided
and other allied services are
provided
Maturity Advances are short-term in nature Advances are generally
medium- term
17.
(e) Venture capital financing
Venture capital financing refers to financing of high-risk ventures promoted by new qualified
entrepreneurs who require funds to shape their ideas. Here, a financier (called venture capitalist)
invests in equity or debt of an entrepreneur (promoter/venture capital undertaking) who has a
potentially successful business idea, but does not have the desired record of accomplishment or
financial backing.

(f) Global depository receipt


Global deposit0ry receipt refers to a negotiable certificate, denominated in US Dollars and which
represents a non-US company’s publicly traded local currency (e.g. Nepalese Rupee) equity shares.
Depository receipts are created when the local currency shares are delivered to the depository’s local
custodian bank, against which the depository bank issues depository receipt in US Dollars.
These depository receipts are freely traded in the overseas markets like any other dollar
denominated security through either a foreign stock exchange or through over the counter market
or among restricted groups like qualified institutional buyers.

(g) Financial Distress


Financial distress is defined as a condition where obligations are not met or are met with difficulty.
A major disadvantage for a firm taking on higher levels of debt is that it increases the risk of
financial distress, and ultimately liquidation. This may have detrimental effect on both the equity
and debt holders.

(h) Speculative motive of holding cash


It refers to the desire of a firm to take advantage of opportunities which present themselves at
unexpected moments and which are typically outside the normal course of business. While the
precautionary motive is defensive in nature where the firms make provisions to tide over
unexpected contingencies, the speculative motive represents a positive and aggressive approach.
Firms aim to exploit profitable opportunities and keep cash in reserve to do so.

18.
e) Sensitivity Analysis
Sensitivity analysis is a useful but simple technique for assessing investment risk in a capital
expenditure project when there is uncertainty about the estimates of future cash flows. It is
recognized that estimates of cash flows could be inaccurate, or that events might occur that will
make the estimates wrong.
The purpose of sensitivity analysis is to assess how the NPV of the project might be affected if cash
flow estimates are worse than expected.
There are two main method of carrying out sensitivity analysis on a capital expenditure project.
 Sensitivity analysis can be used to calculate the effect on the NPV of a given percentage
reduction in benefits or a given percentage increase in costs.

RTP-CAP II –-2014-December@ICAN Page 85 of 158


 Alternatively, sensitivity analysis can be used to calculate the percentage amount by which
benefits must fall below estimate or cost rise above estimate before the project NPV becomes
negative.

f) Risk & Uncertainty


In common parlance, the terms ‘Risk & Uncertainty’ have synonymous meaning. However, they
differ from each other:
Risk may be defined as “the chance of future loss that can be foreseen”. In other words, in case of
risk an estimate can be made about the degree of happening of the loss. This is usually done by
assigning probabilities to the risk on the basis of past data and the probable trends.
Whereas uncertainty may be defined as “the unforeseen chance for future loss or damages.” In case
of uncertainty since the firm cannot anticipate the future loss and hence it cannot directly deal with
it in its planning process, as is possible in the case of risk.

g) Acid test ratio


The acid test ratio is the ratio between quick current assets and current liabilities and is calculated
by dividing the quick assets by the current liabilities. The acid test ratio is also referred to as a
measure of a firm’s ability to convert its current assets quickly into cash in order to meet its current
liabilities.
The acid test ratio is rigorous measure of a firm’s ability to service short term liabilities. the
usefulness of the ratio lies in the fact it is widely accepted as the best available test of the liquidity
position of a firm.

h) Residual theory of dividend policy


The residual theory of dividend policy is that the optimal amount of dividends should be decided as
follows:
 If a company has capital investment opportunities that will have a positive NPV, it should invest
in them because they will add to the value of the company and its shares.
 The capital to invest in these projects should be obtained internally (from earnings) if possible.
 The amount of dividends paid by a company should be the residual amount of earnings
remaining after all these available capital projects have been funded by retained earnings.
 In this way, the company will maximize its total value and the market price of its shares.
A practical problem with residual theory is that annual dividends will fluctuate, depending
on the availability of worthwhile capital projects. Shareholders will therefore be unable to
predict what their dividends will be.

19.
Dividend yield is calculated as the dividend divided by the share price at the start of the
year.
2008: dividend yield = 38.5/740 x 100= 5.2%
2009: dividend yield = 40/835 x 100 = 4.8%
The capital gain is the difference between the opening and closing share prices, and may be
expressed as a monetary amount or as a percentage of the opening share price.
2008: capital gain = 835 – 740 = NRs. 95 or 12·8% (100 x 95/740)
2009: capital gain/ (loss) = 648 – 835 = (NRs. 187) or (22·4%) (100 x –187/835)
The total shareholder return is the sum of the percentage capital gain and the dividend
yield, or the sum of the dividend and the monetary capital gain, expressed as a percentage
of the opening share price.
2008: total shareholder return = 100 x (95 + 38·5)/740 = 18·0% (5·2% + 12·8%)

RTP-CAP II –-2014-December@ICAN Page 86 of 158


2009: total shareholder return = 100 x (–187 + 40)/835 = –17·6% (4·8% – 22·4%)
The return on equity predicted by the CAPM
The actual return for a shareholder of QSX Co, calculated as total shareholder return, is very
different from the return on equity predicted by the CAPM. In 2008, the company provided
a better return than predicted and in 2009, the company gave a negative return while the
CAPM predicted a positive return.

Year 2009 2008


Total shareholder return (17·6% 18·0%
)
Return on equity predicted by 8% 12%
CAPM

20.
a) Portfolio theory suggests that the total risk of a portfolio of investments can be reduced by
diversifying the investments held in the portfolio, e.g. by investing capital in a number of
different shares rather than buying shares in only one or two companies.
Even when a portfolio has been well-diversified over a number of different investments, there is a
limit to the risk-reduction effect, so that there is a level of risk which cannot be diversified away.
This undiversifiable risk is the risk of the financial system as a whole, and so is referred to as
systematic risk or market risk. Diversifiable risk, which is the element of total risk which can be
reduced or minimised by portfolio diversification, is referred to as unsystematic risk or specific risk,
since it relates to individual or specific companies rather than to the financial system as a whole.
Portfolio theory is concerned with total risk, which is the sum of systematic risk and unsystematic
risk. The capital asset pricing model assumes that investors hold diversified portfolios, and so is
concerned with systematic risk alone.

b) The value of a company can be expressed as the present value of its future cash flows, discounted at
its weighted average cost of capital (WACC). The value of a company can therefore theoretically be
maximised by minimising its WACC. If the WACC depends on the capital structure of a company,
i.e. on the balance between debt and equity, then the minimum WACC will arise when the capital
structure is optimal.
The idea of an optimal capital structure has been debated for many years. The traditional view of
capital structure suggests that the WACC decreases as debt is introduced at low levels of gearing,
before reaching a minimum and then increasing as the cost of equity responds to increasing
financial risk.
Miller and Modigliani originally argued that the WACC is independent of a company’s capital
structure, depending only on its business risk rather than on its financial risk. This suggestion that it
is not possible to minimise the WACC, and hence that it is not possible to maximise the value of a
company by selecting a particular capital structure, depends on the assumption of a perfect capital
market with no corporate taxation.
However, real world capital markets are not perfect and companies pay taxes on profit. Since
interest is a tax-allowable deduction in calculating taxable profit, debt is a tax-efficient source of
finance and replacing equity with debt will decrease the WACC of a company. In the real world,
therefore, increasing gearing will decrease the WACC of a company and hence increase its value.
At high levels of gearing, the WACC of a company will increase due, for example, to increasing
bankruptcy risk. Therefore, it can be argued that use of debt in a company’s capital structure can
reduce its WACC and increase its value, provided that gearing is kept to an acceptable level.

c) The objectives of working capital management are usually taken to be profitability and liquidity.
Profitability is allied to the financial objective of maximizing shareholder wealth, while liquidity is
needed in order to settle liabilities as they fall due. A company must have sufficient cash to meet its
RTP-CAP II –-2014-December@ICAN Page 87 of 158
liabilities, since otherwise it may fail. However, these two objectives are in conflict, since liquid
resources have no return or low levels of return and hence decrease profitability. A conservative
approach to working capital management will decrease the risk of running out of cash,
favoring liquidity over profitability and decreasing risk. Conversely, an aggressive approach to
working capital management will emphasize profitability over liquidity, increasing the risk of
running out of cash while increasing profitability.
Working capital management is central to financial management for several reasons. First,
cash is the life-blood of a company’s business activities and without enough cash to meet short-
term liabilities, a company would fail. Second, current assets can account for more than half of a
company’s assets, and so must be carefully managed. Poor management of current assets can lead to
loss of profitability and decreased returns to shareholders. Third, for SMEs current liabilities are a
major source of finance and must be carefully managed in order to ensure continuing availability of
such finance.

RTP-CAP II –-2014-December@ICAN Page 88 of 158


PAPER 5: COST AND MANAGEMENT ACCOUNTING

Cost Concepts and Costing Methods:


Question No. 1

a) Define the terms „Cost center‟ and „Cost unit‟.


b) Indicate whether the following statements are True or False:
(i) Fixed cost per unit remains fixed.
(ii) All costs are controllable.
(iii) Conversion cost is equal to direct wages plus factory overhead.
(iv) Variable cost per unit varies with increase or decrease in the volume of output.
(v) Depreciation is an output of pocket cost.
(vi) An item of cost that is direct for one business may be indirect for another.
c) How does a production account differ from a cost sheet?
d) Define explicit costs. How is it different from implicit costs?

Material Control:
Question No. 2
a. List five types of inefficiencies in the use of materials that may be discovered as a result of investigating
material quantity variances.

b. Explain briefly how storage losses and insurance costs on stock of raw materials are to be treated in costs.

c. A manufacturer of Butwal purchased three Chemicals A, B, C from India. The invoice gave the following
information:
Chemical A: 3,000 kg @ Rs. 4.20 per kg 12,600

Chemical B: 5,000 kg @ Rs. 3.80 per kg 19,000

Chemical C: 2,000 kg @ Rs. 4.75 per kg 9,500

Sales Tax 2,055

Freight Charges 1,000

Total 44,155

A Shortage of 200 kg in Chemical A, 280 kg in Chemical B and 100 kg in Chemical C was noticed due to
breakage. At Butwal, the manufacturer paid duties @ Re. 0.10 per kg. He also paid Cartage Rs. 22 for
Chemical A, Rs. 63.12 for Chemical B and Rs. 31.80 for Chemical C. Calculate the stock rate that you would
suggest for pricing issue of chemical assuming a provision of 5 % towards further deterioration.

d. Mayaz Ltd. has received an offer quantity discount on his order of materials as under:
Price per ton(Rs.) Order quantity

1,200 Less than 500

1,180 500 and Less than 1,000

RTP-CAP II –-2014-December@ICAN Page 89 of 158


1,160 1,000 and Less than 2,000

1,140 2,000 and Less than 3,000

1,120 3,000 and above

The annual requirement for the material is 5,000 tons. The ordering cost per order is Rs. 1,200 and the stock
holding cost is estimated at 20%. You are required to compute the most economical order quantity.

What would be your answer to the above question if there are no discounts offered and the price per ton is Rs.
1,500?

Labor Control:
Question no. 3
a. What is overtime premium? Explain the treatment of overtime premium in cost accounting.
b. Calculate the earnings of workers A and B from the following particulars for a month and allocate the labor
cost to the jobs X, Y, and Z.
A B

Basic Wages Rs. 100 Rs. 160


Dearness Allowances 50% 50%
Contribution to Provident Fund
(On basic wages) 8% 8%
Contribution to Employees‟ Insurance 2% 2%
(On basic wages)
Overtime 10 Hours
The normal working hours for the month are 200. Overtime is paid at double the total of normal
wages and dearness allowance. Employers‟ contribution to Employees‟ Insurance and Provident
Fund are at equal rates of employees‟ contribution. The two workers were employed on jobs X, Y, Z
in the following proportions:
Jobs
X Y Z
Worker A 40% 30% 30%
Worker B 50% 20% 30%

Overtime was done on Job Y.

c. Which is better plan out of Hasley 50% bonus scheme and Rowan bonus scheme for an efficient worker? In
which situation the worker gets same bonus under both schemes?

d. From the following information, calculate labor turnover rate and labor flux rate.

No. of workers as on 01.01.2000 = 7,600


No. of workers as on 31.12.2000 = 8,400

During the year, 80 workers left while 320 workers were discharged 1,500 workers were recruited
during the year of which 300 workers were recruited because of exits and rest were recruited in
accordance with the expansion plans.

RTP-CAP II –-2014-December@ICAN Page 90 of 158


Overhead Control:
Question no. 4
a. A cost center in a factory furnishes the following working conditions:
Normal working week 40 hours

No. of machines 15

Normal weekly loss of hours on maintenance 4 hrs. per machine

Estimated annual overhead Rs. 155,520

Estimated direct wage rate Rs. 3 per hour

No. weeks worked per year 48

Actual results during a four week period:

Overhead incurred Rs. 15,000

Wages incurred Rs. 7,000

Machine hours produced 2,200

You are required to:


i. Calculate the overhead rate per machine hour.
ii. Calculate the amount of under and over absorption of both wages and overhead.

b. The level of production activity fluctuates widely in your company from month to month. Because of this, the
incidence of depreciation on unit cost varies considerably. Suggest a suitable method under the situation.

c. A Ltd. manufactures two products X and Y. The manufacturing division consists of two production
departments P1 and P2 and two services S1 and S2.

Budgeted overhead rates are used in the production departments to absorb factory overheads to the
products. The rate of Department P1 is based on direct machine hours, while the rate of Department
P2 is based on direct labor hours. In applying overheads, the pre-determined rates are multiplied by
actual hours.
For allocating the services department costs to production departments, the basis adopted is as
follows:
(i) Cost of Department S1 to Department P1 and P2 equally, and
(ii) Cost of Department S2 to Department P1 and P2 in the ratio 2:1 respectively.

The following budgeted and actual data are available:


Annual profit plan data:
Factory overhead budgeted for the year:
Rs. Rs.

Departments P1 2,550,000 S1 600,000


P1 2,175,000 S2 450,000

Budgeted output in units:


Product X – 50,000
Product Y – 30,000
Budgeted raw material cost per unit:
RTP-CAP II –-2014-December@ICAN Page 91 of 158
Product X – Rs. 120
Product Y – Rs. 150
Budgeted time required for production per unit:
Department P1: Product X: 1.5 machine hours
Product Y: 1.0 machine hours

Department P2: Product X: 2 direct labor hours


Product Y: 2.5 direct labor hours

Average wage rates budgeted in department P2 are: Product X – Rs. 72 per hour and Product Y –
Rs. 75 per hour.
All materials are used in Department P1 only.
Actual data (for the month of June, 2014)
Units actually produced: Product X: 4,000 units
Product Y: 3,000 units

Actual direct machine hours worked in department P1: On product X – 6,100 hours, Product Y –
4,150 hours.
Actual direct labor hours worked in department P2: On product X – 8,200 hours, Product Y –
7,400 hours.
Costs actually incurred:
Product A Product B

Raw materials: Rs. 489,000 Rs. 456,000


Wages: Rs. 591,900 Rs. 552,000
Overheads:
P1 Rs. 231,000 S1 Rs. 60,000
P2 Rs. 204,000 S2 Rs. 48,000

You are required to:


(i) Compute the predetermined overhead rate for each production department.
(ii) Prepare a performance report for the month of June 2014 that will reflect the budgeted costs and actual
costs.

d. Distinguish between cost allocation and cost absorption.

Cost Accounts System, Cost Control Accounts (Integrated and Non – integrated Accounting System)
Question no. 5
a. Journalize the following transactions assuming that Cost and Financial accounts are integrated:

(i) Materials issued:

Direct 425,000
Indirect 115,000

(ii) Allocation of wages(25% indirect) 650,000

(iii) Under/Over absorbed overheads:


RTP-CAP II –-2014-December@ICAN Page 92 of 158
Factory (Over) 350,000
Administration (Under) 175,000

(iv) Payment of Sundry Creditors 220,000

(v) Collection from Sundry Debtors 360,000

b. When is the reconciliation Statement of Cost and Financial accounts not required?
c. The financial statements of Kamal Manufacturers Ltd. reveal the following for the year ended 16 July 2014:
Rs. In thousands

Sales (20,000 units) 4,000


Materials 1,600
Wages 800
Factory overheads 720
Office and Administrative overheads 416
Selling and distribution overheads 288
Finished goods (1,230 units) 240
Work in progress 48
Labor 32
Overheads (Factory) 32 112
Goodwill written off 320
Interest on capital 32

In the costing records, factory overhead is charged at 100% wages, administration overhead 10% of
factory cost and selling and distribution overhead at the rate of Rs. 16 per unit sold.

Prepare a statement reconciling the profit as per cost records with the profit as per financial records
of the company.

Methods of Costing
Question no. 6
a. SB Constructions Limited has entered into a contract at an agreed price of Rs. 15,000,000 subject to an
escalation clause for material and labor as spent out on the contract and the corresponding actuals are as
follows: (June 2014)

Material: Actual
Quantity Rate per Quantity Rate per
in tons Ton (Rs.) in tons Ton (Rs.)
A 3,000 1,000 3,400 1,100
B 2,400 800 2,300 700
C 500 4,000 600 3,900
D 100 30,000 90 31,500

Labor: Standard Actual


Hours Hourly Hours Hourly

RTP-CAP II –-2014-December@ICAN Page 93 of 158


Rate (Rs) Rate (Rs)
L1 60,000 15 56,000 18
L2 40,000 30 38,000 35

You are required to:


(i) Give your analysis of admissible escalation claim and determine the final contract price payable.
(ii) Prepare the contract account, if all the expenses other than material and labor related to the contract
are Rs. 1,345,000.

b. A transport company has been given a 40 kilometer long route to run 5 buses. The cost of each bus is Rs. 6,
50,000. The buses will make 3 round trips per day carrying on an average 80% passengers of their seating
capacity. The seating capacity of each bus is 40 passengers. The buses will run on an average 25 days in a
month. The other information for the year 2013-14 are given below

Garage rent Rs. 4,000 per month


Annual repairs and maintenance Rs. 22,500 each bus
Salaries of 5 drivers Rs. 3,000 each per month
Wages of 5 conductors Rs. 1,200 each per month
Manager‟s salary Rs. 7,500 per month
Road tax, permit fee, etc. Rs. 5,000 for a quarter
Office expenses Rs. 2,000 per month
Cost of diesel per liter Rs. 33
Kilometer run per liter for each bus 6 kilometers
Annual depreciation 15% of cost
Annual insurance 3% of cost

You are required to calculate the bus fare to be charged from each passenger per kilometer, if the
company wants to earn profit of 33 1/3 percent on taking (total receipts from passengers).
c. A company within the food industry mixes powdered ingredients in two different processes to produce one
product. The output of Process I becomes the input of Process II and the output of Process II is transferred to
the packing department.

From the information given below, you are required to open accounts for Process I, Process II,
abnormal loss and packing department and to record the transactions for the week ended 24th
February, 2014.
Process I
Input:
Material A 6000 kilograms at 50 paisa per kilogram
Material B 4000 kilograms at Rupee 1 per kilogram
Mixing Labor 430 hours at Rs. 2 per hour
Normal Loss 5 % of weight input, disposed at 16 paisa per kg
Output 9,200 kg

No work in process at the beginning or end of the week.


Process II
Input:
Material C 6,600 kilograms at Rs. 1.25 per kilogram
Material D 4,200 kilograms at Re. 0.75 per kilogram
Flavoring essence Rs. 330
Mixing Labor 370 hours at Rs. 2 per hour
Normal Waste 5 % of weight input with no disposal value
Output 18,000 kilograms

RTP-CAP II –-2014-December@ICAN Page 94 of 158


No work in process at the beginning of the week but 1,000 kilograms in process at the end of the
week and estimated to be only 50% complete so far as labor and overhead were concerned.
Overhead of Rs. 3,200 incurred by the two processes to be absorbed on the basis of mixing labor
hours.

d. Distinguish between Job costing and Process costing.

e. A company‟s plant processes 150,000 Kg. of raw material in a month to produce two products P and Q. The
cost of raw material is Rs. 12 per kg. The process costs month are:

Rs.
Direct Materials 90,000
Direct Wages 120,000
Variable Overheads 100,000
Fixed Overheads 100,000

The loss in process is 5% of input and the output ratio of P and Q which emerge simultaneously is
1:2. The selling prices of the two products at the point of split off are Rs. 12 /kg for product P and Rs.
20/kg for product Q. A proposal is available to process P further by mixing it with other purchased
materials. The entire current output of the plant can be so processed further to obtain a new produce
S. The price per Kg. of S is Rs. 15 and each kg of output of S will require one kilogram of product P
as an input. The cost of processing of P into S (including other materials) is Rs. 185,000 per month.
You are required to prepare a statement showing monthly profitability based both on the existing
manufacturing operations and on further processing. Do you recommend further processing?

Cost Concepts for Decision Making


Question no. 7
a. What is the meaning of Margin of Safety (MOS)? State the relationship between Operating Leverage and
Margin of safety Ratio.
b. What do you understand by Key Factor?
c. Mega Co. has just completed its first year of operations. The unit costs on a normal coating basis are as under:
Rs.

Direct material 4 Kg @ Rs. 4 16

Direct labor 3 hrs. @ Rs. 18 54

Variable Overhead 3 hrs. @ Rs. 4 12

Fixed Overhead 3 hrs. @ Rs.6 18

100

Selling and administrative costs:

Variable Rs. 20 per unit

Fixed Rs. 760,000

During the year the company has the following activity:


Units produced = 24,000

Units sold = 21,500


RTP-CAP II –-2014-December@ICAN Page 95 of 158
Unit selling price = Rs. 168

Direct labor hours worked = 72,000

Actual fixed overhead was Rs. 48,000 less than the budgeted fixed overhead. Budgeted variable overhead
was Rs. 20,000 less than the actual variable overhead. The company used an expected actual activity level
of 72,000 direct labor hours to compute the predetermine overheads rates.

Required:

(i) Compute the unit cost and total income under:


a. Absorption Costing
b. Marginal Costing
(ii) Under or over absorption of overhead
(iii) Reconcile the difference between the total income under absorption and marginal costing.

d. Distinguish between Marginal and Absorption Costing.

Costing for Planning and Control – Budgets


Question no. 8
a. List the eight functional budgets prepared by a business.
b. The following data are available in a manufacturing company for a year period:

Fixed expenses Rs. (lakhs)

Wages and salaries 9.5

Rent, rates and taxes 6.6

Depreciation 7.4

Sundry administrative expenses 6.5

Semi-variable expenses (at 50% capacity)


Maintenance and repairs 3.5
Indirect labor 7.9
Sales department salaries etc. 3.8
Sundry administrative expenses 2.8

Variable expenses (at 50% of capacity)


Materials 21.7
Labor 20.4
Other expenses 7.9
98.0
Assume that the fixed expenses remain constant for all levels of production, semi-variable expenses
remain constant between 45% and 65% of capacity increasing by 10% between 65% and 80%
capacity and by 20% between 80% and 100% capacity.

Sales at various levels are:


Rs. (lakhs)
50% capacity 100
RTP-CAP II –-2014-December@ICAN Page 96 of 158
60% capacity 120
75% capacity 150
90% capacity 180
100% capacity 200

Prepare a flexible budget for the year and forecast the profits at 60%, 75%, 90% and 100% of
capacity.

Uniform Costing and Inter firm comparison


Question no. 9
a. What do you mean by Uniform Costing? Enumerate the points on which uniformity is essential before
introducing uniform costing system?
b. Write short notes on Uniform Cost Manual.
c. Write the limitations of inter-firm comparison.

Cost Control and Cost Reduction


Question no. 10 Write short notes on Value Analysis.

Cost Audit
Question no. 11 What are the important aspects of cost audit? How is it useful to the shareholders of a
company?

SUGGESTED ANSWERS/ HINTS

Answer 1(a)

Cost center:

The term cost center is defined as a location, person or an item of equipment or a group of these for which costs
may be ascertained and used for the purpose of cost control. Cost centers can be personal cost centers,
impersonal cost centers, operation cost centers and process cost centers.

Cost Unit:

The term cost unit is defined as a unit of quantity of product, service or time (or a combination of these) in
relation to which costs may be ascertained or expressed. It can be for a job, batch, or product group.

Answer 1(b)

(i) False
(ii) False
(iii) True
(iv) False
(v) False
(vi) True

RTP-CAP II –-2014-December@ICAN Page 97 of 158


Answer 1(c)

The following are the differences between a production account and a cost sheet:

(i) Production Account is based on double entry system whereas cost sheet is not based on double entry
system.
(ii) Production Account consists of two parts. The first part shows cost of the component and total
production cost. The second part shows the cost of sales and profit for the period. Cost sheet presents the
elements of costs in a classified manner and the cost ascertained at different states such as prime cost,
works cost, cost of production, cost of goods sold, cost of sales and total cost.
(iii) Production Account shows the cost in aggregate and thus facilitates comparison with other financial
accounts. Cost sheet shows the cost in a detailed and analytical manner which facilitates comparison of
cost for the purpose of cost control.
(iv) Production Account is not useful for preparing tenders or quotations. Estimated cost sheets can be
prepared on the basis of actual cost sheets and these are useful for preparing tender quotations.

Answer 1(d)
Explicit Costs: These costs are also known as out of pocket costs. They refer to those costs which
involves immediate payment of cash. Salaries, wages, postage and telegram, interest on loan etc. are
some examples of explicit costs because they involve immediate cash payment. These payments are
recorded in the books of account and can be easily measured.
Main differences between explicit and implicit costs are:
(i) Implicit costs do not involve any immediate cash payment. As such they are also known as imputed
costs or economic costs.
(ii) Implicit cost are not recorded in the books of account but yet, they are important for certain types of
managerial decisions such as equipment replacement and relative profitability of two alternative
courses of action.

Answer 2(a)

The five types of inefficiencies in the use of materials that may be discovered as a result of investigating materials
quantity variances are as follows:

i. Purchase of inferior quality of materials


ii. Inefficient labor force leading to excessive utilization of materials.
iii. Defective machines, tools and equipment and bad or improper maintenance leading to breakdowns resulting
in excessive usage of materials.
iv. Inaccurate technical specifications and slackness in inspection may cause more rejections, resulting in
greater requirement of materials for rectification of defects.
v. Faulty material processing.

Answer 2(b)
Storage Losses: The losses arising out of storage of material can be classified into two categories. The
treatment of losses under each category in Cost Accounts is as follows:
 Losses due to reason like evaporation, shrinkage, absorption and moisture, etc. are considered as normal losses.
Such losses are absorbed by good production units by inflating the cost of material issued for production.
 Losses due to fire, flood, storm, theft etc. are treated as abnormal losses. If these losses are heavy and are not
recoverable from insurance authorities, it is preferred to charge them to Costing Profit and Loss Account.

Insurance costs on stocks of raw materials: The amount paid as insurance costs on stocks of raw
materials is meant for covering the risk which may arise due to fire, theft, riot etc. The insurance cost is
apportioned over different materials on the basis of their value. The cost may be charged directly to cost
of material.
RTP-CAP II –-2014-December@ICAN Page 98 of 158
Answer 2(c)
Statement showing the Issue rate of Chemicals
A B C
Particulars
Rs. Rs. Rs.
Purchase Price 12,600 19,000 9,500
Add: Sales tax @ 5% of Purchase price (W.N. 630 950 475
2)
Add: Freight charges in the ratio 3:5:2 (W.N. 300 500 200
3)
Add: Duty @ 0.10 per kg (W.N. 1) 280 472 190
Add: Cartage costs 22 63.12 31.80
Total Price 13,832 20,985.12 10,396.8

Calculation of Rate of issue: A B C


Rate of issue = Total price of Rs. Rs.20,985.12 Rs.10,396.8
chemical 13,832 4,484 kg. 1,805 kg.
Qty. available for use (W.N. 2,660 kg.
1)
Rs. 5.20 Rs. 4.68 Rs. 5.76

Therefore, the rate of issue for per Kg. of material A, B and C are Rs. 5.20, Rs. 4.68 and Rs. 5.76
respectively.

Working Notes (W.N.):


1. Statement showing the quantity of Chemicals available for issue
A B C

Kg. Kg. Kg.

Quantity purchased 3,000 5,000 2,000


Less: Shortage
(Assumed to be normal) 200 280 100
Quantity received 2,800 4,720 1,900
Less: Provision for further
Deterioration 5% 140 236 95
Quantity available for use 2,660 4,484 1,805

2. Rate of Sale Tax:

(Sales Tax/Total Purchase price of Chemicals) x100


= (Rs. 2,055/Rs. 41,100) x 100 = 5%

3. Freight Charges: It has been charged on the basis of quantity purchased i.e. A: 3,000 kg, B: 5,000 kg, C:
2,000 kg in the ratio of 3:5:2.

Answer 2(d)

Statement of calculation of total costs for annual requirement of 5,000 tons at various order sizes
(a) (b = 5000/ (c) (c x 5,000 (b x Rs. (1/2 of a) x
RTP-CAP II –-2014-December@ICAN Page 99 of 158
a) tons) 1,200) (20% of c)
Order No. of Cost Cost of Ordering Carrying cost Total
Size orders per unit inventory cost (Rs.) costs
(Rs.) (Rs.) (Rs.) (Rs.)
400 12.5 1,200 6,000,000 15,000 48,000 6,063,000
500 10 1,180 5,900,000 12,000 59,000 5,971,000
1,000 5 1,160 5,800,000 6,000 116,000 5,922,000
2,000 5 1,140 5,700,000 3,000 228,000 5,931,000
3,000 1.666 5,600,000 2,000 336,000 5,938,000

From the above table, the total cost of 5,000 units including ordering and carrying costs is minimum (Rs.
59, 22,000) when the order size is 1,000 units. Hence the most economical order size is 1,000 units.

If no discount is offered and the price per ton is Rs. 1,500 then at annual requirement of 5,000 units most
economical purchase order can be calculated as follows:

Where; A= Annual inventory requirement


EOQ = √2AO/C O= Ordering cost per order
= √ (2 x 5,000 x 1,200)/ (20 % x 1,500) C= Carrying cost per unit
= 200 tons

Answer 3(a)
Overtime is the amount of wages paid for working beyond normal working hours as specified by a
mutual agreement between the workers union and the management. The payment of overtime consists of
two elements, the normal wages i.e. the usual amount and the extra payment i.e. the premium. The
amount of extra payment paid to a worker under overtime is known as overtime premium.
In cost accounting overtime premium is treated as follows:
a. If the overtime is resorted to at the desire of the customer, then the entire amount of the overtime including
overtime premium should be charged to the job directly.
b. If it is due a general pressure of work to increase the output, the premium as well as overtime wages may
be charged to general overheads.
c. If it is due to the negligence or delay of workers of a particular department, it may be charged to the
concerned department.
d. If it is due to circumstances beyond control, it may be charged to Costing Profit & Loss Account.

Answer 3(b)
Statement showing earnings of worker A and B
A B

Rs. Rs.

Basic wages 100 160

Dearness Allowance

(50% of basic wages) 50 80

Overtime wages

(Refer W.N. 1) 15 -
RTP-CAP II –-2014-December@ICAN Page 100 of 158
Gross wages earned 165 240

Less:

Provident Fund - 8% of basic wages (8) (12.8)

Employees‟ Insurance - 2% of basic wages (2) (3.2)

Net wages Paid 155 224

Statement of labor cost:

Gross Wages (Excluding overtime) 150 240

Employers‟ Contribution 10 16

Ordinary wages 160 256

Labor rate per hour 0.80 1.28

(Rs.160/200) (Rs. 256/200)

Statement showing Allocation of wages to Jobs

Total Wages X Y Z

Rs. Rs. Rs. Rs.

Worker A:

Ordinary wages (4:3:3) 160 64 48 48

Overtime 15 - 15 -

Worker B:

Ordinary wages (5:2:3) 256 128 51.20 76.8

431 192 114.2 124.8

Working notes (W.N.):


1. Calculation of overtime
Overtime = (2 x (Basic wage + D.A) /200) x 10 hours

= 2 x (Rs. 150/200) x 10 hours = Rs. 15

Basic Wages are considered as Normal wages.

RTP-CAP II –-2014-December@ICAN Page 101 of 158


Answer 3(c):
Rowan Bonus Scheme pays more bonus if the time saved is below the 50% of time allowed and if the
time saved is more than 50% of time allowed then Hasley bonus scheme pays more bonus. Generally,
time saved by a worker is not more than 50 % of time allowed. So, the Rowan bonus scheme is better for
an efficient worker. When the time saved is equal to 50% of time allowed, then both plan pays same
bonus to a worker.
Bonus under Halsey Plan:
Standard wage rate x 50/100 x Time Saved
Bonus under Rowan Plan:
Standard wage rate x Time Taken/Time allowed x Time Taken
Bonus under Halsey Plan will be equal to the Bonus under Rowan Plan when the following condition
holds good:
Standard wage rate x 50/100 x Time saved = Standard wage rate x Time taken/time allowed x time saved
Or, ½ = time taken/time allowed
Or, time taken = ½ of time allowed
Hence, when the time taken is 50% of the time allowed, the bonus under Halsey and Rowan plans is
equal.

Answer 3(d)
Labor Turnover rate:
It comprises of computation of labor turnover by using following methods:
(i) Separation Method:

= (No. of workers left + No. of workers discharged / Average no. of workers) x 100
= ((80 + 320) / (7,600 + 8,400) ÷ 2) x 100
= (400/8,000) x 100
= 5%
(ii) Replacement Methods:

= (No. of workers replaced/Average no. of workers) x 100


= 300/8000 x 100
= 3.75 %
(iii) New Recruitment:

= (No. of workers newly recruited /Average no. of workers) x 100


= 1,200/8,000 x 100
=15%
(iv) Flux Method:

= (No. of separations + No. of accessions)/Average no. of workers x 100


= ((400 + 1,500)/ (7,600 + 8,400) ÷2) x 100
=1,900/8,000 x 100
= 23.75%

Answer 4 (a)
i. Calculation of overhead rate per machine hour:

Annual Overhead – Rs. 1, 55,520

Annual working hours (Normal) = No. of machines x no. weeks p.a. x effective weekly hrs.
= 15 x 48 x 36
= 25, 920
Overhead rate per machine hour = Rs. 1, 55,520/25, 920 = Rs. 6
ii. Calculation of over/under absorption of wages and overhead:
RTP-CAP II –-2014-December@ICAN Page 102 of 158
Overhead incurred = Rs. 15,000

Overhead absorbed = Rs. 13,200 (i.e. 2,200 x Rs. 6)

Under – absorbed overhead = 15,000 – 31,200 = Rs. 1,800

Wages incurred = Rs. 7,000

Wages absorbed = Rs. 7,200 (i.e. 40 hrs. x 15 machines x Rs. 3 x 4 weeks)

Wages over – absorbed = 7,200 – 7,000 = Rs. 200

Answer 4(b)
The incidence of depreciation on unit cost, due to wide fluctuations in the production activity can be
overcome by using the method known as production unit method.
Under production unit method, depreciation is charged at a rate per unit of production, by dividing the
cost of assets by the estimated no. of units to be produced during the life of the asset. The formula for
calculating depreciation under this method is:
D= (Original Cost – Residual Value)/ (Estimated output during its life)
This method recognizes the fact that depreciation should vary according to the volume of the output. It
satisfies the costing requirement that the cost of an asset should be evenly distributed over the work done
by it. According to this method, the incidence of depreciation only arises when the asset is employed in
production and not when it remains idle. It does not recognize the time factor, but only the usage factor.
Consequently, no deprecation is provided only for the lapse of time. This method is suitable when the
units of production are identical or uniform.

Answer 4(c)
(i) Computation of predetermined overhead rate for each production department from budgeted data

Production dept. Service dept.


Particulars
P1 P2 S1 S2
Budgeted factory overheads for the year 2,550,000 2,175,000 600,000 450,000
in (Rs.)
Allocation of service department S1‟s 300,000 300,000 (600,000)
costs to production depts. P1 and P2 -
equally
Allocation of service department S2‟s 300,000 150,000 (450,000)
costs to production depts. P1 and P2 in -
the ratio of 2:1
Total (Rs.) 3,150,000 2,625,000 Nil Nil

Budgeted machine hours in Department P1 (Refer to W.N. 1) 105,000

Budgeted machine hours in Department P2 (Refer to W.N. 1) 175,000

Budgeted machine hour rate (Rs. 31, 50,000/1, 05,000) Rs.30

Budgeted machine hour rate (Rs. 26, 25,000/1, 75,000) Rs.15

RTP-CAP II –-2014-December@ICAN Page 103 of 158


Performance report for the month of June 2014

(when 4,000 and 3,000 units of products X and Y respectively were actually produced)

Budgeted Actual

Rs. Rs.

Raw material used in department P1

X: 4,000 units x Rs. 120 480,000 489,000

Y: 3,000 units x Rs. 150 450,000 456,000

Direct Labor

Cost on the basis of labor hours

worked in department P2

X: 4,000 x 2 hrs. x Rs.72 576,000 591,900

Y: 3,000 x 2.5 hrs. x Rs. 75 562,500 552,000

Overhead absorbed

On machine hour basis in department P1

X: 4,000 x 1.5 hrs. x Rs. 30 180,000 174,400*

Y: 3,000 x 1 hr x Rs.30 90,000 118,649

Overhead absorbed

On labor hour basis in department P2

X: 4,000 x 2 hrs. x Rs. 15 120,000 131,364**

Y: 3,000 x 2.5 hrs. x Rs. 15 112,500 118,548

2,571,000 2,631,861

* Refer to W.N. 4; ** Refer to W.N. 5

Working notes (W.N.):

Product X Product Y Total

1. Budgeted output 50,000 30,000


(in units)

Budgeted machine hours 75,000 30,000 105,000

In department P1 (50,000 x 1.5 hrs.) (30,000 x 1 hrs.)

RTP-CAP II –-2014-December@ICAN Page 104 of 158


Budgeted labor hours 100,000 75,000 175,000

In department P2 (50,000 x 2 hrs.) (30,000 x 2.5 hrs.)

2. Actual output 4,000 3,000


(in units)

Actual machine hours

Utilized in department P1 6,100 4,150 10,250

Actual labor hours

Utilized in department P2 8,200 7,400 15,600

3. Computation of actual overhead rates for each production department from actual data

Production dept. Service dept.


Particulars
P1 P2 S1 S2

Actual factory overhead for the month 231,000 204,000 60,000 48,000
of June, 2014 in (Rs.)

Allocation of service dept. S1‟s costs 30,000 30,000 (60,000) -


in (Rs.) over production department
P1 and P2 equally.
Allocation of service department S2‟s 16,000 - (48,000)
costs in (Rs.) over production 32,000
department P1 and P2 in the ratio of
2:1
Total (Rs.) 293,000 250,000 Nil Nil

Actual machine hours in dept.P1 - - -


(Refer to W.N. 2) 10,250

Actual labor hours in dept. P2 - 15,600 - -


(Refer to W.N. 2)
Machine hour rate Rs. 28.59
(Rs. 2, 93,000/10,250)
Labor hour /rate Rs.
(Rs. 2, 50,000/15,600) 16.02

4. Actual overheads absorbed (based on machine hours):


X: 6,100 hrs. x Rs. 28.59 = Rs. 1, 74,400

Y: 4,150 hrs. x Rs. 28.59 = Rs. 1, 18,649

RTP-CAP II –-2014-December@ICAN Page 105 of 158


5. Actual overheads absorbed (based on labor hours):
X: 8,200 hrs. x Rs. 16.02 = Rs. 1, 31,364

Y: 7,400 hrs. x Rs. 16.02 = Rs. 1, 18,548

Answer 4(d)
Cost allocation is defined as the allotment of whole items of cost to cost centers. For e.g. if a labor works
for certain specific department, then the wages paid to him shall be charged to that specific department.
This technique of charging the entire overhead expenses to a cost center is known as cost allocation.
Cost absorption is defined as the process of absorbing all overhead costs allocated to or apportioned over
particular cost center or production department by the units produced. For e.g. the overhead cost of
machine center may be absorbed by the rate per machine hour.
Cost absorption can take place only after cost allocation. In other words, the overhead costs are either
allocated or apportioned over different cost centers and afterwards they are absorbed on equitable basis
by the output of the same cost centers.

Answer 5 (a)
Journal Entries under Integrated System of Accounting
S.N. Particulars Debit (Rs.) Credit (Rs.)
(i) Work in progress Ledger Control A/C 4,25,000
Dr.
Factory Overhead Control A/C 1,15,000
Dr.
To Stores Ledger Control A/C 5,40,000
(Being issue of Direct and Indirect materials)
(ii) Work in Progress Ledger Control A/C 4,87,500
Dr.
Factory Overhead Control A/C 1,62,500
Dr.
To Wages Control A/C 6,50,000
(Being allocation of Direct and Indirect wages)
(iii) Factory Overhead Control A/C 3,50,000
Dr.
To Costing Profit & Loss A/C 3,50,000
(Being transfer of over absorption of Factory
overhead)
Costing Profit & Loss A/C 1,75,000
Dr.
To Administration Overhead Control A/C 1,75,000
(Being transfer of under absorption of
Administrative Overhead)
(iv) Sundry Creditors A/C 2,20,000
Dr.
To Cash/ Bank A/C 2,20,000
(Being Payment made to creditors)
(v) Cash/ Bank A/C 3,60,000
Dr.
To Sundry Debtors A/C 3,60,000
(Being payment received from debtors)
RTP-CAP II –-2014-December@ICAN Page 106 of 158
Answer 5(b)
When the Cost and Financial Accounts are integrated – there is no need to have a separate reconciliation
statement between the two sets of accounts. Integration means that the same set of accounts fulfill the
requirement of both i.e. Cost and Financial Accounts.

Answer 5 (c)
Profit and loss Account of Kamal Manufacturers Ltd.
For the year ended 16 July 2014
(Rs. In thousands)
Particulars Amount Particulars Amount
To, Materials 1,600 By, Sales (20,000 4,000
units)
To, Wages 800 By, Finished goods 240
To, Factory overheads 720 By, Work in progress 112
To, Office & Adm. 416
overheads
To, Selling and dist. 288
overheads
To, Goodwill written off 320
To, Interest on Capital 32
To, Net profit 176
4,352 4,352

Profit as per cost records


(Rs. In thousands)
Particulars Amount (Rs.)
Materials 1,600
Wages 800
Prime cost 2,400
Factory Overheads (100% of wages) 800
Gross Factory cost 3,200
Less:
Closing WIP 112
Factory Cost (21,230 units) 3,088
Add: Office & Administrative Overhead (10% of Factory Cost) 308.80
Total cost of output 3,396.80
Less: Closing stock (1230 units) of finished goods (see working note 1) 196.80
Cost of production of 20,000 units 3,200
Selling & distribution overhead @Rs. 16 p.u. 320
Cost of sales (20,000 units) 3,520
Sales Revenue (20,000 units) 4,000
Profit 480

Reconciliation Statement
Rs. („000) Rs. („000)
Profit as per cost accounts 480
Add: Factory Overhead Over absorbed

RTP-CAP II –-2014-December@ICAN Page 107 of 158


(800-720) 80
Selling and distribution Overhead Over absorbed
(320-288) 32
Closing stock overvalued in Financial Accounts
(240-196.8) 43.20 152.2
635.20
Less: Office & Administrative Overhead under absorbed 107.20
(416-308.80)
Goodwill written off 320
Interest on capital 32 459.20
Profit as per Financial Statements 176

Answer 6 (a)
(i) Statement showing additional claim due to escalation clause:

Material Std. Qty. Std. rate Actual Variation Escalation


(a) (b) rate (c) in Rate Claim (Rs.)
(Rs.) (e) = (a x
(d) = (c-b) d)
A 3,000 1000 1100 +100 +300,000

B 2,400 800 700 -100 -240,000

C 500 4,000 3,900 -100 -50,000

D 100 30,000 31,500 +1,500 +150,000

Material Escalation claim 160,000

Labor Std. Std. rate Actual Variation Escalation


Hours (b) rate (c) in Rate Claim (Rs.)
(a) (Rs.) (e) = (a x
(d) = (c-b) d)
L1 60,000 15 18 +3 +180,000

L2 40,000 30 35 +5 +200,000

Labor Escalation Claim 380,000

Statement Showing Final Contract Price

Agreed Contract Price 15,000,000

Add: Agreed Escalation claim: Rs.

Material Cost 160,000

Labor Cost 380,000 540,000

RTP-CAP II –-2014-December@ICAN Page 108 of 158


Final Contract Price 15,540,000

(ii) Contract Account

Dr. Cr.
Particulars Amount (Rs.) Particulars Amount (Rs.)

To, material: 10,525,000 By, Contractee‟s A/C 15,540,000

A – 3,400xRs. 1,100
B – 2,300xRs. 700
C – 600xRs. 3,900
D – 90xRs. 31,500

To, Labor: 2,338,000


L1 – 56,000xRs. 18
L2 – 38,000xRs.35

To, Other expenses 1,345,000

To, Profit and Loss 1,332,000


A/c
Total 15,540,000 Total 15,540,000

Answer 6(b)
Operating Cost Sheet

For the year 2013-14

(Total passenger Km = 11,520,000)

Particulars Total Cost (Rs.) Cost /passenger Km (Rs.)

A. Fixed Charges
Garage rent (4,000x12) 48,000

Salary of drivers (3,000x5x12) 180,000

Wages of Conductors (1200x5x12) 72,000

Manager‟s Salary (7,500x12) 90,000

RTP-CAP II –-2014-December@ICAN Page 109 of 158


Road Tax, Permit fee, etc. (5,000x4) 20,000

Office expenses (2,000x12) 24,000

Insurance (650,000x3/100x5) 97,500

Total A 531,500 0.046

B. Variable Charges
Repairs and maintenance (22,500x5) 112,500 0.010

Depreciation (650,000x15/100/5) 487,500 0.042

Diesel (360,000/6x Rs. 33) 1,980,000 0.172

Total B 2,580,000 0.224

Total Cost (A+B) 3,111,500 0.270

Add: 33 1/3% profit on takings

or 50% on cost 1,555,750 0.135

Bus fare to be charged from each passenger 4,667,250 0.405


Per km

Working Notes:
(i) Total Kilometers to be run during the year 2013-14
= 40x2x3x25x12x5=360,000 kilometers

(ii) Total passenger Kilometers


=360,000x40x80/100= 11,520,000 passenger km.

Answer 6(c)
Process – I Account
Particulars Kg. Per Kg. Amoun Particulars Kg. Per Amount
(Rs.) t (Rs.) Kg. (Rs.)
(Rs.)
To, Material A 6,000 0.50 3,000 By, Normal Loss 500 0.16 80
To, Material B 4,000 1.00 4,000 By, Abnormal 300 1.00 300
Loss (W.N. 2)
To, Mixing 860 By, transfer to 9,200 1.00 9,200
labor Process II
(430 hrs
@2/hr)
To, Overhead 1,720
Total 10,00 10,00
9,580 Total 9,580
0 0

Process – II Account
Particulars Kg. Per Kg. Amount Particulars Kg. Per Amount
RTP-CAP II –-2014-December@ICAN Page 110 of 158
(Rs.) (Rs.) Kg. (Rs.)
(Rs.)
To, Process I 9,200 1.00 9,200 By, Normal Loss 1,000
A/c
To, material C 6,600 1.25 8,250 By, Work in 1,000 1,160
Process
To material D 4,200 0.75 3,150 By, Packing Dept. 18,000 1.22 21,960
(W.N. 3)
To, Flavoring 300
essence
To, Mixing 740
labor (370 hrs.
@2/hr)
To, Overhead 1,480
Total 20,000 23,120 Total 20,000 23,120

Abnormal Loss Account


Particulars Kg. Per Amount Particulars Kg. Per Amount
Kg. (Rs.) Kg. (Rs.)
(Rs.) (Rs.)
To, Process I 300 1.00 300 By, Sale Account 300 0.16 48
A/c
Balance to P/L a/c 252
Total 300 300 Total 300 300

Packing Department Account


Particulars Kg. Per Amount Particulars Kg. Per Amount
Kg. (Rs.) Kg. (Rs.)
(Rs.) (Rs.)
To, Process II 18,000 1.22 21,960 By, Balance 21,960
A/c
Total 21,960 Total 21,960

Working Notes (W.N.):


1. Total overhead expenses : Rs. 3,200
Total Labor hours in Process I and II: 800

Overhead absorption rate: Rs. 3,200/800 hours: Rs. 4 per labor hour

Overhead under Process – I : 430 x Rs. 4 : Rs. 1,720

Overhead under Process – II : 370 x Rs. 4 : Rs 1,480

2. Cost of 9,500 kg of output is (Rs. 9,580 - Rs. 80) i.e. Rs. 9,500
Hence cost per kg. of output is Re. 1.00

3. Equivalent units Statement of output


Output units Equivalent units

Material Labor Overhead


RTP-CAP II –-2014-December@ICAN Page 111 of 158
Completed 18,000 18,000 18,000 18,000

WIP 1,000 1,000 500 500

(100% material and

50% labor & OH)

Normal Waste 1,000 0 0 0

20,000 19,000 18,500 18,500

Cost Statement for the week ending 24th February 2014


Rs.
Material (Process I) 9,200
Material C 8,250
Material D 3,150
Flavoring Essence 300
Total Material Cost 20,900
Total Mixing Labor Cost 740
Total Overhead Cost 1,480

Cost per Equivalent Unit:


Material = Rs. 20,900/19,000 = Rs. 1.10
Labor = Rs. 740/18,500 = Re. 0.04
Overhead = Rs. 1,480/18,500 = Re. 0.08

Work in Progress:
Material (1,000 x Rs. 1.10) = Rs. 1,100
Labor (500 x Re. 0.04) = Rs. 20
Overhead (500 x Re. 0.08) = Rs. 40
Total Cost = Rs. 1,160

Answer 6(d)
The main point of distinction between Job Costing and Process Costing are as follows:
Job Costing Process Costing
1 Job costing is a specific order costing Process costing is a method of costing used to
ascertain the cost of a product at each stage of
manufacture.
2 Cost here is determined on job basis Costs are accumulated for each process
separately for a given period of time.
3 Each job needs special treatment and no Finished product of one process becomes the
two jobs are alike. raw material for the next process.
4 Costs are computed when job is Costs are computed for each process at the end
completed. of each period.
5 As each job is distinct or is of different As the process operations are standardized,
nature, more detailed supervision and accumulation of costs and supervision and
control are necessary. control are comparatively easier.

RTP-CAP II –-2014-December@ICAN Page 112 of 158


RTP-CAP II –-2014-December@ICAN Page 113 of 158
Answer 6(e)
Statement showing Monthly Profitability

Based on existing operations Based on further processing of P


into S
Particulars Product P Product Q Total Product S Product Q Total

(Rs.) (Rs.) (Rs.) (Rs.) (Rs.) (Rs.)

Sales quantity (Kg.) 47,500 95,000 142,500 47,500 95,000 142,500

Sales Revenue 570,000 1,900,000 2,470,000 712,500 1,900,000 2,612,500


(Refer to W.N. 4)

Less: Joint Costs 510,000 1,700,000 2,210,000 695,000 1,700,000 1,595,000

(Refer to W.N.5)
Profit 60,000 200,000 260,000 17,500 200,000 217,500

Recommendation: Further processing of P is not recommended as it results in a lower profit of P.


Working notes (W.N.):
Kgs.
1. Material input 150,000
Less: Loss of material in process 7,500

(5% of 1, 50,000)

Total output 142,500

2. Output of P and Q are in the ratio of 1:2 of the total output


P = (1, 42,500 / 3) x 1 = 47,500 kg.

Q = 1, 42,500 / 3 x 2 = 95,000 kg.

3. Joint Costs Rs.


Material (input) (1, 50,000 kg x Rs. 12) 1,800,000

Direct Material 90,000

Direct Wages 120,000

Variable Overheads 100,000

Fixed Overheads 100,000

2,210,000

4. Sales Revenue of P, Q and S


P= 47,500 x Rs. 12 = Rs. 570,000

Q= 95,000 x Rs. 20 = Rs. 1,900,000

RTP-CAP II –-2014-December@ICAN Page 114 of 158


S = 47,500 x Rs. 15 = Rs. 712,500

5. Apportionment of joint costs viz. Rs. 22,10,000 over P and Q in proportion of their sales value i.e. Rs
5,70,000 and Rs. 19,00,000 i.e. 3:10 is:
Total P Q

Rs. Rs. Rs.

Joint Cost 22, 10,000 5, 10,000 17, 00,000

(Apportionment in the ratio

of 3:10) 22,10,000 x 3 22,10,000 x 10

13 13

6. Total Cost of 47,500 kg. of S = Joint Cost of P + Cost of Processing P into S

= Rs. 510,000 + Rs. 185,000

= Rs. 685,000

Answer 7(a)
Margin of Safety (MoS) is the excess of total sales over the Break even sales. MoS defines the amount
upto which level sales can decline before occurring loss. Therefore;
MoS = Total Sales - Break even sales and MoS ratio = Sales – Break even sales
Sales
Break Even Sales (BE sales) will depend on Contribution margin (BE sales = Fixed Cost ÷ Contribution
margin). Contribution margin is related to operating leverage also. Operating leverage is calculated as
Contribution ÷ Operating profit and contribution margin plays an important role in it. If sales are
expected to increase, higher operating leverage will result in higher profit. When sales are expected to
decrease, lower operating leverage will result in higher profit. Higher variable cost and lower fixed cost
will result into higher MoS and risk will be lower and vice versa.

So like Operating leverage, MoS is a measure of risk as to what extent an organization is exposed to
change in sales volume.

Answer 7 (b)
Key factor is a factor which at a particular time or over a period limits the activities of an undertaking. It
may be the level of demand for the products or service or it may be the shortage of one or more of the
productive resources.
Examples of key factors are:
a. Shortage of raw material
b. Shortage of labor
c. Plant capacity available.
d. Sales capacity available.
e. Cash availability.

Answer 7(c)
(i) Computation of Unit Cost and Total Income

RTP-CAP II –-2014-December@ICAN Page 115 of 158


Unit Cost Absorption Costing (Rs.) Marginal Costing (Rs.)

Direct Material 16.00 16.00

Direct Labour 54.00 54.00

Variable Overhead 12.00 12.00

Fixed Overhead 18.00 -

Unit Cost 100.00 82.00

Income Statements
Absorption Costing
Sales
(21,500 x Rs. 168) 3,612,000
Less: Cost of Goods Sold (21,500 x 100) 2,150,000
Less: Over absorption 28,000 2,122,000
1,490,000
Less: Selling and Distribution Expenses 1,190,000
Profit 300,000

Marginal Costing
Sales 3,612,000
Less: Cost of Goods Sold (21,500 x 82) 1,763,000
Add: Under Absorption 20,000 1,783,000
1,829,000
Less: Selling and Distribution Expenses 430,000
Contribution 1,399,000
Overhead (38,000+7, 60,000) 1,144,000
Profit 255,000

(ii) Under or over absorption of overhead:

Budgeted Fixed Overhead Rs.

72,000 Hrs. x Rs. 6 4, 32,000

Less: Actual Overhead was less than budgeted fixed overhead 48,000

Actual Fixed Overhead 3, 84,000

Budgeted Variable Overhead

72,000 Hrs. x Rs. 4 2, 88,000

Add: Actual Overhead was higher than Budgeted 20,000

Budgeted 3, 08,000

Both Fixed and Variable Overhead applied

72,000 Hrs. x Rs. 10 7, 20,000

Actual Overhead (3, 84,000 + 3, 08,000) 6, 92,000


RTP-CAP II –-2014-December@ICAN Page 116 of 158
Over absorption 28,000

(iii) Reconciliation of Profit


Difference in Profit: Rs. 3, 00,000 – 2, 55,000 = Rs. 45,000

(Due to Fixed Factory Overhead being included in closing stock in Absorption Costing not in
Marginal Costing.)

Therefore,

Difference in profit = Fixed Overhead Rate (Production – Sale)

18 (24,000 – 21,500) = Rs. 45,000

Answer 7(d)

The difference between Marginal Costing and Absorption Costing are narrated as below:

Absorption Costing Marginal Costing

1 It is a total cost technique, i.e. both variable Only variable costs are charged to products,
and fixed costs are charged to products, processes or operations. Fixed costs are charged
processes or operations. as period costs to the profit statement of the
same period in which they are incurred.

2 Fixed factory overheads are absorbed by the The cost of production under this method does
production units on the basis of a not include fixed factory overheads and
predetermined fixed factory overhead recovery therefore, the value of closing stock comprises
rate based on normal capacity. Under/over- of only variable costs. No part of the fixed
absorbed overheads are adjusted before expenses is included in the value of closing
arriving at the figure of profit for a particular stock and carried over to the next period.
period.

3 In spite of best possible forecast and equitable Since fixed overheads are not included in the
basis of apportionment/ allocation of fixed cost of production, therefore the question of
costs, under or over recovery of fixed their under/over recovery does not arise.
overheads generally arises.

4 Managerial decisions under this costing Here decisions are made on the basis of
technique are based on profit, i.e. excess of contribution, i.e. excess of sales price over
sales value over total costs, which may at times variable costs. This basis of decision making
lead to erroneous decisions. results in optimum profitability.

Answer 8 (a)
Following are the types of budgets prepared by a business:
i. Sales Budget
ii. Production Budget
RTP-CAP II –-2014-December@ICAN Page 117 of 158
iii. Plant Utilization Budget
iv. Direct Material Usage Budget
v. Direct Material Purchase Budget
vi. Direct Labor (Personnel) Budget
vii. Factory Overhead Budget
viii. Production Cost Budget

Answer 8 (b)
Statement of Flexible Budget
(Rs. in Lakhs)
Particulars 50% 60% 75% 90% 100%
Capacity Capacity Capacity Capacity Capacity
Variable expenses:
Materials 21.70 26.04 32.55 39.06 43.40
Labor 20.40 24.48 30.60 36.72 40.80
Other expenses 7.90 9.48 11.85 14.22 15.80
Semi-variable expenses:
Maintenance and repairs 3.50 3.50 3.85 4.20 4.20
Indirect labor 7.90 7.90 8.69 9.48 9.48
Sales department (salaries 3.80 3.80 4.18 4.56 4.56
etc.)
Sundry administrative 2.80 2.80 3.08 3.36 3.36
expenses
Fixed Expenses:
Wages and Salaries 9.50 9.50 9.50 9.50 9.50
Rent, rates and taxes 6.60 6.60 6.60 6.60 6.60
Depreciation 7.40 7.40 7.40 7.40 7.40
Sundry administrative 6.50 6.50 6.50 6.50 6.50
expenses
Total estimated Cost 98.00 108.00 124.80 141.60 151.60
Estimated Profit 2.00 12.00 25.20 38.40 48.40
Sales 100.00 120.00 150.00 180.00 200.00

Answer 9 (a)
Uniform costing refers to the use of the same costing principles and practices by several undertakings.
These undertakings may or may not be under the same management. Adherence to the same costing
methods and procedures especially when there can be two or more options is the characteristic feature of
a uniform costing.
Points on which uniformity is essential before introducing uniform costing system are:
i. The firms in the industry should be willing to share/ furnish relevant data/information.
ii. A spirit of cooperation and mutual trust should prevail among participating firms.
iii. Mutual exchange of ideas, methods used, special achievements made, research and know how etc. should
be frequent.
iv. Bigger firms should take the led towards sharing their experiences and know-how with the smaller firms
to enable the latter to improve their performance.
v. Uniformity must be established with regard to several points before the introduction of uniform costing in
an industry. In fact, uniformity should be with regard to following points:
 Size of various units covered by uniform costing.
 Production methods
 Accounting methods, principles and procedures used.

RTP-CAP II –-2014-December@ICAN Page 118 of 158


Answer 9 (b)
It is a written document, which may be in the form of a booklet or bulletin, containing principles,
methods and procedures for the ascertainment and control of cost in uniform costing. It is necessary for
the successful operation of uniform costing system. Such a manual provide guidelines to the participating
firms to organize their cost accounting system on a uniform basis.
The following are the salient features of a uniform cost manual:
 It includes statement of objectives and purpose of the system, scope of the system, advantages and extent
of co-operation necessary.
 It contains the general principles of accounting, nature coding, terminology to be followed,
classification and description of accounts. This section also includes details of stock control, labor
and overhead cost collection and control.
 Essential cost data and various ratios to be computed for comparison of performance and
efficiency in the operation of the participating units.
 Mode, format and time for presenting cost data and reports to the management.
 It provides necessary guidelines about the treatment of depreciation, interest on capital, wastage,
scrap, by-product, etc.

Answer 9(c)
The following are the limitations in the implementation of a scheme of inter-firm comparison:
a. Top management feels that secrecy will be lost.
b. Middle management is usually not convinced with the utility of such a comparison.
c. In the absence of a suitable cost accounting system, the figure supplied may not be reliable for the
purpose of comparison.
d. Suitable basis of comparison may not be available.

Answer 10
It is one of the important tools of modern management in the areas of cost reduction. It is also known by
the other names such as value engineering, value control and product research. Value analysis is the
process of systematic analysis and evaluation of various technique and functions with a view to improve
organizational performance. It aims at reducing and controlling the cost of a product from the point of
view of the value by analyzing the value currently received. It investigate into the economic attributes of
value analysis, believes in a planned action to improve performance and thereby, generates higher value
in a product and ultimately causes reduction in its cost.
The meaning of the term value may vary from person to person, time to time and place to place.
However, in the context of cost reduction and control it refers to the „use value‟.
The reduction in the costs of a product and thus increasing the profitability of a concern is the main
advantage of value analysis.
The benefits of value analysis are being derived in many industries, e.g. engineering, building
construction and the oil industry. It is being applied to components of a product, finished product and
also to the methods of packaging.
The various steps involved in value analysis are:
i. Identification of the problem.
ii. Collecting information about the function, design, material, labor, overhead costs, etc. of the product and
finding out the availability of the competitive products in the market, and
iii. Exploring and evaluating alternatives and developing them.

Answer 11
Cost audit offers valuable assistance to the management in its decision making process by examining the
reliability of cost accounting data and information. Due to the assistance provided by the cost audit,

RTP-CAP II –-2014-December@ICAN Page 119 of 158


management is in a position to know what price is to be fixed for a product, whether the wastage are
avoidable, whether to re-organize sales or inventory system, to make the work more efficient and so on.
Also cost audit is a great help in maintaining internal control and internal check and can be of advantage
even to the statutory financial auditor. Cost audit, apart from having all the normal ingredient of audit,
i.e. vouching, verification, etc. has within its domain elements of efficiency audit and propriety audit as
well.
Efficiency audit is directed towards the measurement of whether corporate plans have been effectively
executed. It is concerned with the utilization of resources in economic and most remunerative manner to
achieve the objectives of the concern. It comprises of studying the plans of organization, comparing
actual performance with plans and investigating the reasons for variances to take remedial action. For
example, the effective utilization of capital in an organization can be gauged by determining the return
on capital employed.
Propriety audit is concerned with the executive actions and plans bearing on the finances and expenditure
of the company. The cost auditor has to judge:
i. Whether the planned expenditure is designed to give optimum results.
ii. Whether the size and channels of expenditure were designed to produce the best results.
iii. Whether the return from expenditure on capital as well as current operations could be bettered by
some other alternative plan of action.

Usefulness of Cost audit to shareholders:


Cost audit ensures that proper records are kept as to purchases and utilization of materials and expenses
incurred on wages, overheads, etc. It also ensures that the unit has been run economically and efficiently.
It also make sure that the valuation of closing stocks and work in progress is on a fair basis. Thus, the
shareholders are assured of a fair return on their investment.

INCOME TAX AND VAT

INCOME TAX
Q. No. 1
M/s Surakshit General Insurance Co. Ltd. has furnished the following information related to F/Y 2069/70
Particular Amount (Rs)000
Net Premium Received 400,000.00
Commission on Insurance Ceded 20,000.00
Opening unexpired Risk reserve 150,000.00
Opening claim out standing 23,000.00
Commission on Reinsurance
accepted 10,000.00
Agent Commission 15,000.00
Management Expenses 100,000.00
Claim paid during the year 100,000.00
Closing claim outstanding 30,000.00
Interest Income in fixed
deposit(Gross) 50,000.00
Allowable Deprecation 60,000.00
Miscellaneous Income 25,000.00
Claim received from Reinsurance 50,000.00
Carried Forward loss for F/Y
2068/69 100,000.00

Additional Information
RTP-CAP II –-2014-December@ICAN Page 120 of 158
i. Sale of salvage accepted by the Company from the insured at the time of claim settlement of Rs.
56,500 including VAT is not recorded above.
ii. Management expenses includes the following
 Telephone expenses of Rs. 100,000 of Jestha & Ashadh 2069.
 Claim payment includes of Rs.550, 000 paid through bearer cheque to Jhunjhun Impex,
Kath
Particular F.Y 2071/72 F.Y 2069/70 mand
u,
Loan Outstanding at the year end 1,200,000.00 1,000,000.00
whic
Loan Loss provision (LLP) at year end as per NRB
h has
Directives 75,000.00 60000.00
annu
Profit before Tax without adjusted LLP & Depreciation 500,000.00 400000.00 al
turno
LLP Expenses during the year 15,000.00 60000.00 ver
of 15
Deprecation 100,000.00 - lakhs
only.
iii.
n F.Y 2068/69, the company has taxable profit of Rs. 50,000,000 before adjusting carried forward
loss.

Required
i. Calculate Taxable Income &Income Tax, if no any advance tax has been paid.
ii. Calculated penalty & interest chargeable under section 117, 118 & 119 (Ignore penalty of
estimated tax return Provision)
 Assume Income Tax Return has been filed within Poush end 2070, time extension in this
regard was obtained upto Mangsir end 2070. Ignore TDS receivable on interest income.

Q. No. 2
PQR Bank Ltd. has furnished the following information.

Additional Information
Deprecation includes Rs. 25,000 on ATMS which is deducted @ 25%, Rs. 20,000 on car @ 20%, Rs.
20,000 on Building constructed on leased land of 10 years @ 10% p.a.
F.Y 2069/70 is first year of operation.
Required
Calculate Income Tax Liability for the F/Y 2069/70 & 2070/71.
Q. No. 3
Mr. Bhai Raja is a person working in a Financial Institution as a Chief Manager. Besides working in
Financial Institution, he is a professor of Account in Shankar Dev Campus. He generates the following
income from the employers in Fiscal Year 2070/71.
i. Monthly Remuneration NPR 40,000; Dashain Allowance NPR 40,000; Education Allowances
NPR 2,000 per month; Expenditure Allowance NPR 2,000 per month; Bonus NPR 60,000;
Remuneration from teaching in Shankar Dev Campus NPR 6,000 per month.
ii. He is provided a car along with driver for his official and personal purpose. Monthly remuneration
and allowance of driver is NPR 8,000.

RTP-CAP II –-2014-December@ICAN Page 121 of 158


iii. Petrol Expenditure is provided for his car which is NPR 10,000 per month and maintenance
expenditure of such car is NPR 25,000 in Fiscal Year 2070/71
iv. He is provided a housing facility by the employer.
v. Monthly expenditure of telephone connected in his resident is NPR 2,000 per month. Out of this
50% is his personal telephone expenditure.
vi. A security guard is being provided in his resident and it is being paid NPR 6,000 per month by the
employer. Because of being used the security guard in his resident, NPR 3,000 is being deducted
from his monthly salary.
vii. He has used the loan amounting to NPR 10,00,000 under housing loan facility at the rate of 5%
interest . 8% interest rate is being charged by the institution under similar loan to other borrowers.
viii. He has received one month salary against his house leave during Mangsir, 2070.
ix. The employer contributes 10% of his drawn salary and the employee same amount from his
salary, to approved provident fund. Also, the employer deposited NPR 20,000 per month to
Citizen Investment Fund up on request of employee by debiting his account.
x. He has covered the insurance of NPR 250,000 and NPR 150,000 of his wife in Rastriya Beema
Sasthan. He pays the premium of NPR 19,000 and NPR 14,000 respectively in that Fiscal Year.
Also, he has contributed to tax exempted entity approved from department as donation amounting
to NPR 60,000.
xi. He has expensed out NPR 20,000 for his treatment in Teaching Hospital.
xii. He and his wife declared that they are couple in the Fiscal Year 2070/71and his wife does not
have any income in that Fiscal Year.
xiii. Assume 12 month salary.

Required
a. Compute Assessable Income, Taxable Income and Tax Liability of Mr. Babu kaji for the Fiscal Year
2070/71.
b. Does he needs to file Income Tax Return?

Q. No. 4
Rolls Airways is conducting the airplane services between Kathmandu and Saudi Arabia. Rolls Airways
has a branch office at Kathmandu for the purpose of operation of its services. The income of Rolls
Airways for the F/Y 2070/71 while conducting the airplane services and other operations are as below:
SN Particulars Amount (Rs.)

1 Amount received from the passengers departed from Kathmandu 10,000,000

2 Amount received from the transportation of the goods having first point 5,000,000
entry at Kathmandu.

3 Income from the travels from Bhutan and Lhasa having transit point at 5,000,000
Kathmandu

4 Amount received from the rentals of the goods carried from Kathmandu 5,000,000
which were transported from Bhutan

5 Amount received from the services of restaurant operated for the facility of 2,500,000
passengers at Kathmandu Airport

6 Amount received from the services of Ground Handling for other airlines‟ 2,500,000

RTP-CAP II –-2014-December@ICAN Page 122 of 158


operation

Total Income 30,000,000

The followings are the details of expenditure of Rolls Airways:

SN Particulars Amount (Rs.)

7 Expenditures for Air tickets 5,000,000

8 Expenditures against the accommodation and fooding for crew members 5,000,000

9 Expenditures against ground handling services 3,000,000

10 Expenditure for the operation of restaurant 2,000,000

11 Expenditure for the transportation of passenger from Bhutan, Lhasa 5,000,000

12 Expenditure for the re-transportation of the goods transported from Bhutan 5,000,000

Total Expenditure 25,000,000

Note: Rolls Airways has managed the ground handling against 100 flights of its own and 100 flights of
other airlines during Fiscal Year 2070/71.

Explain about the inclusion of above incomes and allowances of expenditures of a non-resident person, for
the purpose of section 70 of Income Tax Act, 2058 stating the relevant provisions and explain the income
& expenses other than relevant to section 70.

Q. No. 5
Mr. Ramnarayan has following income during F/Y 2070/71.
 Business Income 1,900,000
 Income from sale of land 1,000,000

Land here falls under Non Business Chargeable Assets.


Calculate income Tax if opted as couple.
Q. No. 6
XYZ Pvt. Ltd. is a trading firm of Mr. X being single shareholder; has the following profit & loss
account for the F/Y 2070/71
in Rs. „000
Amount
Particulars Amount (Dr.) Particulars (Cr.)

To, Opening Stock 20,000.00 By, Sales 100,000.00


To, Purchase 80,000.00 By, Closing Stock 20,000.00
To, Direct Expenses 5,000.00 By, Bad debt recovered 1,000.00
To, Gross Profit b/d 16,000.00
121,000.00 121,000.00
To, Office Expenses 5,000.00 By, Gross Profit c/d

RTP-CAP II –-2014-December@ICAN Page 123 of 158


16,000.00
To, Bank Interest 4,000.00
To, Profit & Loss 7,000.00
16,000.00 16,000.00

Additional Information

i. Direct Expenses includes the followings:


 Rs. 100,000 is a freight bill of 2070.03.28.
 Rs. 200,000 is an expenditure incurred in weeding ceremony of son of Mr. X.
 Rs. 100,000 is a rent paid to house owner through fund transfer from companies bank to
bank of house owner.
ii. Closing Stock is undervalued by Rs. 5,000,000.
iii. The firm has sale of Whiskey & Rice.
iv. Bad debt recovered of Rs. 4 lakhs was not allowed by tax authorities in previous year as expenses.
All bad debt recovered is related to previous years.

Required
Calculate the income tax of XYZ Pvt Ltd for F.Y 2070/71
Q. No. 7
Calculate the maximum allowable Deprecation & Repair and Improvement expenses for a Special
Industry and unabsorbed Repair & Improvement expenses.

Additional Information
Repair &
Opening Purchase / Improvement
Block WDV (Sales) Total Expenses
A 5,000,000.00 15,000,000.00 20,000,000.00 1,400,000.00
B 2,000,000.00 (1,000,000.00) 1,000,000.00 700,000.00
C 10,000,000.00 (5,000,000.00) 5,000,000.00 350,000.00

D 30,000,000.00 - 30,000,000.00 3,000,000.00


i. Purchase of Block A is Rs. 50 lacs each for three times at Poush end, Chaitra end & Ashad
end respectively.
ii. Sale of Block B is at Poush end
iii. Sale of Block C is at Ashad end.
iv. The industry was registered at the Office of Company Registrar.

Q. No. 8
Answer the followings in the light of Income Tax Act, 2058.
i. ABC Investment Company Ltd. is involved in trading of shares listed in NEPSE. It has earned profit of
Rs.20 lakhs in F/Y 2070/71 from transaction and paid capital gain tax of Rs.2 lakhs and earned Rs.5 lakhs
dividend from different listed companies. What is amount of Income tax to be paid if business related
expenses are of Rs.5 lakhs?
ii. Bridge (P) ltd. is a company running rope -way carrier at Tamor Khola earned Rs.10 lakhs taxable profit
in F/Y 2070/71. What is the amount of income tax to be paid?

RTP-CAP II –-2014-December@ICAN Page 124 of 158


iii. XYZ Bank Ltd. earned Rs. 1,000 lakhs in F/Y 2070/71. It is listed in NEPSE. What is the amount of
income tax to be paid?
iv. PQR Power Ltd. is a listed Company in NEPSE and its estimated taxable earning is Rs. 1,000 lakhs in F/Y
2071/72 from sale of electricity to NEA and subsidy of Rs.500 lakhs as per budget speech 2071. What
shall be the amount of income tax to be paid? ( Ignore income tax holiday)
v. Mr. Ram Liquor Trader earned Rs. 40 lakhs, opted as couple for F/Y 2070/71. What is the amount of tax
to be paid, if he contributed Rs.2 lakhs for approve retirement fund &Rs. 40,000 Life Insurance Premium?
His wife is non-resident.

Q. No. 9
Answer the following in the light of Income tax Act 2058.
a) Define the followings
i. Permanent Establishment
ii. Non Business Chargeable Assets
iii. Exempt Organization
iv. Company
v. Service Fee
b) Calculate the amount of withholding tax to be deducted by resident person on following payment
with reference to Income Tax Act 2058.
i. Service fee of Rs. 98 lakhs paid to Timro Hamro Bank Ltd., Kathmandu.
ii. Dividend of Rs. 10 lakhs paid to the natural person by ABC Mutual Fund which includes
Rs. 5 lakhs from dividend income from resident companies.
iii. Commission of Rs. 11.30 lakhs including VAT paid by a manufacturer of cement to the
wholesaler against Tax Invoice issued by wholesaler to manufacturer.
iv. Reinsurance premium of Rs. 10 lakhs paid to non-resident Insurance Company.
v. A Mutual Fund Company gave loan of Rs. 100 lakhs to a resident company, in return it
received Rs. 110 lakhs after one year at the time of loan repayment.
vi. Fee of Rs. 10,000 by ICAN to a Chartered Accountant for preparing a Revision Test
Paper.
vii. Service fee of Rs. 100,000 paid to local consultant not register in VAT.
viii. A resident employment company paid commission of Rs. 5 lacs to non-resident.
ix. Profit of Rs. 20 lakhs distributed by a partnership firm.
x. Rent of land, land & building & furniture Rs50,000 each.

Q. No. 10
Write short notes on the followings in the light of Income Tax Act, 2058.
i. Advance /Personal Ruling
ii. Public Circular
iii. Jeopardy Assessment

Q. No. 11
Discuss the followings in the light of Income Tax Act, 2058.
a) Provision of loan loss for Bank & Financial Institutions.
b) Provision of loan loss for Co-operatives.
c) Final Withholding Payments.
d) Administrative Reviewable Decisions.
e) Estimated Tax Installment.

Q. No. 12
Write short notes of the followings with reference to The Income Tax Act, 2058.
a) Cash payment
RTP-CAP II –-2014-December@ICAN Page 125 of 158
b) What is change in control and how is it calculated?
c) Fee on belated tax return and failure to maintain accounts & records.
d) Different class of depreciable assets, assets included in it & rate of deprecation.

Q. No. 13
Write short notes of the followings the followings with reference to Income Tax Act, 2058.
a) Perquisites for vehicle facility provided to employee and others for personal use.
b) Perquisites for house facility provided to employee and other for personal use.
c) Payments not included in the income from employment.
d) Income from business.
e) Allowable Retirement Contribution.

Q. No. 14
Write short notes of the followings with reference to the Income Tax Act, 2058.
a) Provision of submission of return for tax exempted organization.
b) Provision of collection of due tax in installment.
c) Transfer of assets to spouse or former spouse.
d) Right of Tax Payer.
e) Average tax rate of Nepal.

Q. No. 15
Write short notes of the followings with reference to the Income Tax Act, 2058.
a) Couples
b) Repair &Improvement expenses
c) Tax Exempted Incomes
d) Medical tax credit & approved medical expenses.
e) Basis of tax accounting.

Q. No. 16
a) Answer the following in the light of Value Added Tax Act 2052.

Tasty Cola Pvt. Ltd. (company) is a manufacturer of beverages. It sells its product on reusable glass
bottle. It receives deposit from dealers for such glass bottle supplied to them at the time of sale of
beverages. As per the contract between Company and its dealers, the deposit shall be refunded to the
dealers at the time of full and final settlement i.e. closer of business of dealers. It is also agreed that the
shortage of bottle shall be adjusted at the prevailing cost price to the company at full & final settlement
date. A Tax Officer during his inspection of the company, found that the bottle worth Rs. 30 Lacs
(150,000 bottles at average rate of Rs. 20) were not in physical stock. He makes tax assessment order and
treated the shortage of bottles as sold by the company. The company has proved that the bottles were sent
to dealers; deposit has been received and shown as liability under „Bottle Deposits‟ in accounts.
However, the tax officer did not accept the contention and charged tax & penalty on such shortage.
Meanwhile a dealer came with application to close the business and returned only 40,000 bottles which is
lower than supplied quantity i.e. 50,000 bottles and his deposit was Rs 12.50 Lacs at the rate of Rs. 25
each bottle but prevailing cost to the company as on date of closure of business is Rs. 30 per bottle.
Required
i. Is contention of the Tax Officer correct?
ii. What is to be done while doing full & final settlement of dealer as per VAT Act?

b) What are different tax period under VAT Act & rules?

Q. No. 17
RTP-CAP II –-2014-December@ICAN Page 126 of 158
Write short note of the followings with references to Value Added Tax Act 2052.
a) VAT on commercial complex as per section 8(3).

b) Provision of VAT exemption and zero rated facility to the hydropower.

c) Provision on VAT collected by local diary and tea producer.

d) Provision of VAT exemption on


i. Cultural program
ii. Entry fee to Zoo
iii. Entry fee to Botanical garden
iv. Supply of Manpower abroad
v. Writer getting fee of Rs. 50 lacs against writing of his autobiography from publisher.

RTP-CAP II –-2014-December@ICAN Page 127 of 158


Q. No. 18
Answer the followings:
a) Ram & Sons gave a heavy printer lying in stock, which has cost of Rs 125,000 since 2010 A.D. to
Shyam Brothers. In return, it has received a Laptop having current market price of Rs. 35,000. The
printer was not purchased by anyone while it was kept on sale at Rs. 45,200 including VAT.
What is the taxable Value to Ram & Sons on such transaction?

b) Mr. Ray is going to sell his wood of private land for commercial purpose at Rs. 10,000,000. Other
relevant information are as follows:
 Auction price of equivalent wood of national forest is Rs. 11,000,000.
 Royalty from equivalent wood of national forest is Rs. 12,000,000.
Calculate the taxable value of wood for private land.

c) A-One recondition is a used goods dealer. During a month following transactions were done
Particulars Purchase VAT on Purchase Sale

Sofa set 10,000 1,300 22,000


Table 10,000 1,300 11,000 Other Information
Chair 2,000 - 3,000 Purchase & Sales Register
Total 22,000 2,600 36,000 were not approved form
IRO.
Required
i. Calculate taxable value for each items sold & VAT to be collected.
ii. What is the penalty for not getting Purchase & Sales Register approved from IRO?

d) Tamang & Sharma J/V is a contactor; it has purchased cements of Rs. 1,130,000 including VAT on
the day of registration of PAN (not VAT) only. Next day of PAN registration, they got registered
under VAT. Later on, they knew that VAT paid on such goods can‟t be set off with VAT collected on
sale later as it was purchased before the date of VAT registration. They seek your advice.

Required
Can they set off such VAT? If yes what is the procedure?

RTP-CAP II –-2014-December@ICAN Page 128 of 158


Q. No. 19
Answer the following in references with Value Added Tax Act 2052.
a) Hotel Zoom has following transactions in the month of Shrawan 2071.
Particulars Amount (in 000)
Sales
Room Sales 10,000
Food & Beverages Sale 2,500
Laundry 500
Total 13,000
Purchase
Delivery Van 1,500
Car 2,000
Diesel for Van 200
Above figure of sale are excluding service
Petrol of Car 250
charge of 10% and VAT which it collects
Wine for sale 500
from customer and purchase are also
Food items 700
excluding VAT. Purchased food items used
Other Expenses 100
in daily meal for staff is of Rs. 100,000. It
Total 5,250
has no opening VAT receivable. During the
month it has constructed a building of Rs. 55 lacs, the detail of cost are as follows:
Particulars Cost Amount Remarks
Bricks 5 lacs Without VAT
Cement 15 lacs 50% in VAT
Iron 10 lacs VAT
Other Material 10 lacs VAT
Required
Wage 10 lacs Not in VAT
Calculate the amount of
Consultancy to Engineers 5 lacs Not in VAT
VAT to be paid for
55 lacs
month of Shrawan 2071.
b) When Debit or credit note to be issued and what are its contents?
c) Whether following person required registering under VAT?
i. Liquor shop having turnover of Rs 200,000 p.a. at Baneshwor, Kathmandu Metropolitan City
ii. Person supplying software
iii. Ice cream industry at XYZ Municipality
iv. Tailoring Shop with shirting suiting fabrics on sale at Bagbazar, Kathmandu Metropolitan City
v. Company providing Trekking service

Q. No. 20
Answer the following with references of VAT Act 2052.
a) Mr. Narayan is a small vender registered in VAT voluntarily on 02.04.2071 under monthly tax
period. After three months he got frustrated to comply with the provision of VAT. He wants to cancel
the registration under VAT on Poush 2071.
State the provision of the VAT Act for deregistration on such circumstances.

b) Following are the conditions of a tax payer deregistering under VAT.


Amount in Lacs
VAT Credited Items Cost WDV Mkt. Price
Stock-A 100 - 110
Stock-B 150 - 120
Car 30 (excluding VAT) 10 15
Opening VAT receivable from previous tax period is Rs. 30 Lacs
RTP-CAP II –-2014-December@ICAN Page 129 of 158
Required
What is the amount of VAT to be paid as deemed disposal?
c) State the provision of bank guarantee facility available to taxpayer.
d) State the provision of person collecting VAT other than VAT registered person.

Q. No. 21
Answer the following in the reference with VAT Act 2052.
a) Zip Zap Zoom is a Modeling Company. It got service from Indian company and paid reserve VAT of
Rs. 1.3 lacs at bank of concerned Inland Revenue Office. It offset the VAT paid as reverse VAT. Tax
Officer at the time of tax assessment did not give tax credit on such reverse VAT stating “to claim
VAT credit, there should be tax invoice”.

Required
i. Is Contention of Tax Officer as per provision of VAT Act?
ii. Is there any chance where tax credit is available without tax invoice?

b) Easy Soap Pvt. Ltd. has been running its taxable business since last 10 years. Its account hasn‟t been
assessed by Tax office since last 7 years. Now, during the audit a tax officer demanded records &
documents relating to VAT from previous 7 years onwards. The chairman of the company says he
has no documents early to 4 years since it is not mandatory to maintain.
Discuss the relevant provision of the VAT Act.

RTP-CAP II –-2014-December@ICAN Page 130 of 158


c) State the provision of VAT refund for followings:
i. Significant exporter
ii. Continuous six month credit
iii. VAT paid by tourist
iv. VAT paid by diplomats

d) Is manager working at an entity liable to pay VAT which has to be paid by the entity? State the
provision of the VAT Act.

SUGGESTED ANSWERS

Answer 1

Calculation of Income & Income tax liability of Surakshit Insurance Company ltd.

a) Calculation of taxable Income & Income Tax (Rs ‘000)

Amount included in Income


Net Premium Received 400000
Commission on Insurance ceded 20000
Opening Unexpired Risk Reserve 150000
Opening Claim Outstanding 23000
Interest Income 50000
Miscellaneous Income 25000
Claim Received from Re-
insurance 50000
Sale of Salvage(56500/1.13/1000) 50
Total (A) 718050

Amount included in deductible exp.


Commission on re-insurance accepted 10,000.00
Agent commission 15,000.00
Management Exp.(100000-100-550) 99,350.00
Claim Payment during the Year 100,000.00
Closing Claim Outstanding (30000*115%) 34,500.00
Closing Unexpired Risk Reserve
200,000.00
(50% of Net Premium Received) (400000*50%)
Depreciation 60,000.00
Carry forward loss (Sec 20)
[50,000 is adjusted in fiscal year 68/69] 50,000.00
Total (B) 568,850.00
Taxable Income (A-B) 149,200.00
Income Tax @ 30% 44,760.00
Notes:
i. Expenses not related to income year 2069/70, is not allowed as per Section 13,
ii. Cash payment of Rs. 550000 is not allowed as per Section 21,
RTP-CAP II –-2014-December@ICAN Page 131 of 158
iii. Rs. 5 crore carry forward loss is adjusted from taxable profit of FY 2068/69, rest 5 crore is
available for the year.

Calculation of Penalty u/s 117


0.1% *718050*3/12= Rs 179.512(in 000)= Rs 179512

Calculation of Interest u/s 118 ( upto to Aswin)


Installment Months Tax amount Interest @ 15%p.a ( Rs 000)
1st 3 months 17,904.00 671.40
2nd 3 months 31,332.00 1,174.95
3rd 3 months 44,760.00 1,678.50
Total 3,524.85

Calculation of Penalty u/s 119


= 44760 x 15% x 3/12
= 1678500

Answer 2

Calculation of Income Tax Liability of PQR Bank Ltd


Particulars 2070/71 2069/70
Profit before tax , LLP & Depreciation 500,000.00 400,000.00
Less: Allowable LLP Expenses 10,000.00 50,000.00
Less: Depreciation 90,000.00 -
Taxable Profit 400,000.00 350,000.00
Income tax @ 30% 120,000.00 105,000.00

Note : 1
Loan Loss Provision is allowed maximum of 5% of Loan outstanding at the year-end subject to
LLP to be accounted as per NRB Directives.

Particulars 2070/71 2069/70


Loan Outstanding 1,200,000.00 1,000,000.00
5% of Outstanding 60,000.00 50,000.00
Loan Loss Expenses during the year 15,000.00 60,000.00
Allowable Loan Loss Expenses 10,000.00 50,000.00

Note : 2

Total depreciation 100,000.00


Less: Disallowable
Depreciation 10,000.00

Allowable Depreciation 90,000.00


Calculation of Allowable Depreciation

Depreciation on Allowable Expense Disallowable


ATM: (25000/25%)* 15% (ATM falls under Class D)
15,000 25,000 10,000
Rate of Depreciation is 15%
RTP-CAP II –-2014-December@ICAN Page 132 of 158
Car:
20,000 20,000
Car falls under Class C, Rate of depreciation is 20%
Building: (Rate of Building Constructed in leased land
shall be the rate calculated as divided by useful life of the 20,000 20,000
Assets i.e. Leased Land)

Answer 3

Computation of assessable income, taxable income and tax liability of Mr. Babu Kaji Shakya for the
Fiscal Year 2070/71:
Particulars Amount (NPR)
Salary (40,000 x 12) 480,000.00
Dashain Allowance 40,000.00
Education Allowance (2000 x 12) 24,000.00
Expenditure Allowance (2000 x 12) 24,000.00
Bonus 60,000.00
Vehicle Facility (480,000 x 0.5%) 2,400.00
Driver's Salary NotTaxable
Petrol Expenditure NotTaxable
Maintenance Expenditure NotTaxable
Housing Facility (480,000 x 2%) 9,600.00
Telephone Expenditure (2000 x 12 x 50%) 12,000.00
Security Guard Facility [(6000-3000) x 12] 36,000.00
Loan Facility (8%-5%) x 1,000,00 30,000.00
Salary against house leave 40,000.00
Contribution to Provident Fund (PF) 48,000.00
Income from employment from Financial Institution 806,000.00
Salary from Teaching in Shankardev Campus (6000 x 12) 72,000.00
Assessable Income from employment 878,000.00
Deductible Amount
Deductible Amount against contribution to PF (292,667.00)
Actual Contribution
Employer's Contribution 48,000.00
Employee's Contribution 48,000.00
Contribution/Deposit to Investment Fund by employer 240,000.00
Total Contribution Amount 336,000.00
1/3rd of Assessable Income 292,667.00
Maximum Limit 300,000.00
Adjusted Taxable Income 585,333.00
Donation Expenditure (29,267.00)
5% of Adjusted Taxable Income 29,267.00
Actual Donation Expenditure 60,000.00
Maximum Limit 100,000.00
Taxable Income 556,066.00
Deduction against Insurance premium (20,000.00)
Premium paid for himself and for his wife 33,000.00
Maximum Limit 20,000.00
Taxable Income (For Tax Calculation) 536,066.00
Computation of Tax Liability
RTP-CAP II –-2014-December@ICAN Page 133 of 158
Up to NPR 250,000 @ 1% (For Couple) 2,500.00
NPR 100,000 @, 15% 15,000.00
Balance @ 25% (186,066) 46,517.00
Total 64,017.00
Less: Medical Expenditure (750.00)
Total Tax Liability 63,267.00

Notes:

1. Mr. Babu Kaji Shakya has expensed out NPR 20,000 in medical treatment. 15% of NPR 20,000 comes
to NPR 3,000. Out of this NPR 3,000, NPR 750 is deductible from his tax liability as medical claim and
balance NPR 2,250 can be claimed in subsequent fiscal year.

2. Since, Mr. Babu Kaji Shakya claimed donation expenditure, contributed to approved fund had two
employers at the same time he has to file the return of the income.

3. As per Income Tax Directive, 2066, in case the employer also provides driver and fuel facility to the
employee along with the vehicle facility, the salary of the driver and the cost of the fuel is also not
required to add in the taxable salary of the employee.

4. As per section 1 (2) (s) of schedule 1 of the act, Income Tax at the rate 1% is to be deducted from the
salary income up to Rs. 250,000 for couple. The 1% salary tax is also a part of the Income Tax, but, or is
named as social security tax and as per circular this amount is to be deposited in the separate account.

5. Deposit by employer by debiting his account is not part of employment income.

Answer 4

Section 70 of income tax act, 2058 prescribes the following provision regarding taxation on the income
of non-resident person in Nepal from transport or transmission business. The gross receipts from the
following activities of a non-resident person are treated as taxable income of the person from the
activities.

A person engaged in Nepal in any road, air, water, or Chartered transport services, other than
transmission in Nepal, for any of the following services;

i) The carriage of passenger who embark from within a territory of Nepal.

ii) The services of mail, livestock, or other moveable tangible assets that are embarked from the territory
of Nepal. Expenditure incurred in conducting such activities are neither allowed to be deducted from the
taxable income from the above sources nor allowed to be deducted from any other source of the income
of the person.

RTP-CAP II –-2014-December@ICAN Page 134 of 158


The Tax rate applicable shall be as prescribed under section 2(7) of annexure 1 of income tax act (which
is 5% of gross receipts during the year).

No tax credits shall be allowed to the person to reduce the tax payable by the person under this section.

1. Accordingly, following treatment shall be done for the purpose of section 70 in case of Rolls
Airways. 1. For the purpose of Section 70, the income of Roll Airways for the year as mentioned in
point no. 1 & 2 of question shall be Rs. 15,000,000. Tax shall be 5% of Rs. 15,000,000 = Rs 750,000
as per section 2(7) of annexure 1 of Income Tax Act.

2. The amount mentioned in point no. 3 & 4 in the question shall not be taxed as per Income Tax Act,
2058 being sources of payment not in Nepal.

3. The amount mentioned in point no. 5 and 6 in the question are not the incomes to be included for the
purpose of section 70. These incomes shall be taxed like the entity having general business income of
Nepal. The expenditures incurred in connection with generation of income as mentioned in point no.
5 & 6 in the question are eligible for deduction.

4. The expenditure incurred in connection with generation of income as mentioned in point no. 5 & 6 in
the question are eligible for deduction for general income calculation.

5. The expenditures as mentioned in point no. 7, 8 & 9 in the question are not eligible for the deduction
for the purpose of computation of income as per section 70 even though they are incurred in
connection with the generation of income from ground handling services by its own.

6. The expenditures as mentioned in the point no. 11 & 12 in the question are not eligible for deduction
for the purpose of computation of income under section 70 as these are incurred in connection with
generation of income, the taxation which are not required to be paid under income tax act, 2058.

7. Half of the ground handling expenditure amounting to Rs. 1,500,000 and expenditure against
restaurant operation amounting Rs. 2,000,000 (total Rs. 3,500,000) are allowable expenditure for
deduction as these are incurred in connection with generation of income.

8. Corporate tax @ 25% shall be paid on the amount as mentioned in point no 5 & 6 in question
amounting to Rs. 5,000,000 less expenditure of 3,500,000 is Rs 1,500,000 and income tax (assuming
no other eligible expenditure to be claimed) is Rs 3.75 lakhs.

RTP-CAP II –-2014-December@ICAN Page 135 of 158


Answer 5

Calculation of Income Tax of Mr. Ramnarayan for FY 2070/71


Particulars Amount
Business Income 1,900,000
Income from Sale of Land 1,000,000
Total Taxable Income 2,900,000

Particulars Income Tax Rate Tax


First 250,000
Next 100,000 15% 15,000
Next 1,000,000 5% 50,000
Next 1,150,000 25% 287,500
Balance 400,000 35% 140,000
Total 2,900,000 492500

Answer 6

Calculation of Taxable Income & Income Tax of XYZ Pvt. Ltd.:

Profit as per P/L A/C = 7,000

Add: Under valuation of Closing Stock = 5,000

Add: Direct Expenses:

- Freight (Not related to income year section 13) = 100

- Personal Expenses (Section 21) = 200

- Rent Payment =0

Less: Disallowed bad debt expenses written off = 400 .

Taxable Income = 11,900

Now, Share of Taxable Income & Income Tax to Whiskey Trading & Rice Trading is as follows on
assumption of equal sale, purchase, opening stock, closing stock & all expenses.

RTP-CAP II –-2014-December@ICAN Page 136 of 158


Amount in ‘000
Particulars Income Tax Rate Tax
Whiskey Trading 5,950 30% 1,785
Rice Trading 5,950 25% 1,412.5
Total 11,900 3,272.50

Answer 7

Calculation of maximum allowable depreciation and Repair & Improvement expenses for special
industry;

Addition
Depreciable
Opening From From From Sales / Depreciation
Dep. Basis
Block WDV Shrawan Magh to Baisakh Disposal Total F = (A + B +
(F * Dep.
Rate
(A) to Poush Chaitra to Ashad (E) 2/3 * C +1/3 * Rate)
(B) (C) (D) D - E)
A 5% 5,000,000 5,000,000 5,000,000 5,000,000 20,000,000 15,000,000 750,000
B 25% 2,000,000 1,000,000 1,000,000 1,000,000 250,000
C 20% 10,000,000 5,000,000 5,000,000 5,000,000 1,000,000
D 15% 30,000,000 30,000,000 30,000,000 4,500,000
Total 47,000,000 5,000,000 5,000,000 5,000,000 6,000,000 56,000,000 51,000,000 6,500,000

Additional 1/3 depreciation = 2166667

Total Depreciation = 86666667

As per section of Sub-schedule (2) of Income Tax Act 2058, entity running special industry may claim
additional 1/3 rate of depreciation on block A, B, C and D of depreciable assets.

Repair & Improvement Expenses Deductible under Section 16 of Income Tax Act 2058
Deductible
7 % of Unabsorbed
Expenses Depreciation Repair &
Block Depreciatio Repair &
Amount Basis Improvement
n Basis Improvement
Expenses
A 1,400,000 15,000,000 1,050,000 1,050,000 350,000
B 700,000 1,000,000 70,000 70,000 630,000
C 350,000 5,000,000 350,000 350,000 -
D 3,000,000 30,000,000 2,100,000 2,100,000 900,000
Total 5,450,000 51,000,000 3,570,000 3,570,000 1,880,000

Answer 8
i. As per Section 54(3), dividend distributed from company is final withholding tax. Hence Rs 500,000
is not t be included in taxable income.
Calculation of taxable income & income tax
Profit = 20 lakhs
RTP-CAP II –-2014-December@ICAN Page 137 of 158
Less: Expenses = 5 lakhs
Taxable Profit = 15 lakhs
Tax Rate 25%
Income Tax = 3.75 lakhs
Less: Capital Gain Tax = 2 lakhs
Tax Liability = 1.75 lakhs
ii. Income from rope way is taxable 20%.
Hence, 20% of 10 lakhs = 2 lakhs

iii. Income Tax = 30% of Taxable Income


= 30% x 1000 lakhs

= 300 lakhs

 Income tax rate for income of bank & financial institutions is 30%.
 There is no rebate for banking business, whether it is listed or not.
iv. As per Section 11, listed hydropower company can avail 10% rebate on applicable tax rate. Hence,
effective tax rate is 18%. Again subsidy shall be also treated income from power generation. It is
because subsidy shall be given to the company which compiles the requirement of hydropower
generation.
Hence, Income Tax = Taxable Income x Effective Tax Rate

= (1000 + 500) x 18%

= 270 lakhs

v. Calculation of Taxable Income & Income Tax


Income = 4,000,000.00

Less: Retirement Contribution to ARF = 200,000.00

Less: Life Insurance Premium = 20,000.00

Taxable Income = 3,780,000.00

Calculation of Tax Liability


Slab Income Tax Rate Amount
First 200,000.00 0% -
Next 100,000.00 15% 15,000.00
Next 2,200,000.00 25% 550,000.00
Balance 1,280,000.00 35% 448,000.00
Total Income Tax 1,013,000.00

Since, wife of Mr. Ram is non-resident they can‟t opt couple. Hence his tax is calculated as per
individual.

Answer 9
9. a)
i. Permanent Establishment as per section 2(s)(b): „Permanent Establishment‟ (PE) means an
establishment where a person wholly or partly carries on a business. The establishment refers to

RTP-CAP II –-2014-December@ICAN Page 138 of 158


the head office, factory, premises, site office, branch office etc. In additional to that, these
undernoted establishment are also defined as PE :
a) An establishment where a person wholly or partially carries on a business through an agent,
when the agent is not a general agent of independent status.
b) An establishment where a person has, is using, or is installing main equipment or machineries
(factory premises).
c) One or more establishment within a country where a person furnishes, whether through
employees or otherwise, technical, professional, or consultancy service for a period or periods
aggregating more than 90 days within any 12 months.
d) An establishment where a person is engaged in a construction, assembly, or installation
project for 90 days or more, and a place where a person conducting supervisory activities in
relation to such a project.

ii. Non-business Chargeable Assets:


As per paragraph (b) of Section 2 of Income Tax Act 2058; NBCA means securities or an interest
in an entity as well as land and building but excludes the following assets:

a) Business Assets, depreciable assets or trading stock


b) A private resident of an individual that has been
i) Owned continuously for ten years or more, and
ii) Lived in by the individual continuously or intermittently for a total of ten years or
more.
[Clarification: For the purpose of this paragraph private residence means private building and
area of land covered by building or one ropani land, whichever is lower.]
c) Interest of any beneficiary in retirement fund.
d) A land, land and building and private residence of an individual that is disposed of for less
than 30 lakhs.
e) Assets of an individual that is disposed of by way of any type of transfer other than sales and
purchase made within three generations.

iii. As per paragraph (w) of Section 2 of Income Tax Act, exempt organization means the following
entities:
a) Following entities registered with the Department as an exempt organization:
i) A social, religious, educational, or a charitable organization of public character
established without having a profit motive,
ii) An amateur sporting association formed for the purpose of promoting social or sporting
facilities not involving the acquisition of gain.
b) A political party registered with the Election Commission,
c) A village development committee, municipality or district development committee, provided
that, any entity, giving benefit to any person from the assets of, and amounts derived by the
entity except in pursuit of the entity‟s function as per its objectives or as payment for assets or
service rendered to the entity by the person, is not exempt from tax.
iv. As per paragraph (8) of Section 2, company means a company established under the company
laws for the time being in force and the following institutions shall also be treated as company for
tax purpose:
RTP-CAP II –-2014-December@ICAN Page 139 of 158
a) Corporate body established under the laws for the time being in force;
b) Any unincorporated association, committee, institution, society, or group of persons other
than a partnership or a proprietorship firm (whether or not registered) or a trust;
c) A partnership firm (whether or not registered under the laws for the time being in force) that
has 20 or more partners, a retirement fund, a co-operative, a unit trust, or a joint venture;
d) Foreign company; and
e) Any foreign institution prescribed by the Director-General.
v. As per paragraph (s d) of section 2, service fee means any fee paid to a person based on market
values, for services rendered by the person and includes a commission or a meeting allowance,
management fee, or technical service fee.

9. b)
i) As per section 88 (1)(4), service fee payment to entity engaged in VAT exempt transaction is
1.5% withholding tax deductible, hence 1.5% of 98 lakhs, i.e. 1.47 lakhs to be deducted.
ii) As per section 88 (1)(6), dividend paid by a mutual fund to natural person is taxed @ 5%.
However, as per section 54(3), dividend distributed by any company out of dividend income
which is already dividend tax deducted, is dividend tax exempt payment. Again as per section
88(4)(ª), dividend payment to mutual fund is dividend tax exempt i.e. dividend tax not deducted.
Hence, it is subject to dividend tax deduction @ 5% i.e., 50,000 is to be deducted.
iii) As per section 88(1), commission is withholding tax deductible @ 15%, hence withholding tax
shall be 15% on commission (excluding VAT) is Rs. 1.5 lakhs.
iv) As per section 89 (3)(v), Insurance premium paid to non-resident is 1.5% withholding tax
deductible. Hence, Rs. 15,000 is to be deducted as withholding tax.
v) As per section 88 (4)(u), interest paid to mutual fund is not withholding tax deductible. Here
receipt of amount excess of Loan is interest as per section (2) i.e. Rs 10 lakhs (Rs 110 - 100) is
not withholding tax deductible.
vi) As per section 88 (4), question paper fee is not withholding tax deductible payment. Here, it is
payment against RTP, so withholding tax @ 15% is to be deducted i.e. Rs. 1500.
vii) As per section 88 (1), service paid is 15% withholding tax deductible. Hence, it is Rs. 15,000.
viii) As per section 88 (1)(2), commission paid by resident employment company to non-resident
company is to non-resident is 5% withholding tax deductible, hence, it is Rs. 25,000.
ix) As per section 54, dividend paid by partnership firm is 5% withholding tax deductible. Hence,
withholding tax is Rs. 100,000.
x) As per section 88 (1)(5), rent is 10% withholding tax deductible payment, hence, Rs. 15,000 is to
be deducted.

Answer 10
i) Advance Rulings Sec.76

a) The department may, upon application in writing by a person, issue to the person by notice in
writing a personal ruling setting out the Department‟s position regarding the application of this
Act to the person with respect to an agreement proposed or entered into by the person.
b) Notwithstanding subsection (a), the department may not issue a ruling under subsection (a) with
respect to an issue involving the application of this Act that is presently before a court or that
has been decided by a court.

RTP-CAP II –-2014-December@ICAN Page 140 of 158


c) Where the person makes the following prior to the issue of a ruling under subsection (a), the
ruling binding on the Department with respect to the application of this Act (as in force at the
time of the ruling) to the person with respect to the arrangement.
i. A full and true disclosure to the Department of all aspects of the arrangement relevant to
the ruling; and
ii. The arrangement proceeds in all material respects as described in the person‟s application
for the ruling.
d) Where a personal ruling issued under subsection (a) and a public circular issued under section
(75) contradict each other, priority shall be given to the terms of the personal ruling in respect
of the person to whom the ruling is issued.
e) Before issuing ruling, Department may permit applicant or his representation to furnish any
deficiency of documents.
As per Rule 22;

i. Applicant should file application in prescribed format.


ii. Department has to give decision within 45 days of application.
iii. If department didn‟t issue personal ruling within 45 days, applicant may file for
administrative review or appeal to revenue tribunal.
ii) Public Circular

As per section 75 of Income Tax Act, 2058;

a) To achieve consistency in the implementation of this act and to make the tax administration
simple and provide guidance to the persons affected by this act, including officers of the
Department, the Department may issue in writing public circulars setting out the Department‟s
interpretation of this Act.
b) The Department shall make public circulars issued under subsection (a) available to the public at
offices of the Department and at such other locations or by such other medium as the Department
may determine.
c) A public circular issued under subsection (a) shall be binding on the department.

iii) Jeopardy Assessment


If tax within a income year or after income year but before statutory time limit of filing of return is
Jeopardy assessment. It is of two types:
- Jeopardy assessment as self-assessment [Section 100(1)]
- Jeopardy assessment by tax authorities. [Section 100(2)]

Jeopardy assessment by the IRO is possible only under anyone of these conditions:
a. The person becomes bankrupt, is wound-up, or goes into liquidation.
b. The person is about to leave Nepal indefinitely.
c. The person is about to leave the business conducting in Nepal.
d. The IRD otherwise considers it appropriate.

Under any one of the above conditions the IRO may serve a notice to the taxpayer to submit a tax return
for the specified period of the year within specified days. In the case of a taxpayer who submits the
return as per the notification or does not submit it, in either case, the income tax assessment is supposed
to be made as per the provisions of Sec. 99(2). But Sec.100(2) has given an authority to respective IRO
to make a jeopardy assessment in the above case on the basis of the best judgement adopted by the IRO.

RTP-CAP II –-2014-December@ICAN Page 141 of 158


The period taken by the IRO for such a jeopardy assessment may be a part of the year or the whole year.
In such a case, the notice is meant for an assessment of the whole year, and the taxpayer has to file the
return within the time specified in the notice but in no way can wait for the period as specified in Section
96.
The respective IRO can make a jeopardy assessment only if it has a reasonable belief that the figures
produced or deemed to be produced by the taxpayer do not exhibit the real position of the tax liability of
the taxpayer for the period.
According to Section 100(2), the following pieces of information are considered for the jeopardy
assessment:
a) Assessable income of the taxpayer from business, employment or investment, i.e. from all the
sources.
b) Taxable income of the taxpayer during the year and the total amount of tax due to the taxpayer, and
c) In the case of a taxpayer, which is a foreign permanent establishment, the income remitted to a
foreign country during the period and tax payable on such a remittance.

Before issuing an order for jeopardy assessment, the IRO has to serve a notice to the taxpayer stating the
reason of disagreement over the figures given in the return field or the figures available to the IRO. A
period of seven days should be given to the taxpayer to explain and produce evidence against the IRO‟s
contention.
In case a jeopardy assessment is made for a whole year, the taxpayer is not required to submit a tax
return under Sec. 96(1). But if it is for part of the year, the taxpayer has to file a return under sec.96(1)
and the treatment of tax paid as per the jeopardy assessment shall be as advance payment of tax and
could be adjusted against the tax payable as calculated as per the self assessment for the year.
When the IRO has made a jeopardy assessment, it has to issue an order to the taxpayer stating the
following assessment:
a) The total tax payable by the taxpayer for the period of assessment and the tax due to him;
b) The method of calculation of the tax liability.
c) The reason of the Jeopardy assessment by the IRO.
d) The period within which the tax due is payable; and
e) Where, when and how to appeal against the order if the taxpayer is not satisfied with the jeopardy
assessment.

In case of competent authority of contracting state having treaty regarding „Avoidance of Fiscal Evasion‟
seek any tax arrears on that country to be collected from the person in Nepal, then taxation authority can
collect the tax to the extent of valid and bona fide request of competent authority. The assessment based
on these request are assessment classified as jeopardy assessment of tax collecting for another country.

Answer 11
a) As per section 59(1) (ka), a banking business is allowed a deduction of a loss of loan provided by it
according to the regulation of Nepal Rastra Bank subject to a maximum ceiling of 5% of the
outstanding loan as on the last day of the income year.
Actual expense for a loan loss, if incurred, should be deducted from the provision outstanding on
that date. In case it is deducted as an expense, it is not allowed for tax purpose.

In case the amount of loan loss provision is capitalized or utilized for distribution, or for payment of
dividend in any income year, up to that amount it should be included in the taxable income of the
income year.

RTP-CAP II –-2014-December@ICAN Page 142 of 158


b) As per section 59,1(kha) ( amended by Finance Act 2071) Co-operative is allowed a deduction of a
loss of loan maximum ceiling of 5% of outstanding loan as on the last day of the income year for
fiscal year 2071/72.
c) According to Sec.92, some of withholding taxed payments are final taxed in the case of recipient.
Person getting final withholding taxed income need not be included in the taxable income.
Following payments are final withholding taxed payments:

Income or Payment Condition applicable Rate for 2071/72


In case a resident bank, finance company , a listed
company, or any entity that has issued debentures,
Interest having source in
are paying interest for deposits, debenture, bond 5%
Nepal
etc. to a natural person not in the course of
conducting a business.
In case a resident bank, finance company, a listed
Interest having source in company, or any entity that has issued debentures,
15%
Nepal are paying interest for deposits, debentures, bonds
etc to a tax exempted person under section 2(s)
Rent for land and /or building with or without
Rent having source in including the attachment or equipments installed in
10%
Nepal that land or building payable to a natural person not
in the course of conducting a business.
Meeting or part time In meeting fee payable to a natural person for
15%
teaching meeting fee and periodical teaching fee.
Retirement payments made by GON on approved
Retirement payments retirement fund, on the amount calculated as per 5%
section 65(1)(b).
Retirement payments made by an un-approved
Retirement payments retirement fund, on the amount of the gain to the 5%
employee.
Dividend In case of distributed by company. 5%
Amount received from a life insurance company for
Gain on investment
insurance on the life of individual, on the amount of 5%
insurance
gain to individual
Amount received from any type of windfall gain
Windfall gain 25%
except exempted by GON
Return from Mutual Distribution of Return to Individual by Mutual
5%
Fund Fund

d) According to Section 114, an application against the following decisions may be moved to IRD for an
administrative review;
a) Advance ruling issued under Section 76 by IRD;
b) Re-assessment of estimation of advance payment by a tax-payer made by a Tax Officer under
Section 95(7)
c) Decision by Tax Officer to require a tax-payer to file a return of tax under Section 96(5) or 97
d) Decision of a Tax Officer with regard to an extension of time for filing returns under Sec. 98;
e) Jeopardy assessment under Section 100, an amended assessment under Section 101, an
assessment of expenses incurred on auction sales under Section 105(5), or penalties imposed
under Section 117 to 121;
f) Notification by the IRO of an amount to be set aside by a receiver under Section 108(2);

RTP-CAP II –-2014-December@ICAN Page 143 of 158


g) Order by a tax office to a debtor of the taxpayer to pay the amount due to the tax office instead of
to the taxpayer under Section 109(1);
h) Order by a Tax Officer to a person to pay tax on behalf of a non-resident person under Section
110(1);
i) Decision of IRO on an application by a tax payer for a refund of tax under Section 113 (5) ; and
j) Decision of IRD on an application by a taxpayer for extension of time within which to file an
objection under Section 115(3).

e) As per Section 95, Payment of Income Tax by Installments


A person who derives or expects to derive any assessable income during an income-year from a business
or investment shall require to pay tax for the year by three installments as follows:
Date Payable Amount Payable
By the end of Poush Due tax amount out of 40% of the estimated tax
By the end of Chaitra Due tax amount out of 70% of the estimated tax.
By the end of Ashad Due tax amount out of 100% of the estimated tax.

Answer 12
a) As per Section 21 (2), a person whose annual turnover for an income year exceeds Rs 2,000,000 is not
allowed a deduction for a cash payment in excess of Rs. 50,000 incurred other than in the following
conditions by the person during the year-
a. Payment made to Government of Nepal, a constitutional body, a corporation owned by His
Government of Nepal, or a bank or financial institution;
b. Payment made to a farmer or a producer producing primary agricultural products even in the case
where the product is primarily processed by the famer himself;
c. Payment is a retirement contribution or retirement payment;
d. Payment is made in an area where banking services are not available;
e. Payment is and must necessarily be made in cash or on a day when banking services are closed; or
f. Payment is made into a bank account of the payee.
[As per clarification of section (21) “Cash Payment” means letter of credit, account payee cheque,
bank draft, money order, telegraphic transfer from bank or financial institutions, payment being
deposited by means of money transfer and payment by means of other arrangement between bank or
financial institutions.]

b) As per Section 57 of Income Tax Act, change in control means change of 50 percent or more in the
underlying ownership of an entity as compared with its ownership 3 years previously. For the purpose of
calculating the change of 50 percent or more in the ownership of the entity, the following ownership
shall be included:-
a. The ownership occupied by the shareholder holding one percent or more of the total ownership,
and
b. The ownership occupied by any of the shareholders holding less than one percent of the total
ownership but associated with the shareholder holding one percent or more ownership.

c) Belated Tax Return: In case a person delayed in submission of final tax return under Section 96, it
has to pay the following fees under Sec. 117 (1)(b):

RTP-CAP II –-2014-December@ICAN Page 144 of 158


i. In case the person falls under the category as specified by section 4(4) (tax payable by fix
amount), the fine is Rs 100 per month of delay. A part of a month shall be treated as delay of one
month for this section.
ii. For other taxpayers the fine is 0.1% of the assessable income without deducting any amount or
Rs. 100 per month of delay whichever is higher.

For other taxpayers the fine is 0.1% os the assessable income without deducting any amount or
Rs. 100 per month of delay whichever is higher.

For Failure to maintain Accounts and Records: According to Sec 117(2), if a person has
obligation to maintain accounts and records as per the Act but fails to maintain the same as
instructed in Section 81, a penalty of 0.1% per annum on assessable income without any
deduction or Rs. 1,000 per annum whichever is higher is charged for the whole year in which the
accounts and records are not kept.

d) As per Schedule- 2 of Income Tax Act 2058, different class of depreciable assets, assets included and
rate of depreciation are as follows:

Class Assets Included Rate of Depreciation


Buildings, structures and similar permanent
A 5%
construction
Computers, data handling equipment, fixtures, office
B 25%
furniture and office equipment
C Auto Mobiles, Buses and Mini Buses 20%
Construction and earth moving equipment and any
depreciable assets not included in other class (i.e. A, B
and C) including those referred in Section 17(3) i.e.
D 15%
pollution control cost capitalized & 18(3) research and
development cost capitalized and expenses incurred on
natural resource prospecting.
Rate in percent calculated as
divided by the useful life of the
Intangible asset other than depreciable assets included assets in the pool at the time the
E
in class D assets is most- recently acquired
by the person and rounded down
to the nearest half year.

Answer 13
a) As per Rule 13(1), if any person provides a vehicle to any beneficiary including employee or worker
for partly or fully personal use, the following amounts shall be quantified as income.

i) For employee or worker or person getting monthly remuneration, 0.5% of regular salary.
ii) For others except person mentioned (i) above, 1% of market value of the vehicle.

RTP-CAP II –-2014-December@ICAN Page 145 of 158


b) As per Rule 13(2), if any person provides a house facility to any beneficiaries including employee or
worker for personal use, the following amount shall be quantified as income:
i) For employee or worker or person getting monthly remuneration, 2% of regular salary.
ii) For Person other than mention in (i), if person providing rented house 25% of rent paid.
iii) For person other than mention in (i), if house is not rent payable, 25% of prevailing house rent.

c) As per section (9)(3) of Income Tax Act, there are some payments, those seems as being relating to
employment but these payments are not included in computing income from employment are as follows:
i. Any amount received by an employee for which exemption is given under Section 10 of the Act.
In this regards any remuneration received by resident natural person being paid through public
fund of any foreign government is exempted. In case of security personnel getting pension from
public fund of foreign government, these pension is also exempted.

ii. In case of any non- Nepalese resident person ( or family members) gets remuneration from
foreign public fund, those income are also exempted.

iii. Any amount received which is subject to final withholding of tax.

iv. Work-time meals or refreshment provided by the employer in equal terms for all the employee at
working place or uniform applicable to working place only.

v. Any reimbursement of expenses incurred by the employee:


 That serves the purpose of the business of the employer; or
 That would otherwise be deductible in calculating the individual‟s income from the business
or investment.
 Reimbursement of outstation cost- travelling or daily allowance
vi. Any prescribed small amounts, which are too small and thus unreasonable or administratively
impracticable to make accounting for them. The amount prescribed by the Rule is Rs. 500 at a
time. The expenses prescribed by the Rule include tea expenses, stationery expenses, prizes ,
gifts, emergency medical facility, or other such payments as specified by IRD.

d) As per section 7 of Income Tax Act, income from a Business includes the following.
i. A person‟s income from a business for an income- year is the person‟s profits and gains from
conducting the business for the year.

ii. There shall be included in calculating a person‟s profits and gains from conducting a business for
an income- year the following amounts derived by the person during the year:

a) Service fees;
b) Amounts derived from the disposal of trading stock;
c) Net gains from the disposal of the person‟s business assets or liabilities of the business as
calculated under Chapter 8;
d) Amounts treated as derived under paragraph 4 (2) (a) of Schedule – 2 in respect of excess
depreciation on the disposal of the person‟s depreciable assets of the business;
e) Gifts received by the person in respect of the business;
f) Amounts derived as consideration for accepting a restriction on the capacity to conduct
the business;

RTP-CAP II –-2014-December@ICAN Page 146 of 158


g) Amounts derived that are effectively connected with the business and that would
otherwise be included in calculating the person‟s income from an investment; and
h) Other amounts required to be included under Chapters 6 or 7 or sections 56 or 60.

iii. Notwithstanding subsection (ii), amounts exempt under sections 10, 54, and 69 and final
withholding payments are excluded in calculating a person‟s profits and gains from conducting a
business.

e) As per Rule 21, natural person being beneficiary of approved retirement fund contributing retirement
fund can deduct the lower of the following in any income year from taxable income:

i) Rs 300,000 or
ii) 1/3rd of Assessable Income or
iii) Actual Contribution

Answer 14

a) As per Rule 5, tax exempted organization should compulsorily submit audited financial statements of
the entity within three months from the end of Fiscal Year. Entity not submitting such statement shall
not get tax exemption till non submission.
Again, as per Sec. 117, fee of 0.1% p.a. on amount shown in income shall be charged for not
submitting such returns.

b) As per sec. 110 ( ka), a tax Officer may permit to any person to pay due tax in installment if the
person request in writing to pay due tax in installment before filing suit as per section 111.

c) Transfer of Asset to Spouse or Former Spouse as per section 43 of Income Tax Act 2058 is as
follows:
Where as part of a divorce settlement or bona fide separation agreement, an individual disposes of an
asset by way of transfer to a spouse or former spouse and an election for this section to apply is made
by the spouse in writing, then the following provisions shall apply:-

i. The person shall be as deriving an amount in respect of the disposal equal to the net outgoings for
the asset immediately before the disposal; and
ii. The spouse or former spouse shall be as incurring in acquiring the asset by way of transfer costs
of an amount equal to as mentioned in paragraph (i).

d) Taxpayer‟s Rights as per Sec. 74 of Income Tax Act, 2058.


i. Tax Payer shall abide by the duties in accordance with this Act.
ii. A taxpayer with respect of paying tax under this Act shall have the following rights:-
a. Right to get respectful behavior;
RTP-CAP II –-2014-December@ICAN Page 147 of 158
b. Right to receive tax related information as per the prevailing laws;
c. Right to get the opportunity of submitting proof in own favour in respect of tax matters;
d. Right to appoint lawyers or auditors for defense; and
e. Right to secrecy in respect of tax matters and keep it inviolable.

[Clarification:- For the purpose of this section , taxpayers means a person whom the tax is
imposed on and realized from as referred to in section 3.]
e) As per Section 71 “clarification”, average tax rate of Nepal means “Average rate of Nepal‟s income
tax” means the rate that appears after dividing the tax amount by taxable income that is applicable to
the income tax payable by a person for an income year under Section 3(a) before any foreign tax
credit adjustment is made and multiplying the resulting amount by hundred.

Answer 15
a) As per section 50,
i) A resident natural person and a resident spouse of the person may, by notice in writing elect to be
treated as a single individual for a particular income year.
ii) Each spouse of a couple making an election as above with respect to an income-year is jointly
and severally liable with the other spouse for any tax payable by the couple for the year.
iii) What so ever mentioned in above (i) and (ii) resident widow or widower responsible to take care
of dependents shall be treated as couple.

b) As per Section 16, Repair and Improvement Costs provision is as follows;

i) For the purpose of calculating a person‟s income for an income – year from any business or
investment, there shall be deducted all costs to the extent incurred during the year in respect of
the repair or improvement of depreciable assets owned and used by the person during the year in
the production of the person‟s income from the business or investment.
ii) Notwithstanding mentioned in (i), the deduction allowed with respect to all depreciable assets in a
particular pool of depreciable assets of the person shall not exceed seven percent of depreciation
basis of the pool at the end of the income year. However, the limitation shall not be applicable on
overhauling of aircraft as required by Standards of Civil Aviation Authority of Nepal.
iii) Any excess cost of repair and improvement, or a part thereof, for which a deduction is not
allowed as a result of the limitation of (ii), can be added to the depreciation basis prevailing in the
beginning of the subsequent income year, of the pool to which it relates.

c) As per Section 10 of Income Tax Act, the following amounts are tax exempted incomes:
i) Amounts derived by a person entitled to privileges under a bilateral or a multilateral treaty
concluded between Nepal Government and a foreign country or an international organization.
ii) Amounts derived by an individual from employment in the public service of the government
of a foreign country.
Provided that-

a) The individual is a resident person solely by reason of performing the employment or is a


non-resident person; and
b) The amounts are payable from the public funds of the country.
RTP-CAP II –-2014-December@ICAN Page 148 of 158
iii) Amounts derived from public fund of the foreign country by an individual who is not a citizen
of Nepal as referred to in paragraph (ii) or by a member of the immediate family of the
individual.
iv) Amounts derived by an individual who is not a citizen of Nepal from employment by Nepal
Government on terms of tax exemption.
v) Allowances paid by Nepal Government to widows, elder citizens, or physically disabled
individuals.
vi) Amounts derived by way of gift, bequest, inheritance, or scholarship, except as required to be
included in calculating income under section 7, 8 or 9 and
vii) Amounts derived by an exempt organization by way of-
a) Gift; or
b) Other contributions that are directly related to the organization‟s function referred to in
paragraph(s) (1) of the definition of exempt organization in section 2, whether or not the
contribution is made in return for consideration provided by the organization, or
c) Income earned by Nepal Rastra Bank as per its objective, or
d) Income earned by Securities Board of Nepal as per its objective.
viii) Pension received by a Nepali citizen retired from the army or police service of a foreign
country provided the amounts are payable from the public fund of that country.
ix) Any income of Nepal Government.

d)
If a resident natural person becomes ill, his treatment cost is qualify for medical tax credit under Sec. 51.
Eligible Medical Cost (EMC) is cost of treatments including fee paid to doctor, lab cost, dispensary cost
and other associated costs. In EMC, cosmetic medical cost is not included. If a person has medical
insurance, premium paid for it is deemed as EMC.
Medical Tax Credit Limit (MTCL) is as follows:
= (EMC + Medical Insurance Premium – Insurance Compensation) * 15%
Maximum limit of medical tax credit is Rs. 750 for a year. If a person has MTCL is more than Rs. 750 or
have tax payable is less than Rs. 750, any unrelieved MTCL is carried forward to next years.
As per Rule 17(1), the following shall be treated as approved medical expenses:
 Health Insurance Premium
 Amount as per bill including medical expenses of approved hospital, nursing home, health center
or doctor
As per Rule 17(2), the following shall not be included in approved medical expenses:
 Expenses of Cosmetic Surgery
 Insurance claimed medical expenses

e) Basis of Tax Accounting


As similar in accounting, basis of accounting is also cash basis of accounting and accrual basis of
accounting. Both cash basis and accrual basis are not conceptually same for both tax and accountings.
Statutory cash basis for tax: Income from employment and income from investments in case of a
natural person has to be accounted in cash basis of accounting.
Statutory accrual basis for tax: Company should keep its tax accounts on accrual basis of accounting.
Banking business, as licensed from Nepal Rastra Bank can keep its accounts based on directives. Again,
co-peratives can maintain its interest income in cash basis.
Optional basis for tax: Income from business of a natural person and income of entity other than a
company (partnership and trust) may opt either basis for taxation.

RTP-CAP II –-2014-December@ICAN Page 149 of 158


Different basis for accounting and for tax: A person adopting accrual basis for financial accounts can
opt (sometimes requires too) cash basis and vice versa for tax purpose.

RTP-CAP II –-2014-December@ICAN Page 150 of 158


VALUE ADDED TAX
Answer 16;
a)
i. Tasty Cola Pvt. Ltd. is selling beverages only not bottles. When beverage is consumed by
consumer, the empty bottle is returned to the producer. Thus, the supply of empty bottles on
deposit does not constitute transaction as per VAT Act.
Section 2(g) of the VAT Act, defined the term supply which means act of selling, exchanging and
delivering any goods and services, or the act of granting permission thereto, or of contract thereof
for a consideration. In this case the supply of bottle is not for consideration and so it is not treated
as supply for transaction purpose.

It was held in Pepsi Cola Nepal Pvt. Ltd. vs VAT Department, the Revenue Tribunal has
interpreted that the bottles would be treated as sold in case of breakage or lost by the dealer and
when the producer charging for such breakage or shortage to the dealer. The tribunal has pointed
out Section 12(8) of the Act, according to which deposit amount could be treated as sales when it
is adjusted against consideration receivable for supply of goods or services.

That is why; the shortage of bottles at factory premises cannot be treated as sold in case it is
proved that the bottles are lying with the dealers.

ii. Here, at the time of full & final settlement, it is agreed that the value of shortage of bottle shall be
adjusted at the prevailing cost price to the company. Hence the value shall be adjusted with
“Bottle Deposits”. As per Section 12(8), it is to be treated as taxable value.
As per Section 12(1), when only the money is consideration, the taxable value shall be the price
the supplier charges for the recipient. Hence, VAT is levied on the shortage quantity only at the
rate of Rs 30 per bottle.

Since, the shortage of bottles from the dealer is 10,000 (50,000 - 40,000) bottles the taxable value
for the manufacturer is Rs 300,000 (10,000 x Rs 30). Hence, Rs 39,000 (300,000 x 13%) is to be
collected/settled as VAT and Rs 300,000 as sale of bottle.

b) The different tax period as per Rule 26 are described as follows;


i. Monthly Tax Period
A registered person has to adopt a month as per Nepali calendar as tax period except for the
following cases.

ii. Bi-Monthly Tax Period


Registered person having taxable transaction during previous 12 months more than Rs 2
million but not more than Rs 10 million and hotel & tourism sector, if desires so, tax payers‟
taxpayer tax period is 2 months. The tax period shall be, Shrawan & Bhadra, Ashwin &
Kartik, Marga & Poush, Magh & Falgun, Chaitra & Baisakh and Jestha & Ashad.

iii. Trimester Tax Period

RTP-CAP II –-2014-December@ICAN Page 151 of 158


Voluntarily registered person having taxable transaction during previous 12 months less than
Rs 2 million have four months tax period. The tax period shall be, Shrawan to Kartik, Marga
to Falgun and Chaitra to Ashad.

iv. First Tax Period Tax Period


First tax period of a registered person shall be period of day of registration to end of concern
tax period.

v. Different Tax Period Tax Period


Registered person maintaining computerized accounts, if it could not report in Nepalese
month system, may apply Tax Officer to adopt different tax period. In case the Tax Officer
satisfies on the application, may allow to adopt different tax period. In this case, tenure of
period shall be same but only be change in starting and ending day of tax period.

Answer 17;
a) As per section 8(3) of the Act, any commercial building or apartment or shopping complex or
similar structure as prescribed by the department of more than 50 lacs must be constructed thorough a
VAT registered party. If such structures are constructed from unregistered party then the tax shall be
collected from the owner of such structure considering as constructed by a registered party.
b) VAT shall be exempted on Equipment, Machinery Tools & their Spare parts, Penstock pipe or iron
plate used to make Penstock Pipe (which are not produced in Nepal) on the recommendation of
Alternative Energy Promotion Centre or Department of Electricity Development as the case may be.
Zero rates shall be applicable on the supply of equipment, spares & parts thereof, penstock pipe or
iron plate used for making penstock pipe by the Domestic Industries manufacturing and directly
supplying equipment to the Hydropower Projects on the recommendation of Alternative Energy
Promotion Center or Department of Electricity Development and as process prescribed by IRD.

c) 50% of VAT collected on sale of its production by domestic dairy industry shall be refunded as
prescribed by IRD. And, 50% of VAT collected on sale of its production to VAT registered tax payer
by a tea producing and processing industry in local shall be refunded as prescribed by IRD.
d) Provision of VAT exemption on
i. Cultural program
 Not Exempted

ii. Entry fee to Zoo


 Exempted

iii. Entry fee to Botanical garden


 Not Exempted

iv. Supply of Manpower abroad


 Exempted

v. Writer getting fee of Rs. 50 lacs against writing of his autobiography from publisher
 Exempted

Answer 18;

RTP-CAP II –-2014-December@ICAN Page 152 of 158


a) As per section 12(4) of the VAT Act, the taxable value of any goods or service exchanged or bartered
shall be equal to the market value of the goods or services so exchanged or bartered. Hence, the
taxable value is Rs 35,000 to Ram & Sons for the item exchanged. It is because printer was not sold
on Rs 45,200 (including VAT) though its cost was Rs 125,000.

b) As per section 12(ka)(1) and section 12(ka)(2) the taxable value for sales of wood from private land
shall be equal to the royalty amount of wood from national forest or auction price of wood from
national forest, whichever is higher. Hence, the taxable value of wood for Mr. Ray is Rs 12,000,000.
c)
i. As per Rule 33(1), the tax of the goods already used shall be assessed only on the balance
between the selling price and cost price. And as per Rule 33(1), the cost price as mentioned in
sub-rule 33(1) shall include the cost with tax. The calculation of taxable value and VAT is as
follows;
Total Cost Sales Balance Taxable
Particulars Purchase VAT VAT
(A) (B) (B – A) Value
Sofa set 10,000 1,300 11,300 22,000 10,700 10,700 1,391
Table 10,000 1,300 11,300 11,000 (300) - -
Chair 2,000 - 2,000 3,000 1,000 1,000 130
Total 2,000 2,600 24,600 36,000 11,400 11,700 1,521

ii. As per section 16(3) of the Act, a registered person shall use, for the purpose of keeping accounts,
the purchase book and sales book certified by the concerned tax officer. And again as per section
29(g), on infringement of this Act and Rules thereunder Rupees ten thousand for each time.
Hence, penalty of Rs 10,000 for not getting the Purchase & Sales Register approved shall be
applicable.
d) As per Rule 43(1), a tax payer may make an application in the form of Schedule 16 to the Tax Officer
to have deduction of the VAT charged on the goods which were in stock at the time of his
registration. At the time of application, the tax payer needs to furnish tax invoices and other
evidences of the tax payment within 15 days of registration. If there are no such evidences, deduction
will not be allowed. Again as per Rule 43(4), if the claim as referred to above Rule is accepted by the
Tax Officer, the concerned taxpayer may deduct the amount pursuant to Sub-rule (4) of Rule 39.

Answer 19;
a) The calculation of VAT payable for the month of Shrawan is as follows;
Calculation of collected VAT on Sales;
Amount Service Charge Total
Sales VAT Amount
(A) (B) (A + B)
Room Sales 10,000,000 1,000,000 111,000,000 1,430,000
Food & Beverages Sale 2,500,000 250,000 2,750,000 357,500
Laundry 500,000 50,000 550,000 71,500
13,000,000 1,300,000 14,300,000 1,859,000

Calculation of VAT paid on Purchase;


Purchase Amount VAT VAT Claimable Remarks
Delivery Van 1,500,000 195,000 195,000 Rule 41(2) Clarification
Car 2,000,000 260,000 104,000 Only 40% credit available
RTP-CAP II –-2014-December@ICAN Page 153 of 158
Rule 41(2) (Kha)
Diesel for Van 200,000 26,000 26,000 Rule 41
Petrol of Car 250,000 32,500 - Not claimable [Rule 41(1)(ga)]
Wine 500,000 65,000 65,000 Rule 41(3)
Not claimable for Rs 100,000 of
Food items 700,000 91,000 78,000
staff meal (As per circular)
Other Expenses 100,000 13,000 13,000
Total 5,250,000 682,500 481,000

As per Section 8(3) and circular, VAT on construction of building


Particulars Cost Amount VAT VAT Payable Remarks
Bricks 500,000 - 65,000 Without VAT
Cement 1,500,000 97,500 97,500 50% in VAT
Iron 1,000,000 130,000 - VAT
Other Material 1,000,000 130,000 - VAT
Wage 1,000,000 - 130,000 Not in VAT
Consultancy to Engineers 500,000 - 65,000 Not in VAT
5,500,000 357,500 357,500

Calculation of Net VAT Payable


Particulars Amount
VAT Payable from Sales 1,859,000
VAT Payable from Construction of Building [Section 8(3)] 357,500
Less:
Input Tax Credit From Purchase 481,000
Net VAT Payable for Shrawan 2071 1,735,500

b) When there is change in value of goods or services supplied, Debit or Credit notes are to be issued.
As per Rule 20(1), if a registered person has to issue a credit or debit note because of change in the
price of the goods or services supplied by him, he has to write down credit or debit more in such note
clearly and set out therein the following matters as well:
(a) Serial number,
(b) Date of issue,
(c) Name, address and registration number of supplier,
(d) Name, address of recipient, and registration number if he/she is a registered person,
(e) Name and date of tax invoice related with transaction,
(f) Details of goods or services and of credit and debit,
(g) Amount of credit or debit,
(h) Amount of tax of credit or debit.

c)
i. As per Section 10(2)(ka) a liquor distributer or shop must register in VAT. Hence, the liquor shop
must register in VAT irrespective of its transaction and area.

RTP-CAP II –-2014-December@ICAN Page 154 of 158


ii. As per Section 10(2)(ka) a person supplying software must register in VAT. Hence, person
supplying software must register in VAT.
iii. As per Section 10(2)(kha) an ice-cream industry established in Metropolitan, Sub-Metropolitan,
Municipality or area as directed by IRD must register in VAT. Hence, the ice-cream industry at
xyz Municipality must register in VAT.
iv. As per Section 10(2)(kha) a tailoring shop with shirting suiting fabrics established in
Metropolitan, Sub-Metropolitan, Municipality or area as directed by IRD must register in VAT.
Hence, the tailoring shop with shirting suiting fabrics on sale at Bagbazar, Kathmandu
Metropolitan City must register in VAT.
v. As per Section 10(2)(ka) a person providing trekking service must register in VAT. Hence,
company providing trekking service must register in VAT.

Answer 20;
a) As per Section 11(1)(ka), small vendors who have registered voluntarily cannot deregister before
completion of one year. The small vendors have to submit an application before the concerned Tax
Officer after completion of one year of registration of business, if they want to deregister.
Also, as per Section 11(1) of the Act, the tax officer may cancel the registration of a registered person
in any of the following cases:

(a) In the case of an incorporated body, if the incorporated body is closed down, sold or transferred
or if the incorporated body otherwise ceases to exist.
(b) In the case of an individual ownership, if the owner dies.
(c) In the case of a partnership firm, if it is dissolved or if a partner dies.
(d) If a registered person ceases to be engaged in taxable transactions.
(d)(1) In case the registered persn submits zero tax return continuously for one year or it has not
submitted tax return till the date.

(e) If a person is registered in error.

b) As per Section 11(3) of the Act, all the goods that remain available at the time of the cancellation of
registration (capital goods as well) on which an offset had already been taken, tax shall be assessed
and collected on market price. Hence, the VAT to be paid as deemed disposal for the given case is as
follows;

Mkt. Price
Items VAT
(Taxable Value)
Stock-A 11,000,000 1,430,000
Stock-B 12,000,000 1,560,000
Car 1,500,000 195,000
24,500,000 3,185,000
Less:
Opening VAT receivable 3,000,000
Net VAT amount payable 185,000

RTP-CAP II –-2014-December@ICAN Page 155 of 158


c) Sec 8(Ka) provides some facility to the importer whose imported material is re-export with value
addition and duty free bonded warehouses. In such case, importer may use bank guarantee facility for
the portion of VAT to be paid in import at custom.
Conditions for re-exporter
In the following conditions bank guarantee facility is availed:
1. Manufacturing industry exporting more than 60% of its sales during last 12 months or duty free
under bonded warehouse.
2. This facility is availed to importing raw materials only.
3. Exporter need to value add at least 10%.

Condition for bonded warehouse:

Bank guarantee facility is availed upon the following conditions:

1. There should be bonded warehouse for duty free shops.


2. Sale of duty free alcoholic goods and cigarettes should be limited to diplomatic person or entity
getting duty facility by Ministry of Foreign affairs.

The bank guarantee in both cases shall be released by the custom office as per prescribed by IRD.

d) As per section 15 of VAT Act,


(1) A person who is not registered shall not issue an invoice or other document showing the
collection of tax and shall not collect tax.
(2) If a person who is not registered collects tax, the tax so collected shall be assessed and collected
from him.
(3) Government of Nepal, local bodies, international agencies and commissions or public corporation
must collect tax if transaction of taxable goods or services is done.

Answer 21;
a)
i. As per Section 17(5)(kha), a taxpayer may offset the VAT amount paid against reverse charging
as per section (8)(2). Hence, contention of the tax officer is not as per provision of VAT Act.
ii. Tax credit is available without tax invoices under following cases;
a) Bills from Electronic Cash Register.
b) VAT paid voucher against wood from forest.
c) VAT paid voucher against reverse charging.
d) VAT paid voucher against import.
e) VAT collected by GON, Local Government or INGOs.

b) As per Section 16(4) and Rule 23(7), the registered person has to retain safely the records maintained
pursuant to this Rule for up to Six years. Hence, the clarification of the chairman in the given case is
not as per the Act.

c)
RTP-CAP II –-2014-December@ICAN Page 156 of 158
i. As per Section 24(4), any registered person whose export sales for a month are more than 40
percent of his total sales for that month, and files a claim following the procedures underlined in
this section for the refund of the amount pertaining to Section 17 shall be entitled to a refund of
the remaining excess after offsetting any outstanding amount.
ii. As per section 24(3), a registered person may file a claim to a tax officer for a lump sum refund,
as prescribed, of the amount of the remaining excess after offsetting for a continuous period of six
months under this section.
iii. As per Section 25(ka), VAT amount shall be refunded to tourists returning from airway and
carrying items purchased of more than Rs 25,000 with them as directed by IRD. Three percent of
such refundable amount shall be deducted as service charge.
iv. As per Section 25(1) and Section 25(1)(ka), tax amount paid in purchases in Nepal by a diplomat
to Nepal of a foreign country if the foreign country grants, on reciprocal basis, the tax exemption
privileges to Nepalese diplomats to that country shall be refunded. However, tax on purchase
below Rs 5,000 shall not be refunded.

d) As per section 29(kha) of the Act,


(1) if any entity fails to comply with the VAT Act then the person acting as the officer at that period
of that time shall be held responsible.
(2) If any entity didn‟t pay VAT within stipulated date, existing officers or officers within previous
six month shall be liable jointly or separately.
(3) The above clause are only applicable on following circumstances;
- If the company has acted against the knowledge or consent of the officer.
- If the officer has maintained caution, hard work and skills like any other simple human being
would have in similar circumstances.
[Clarification: Officers mentioned in this section refer to the managers of the company or any
person acting as managers.]

RTP-CAP II –-2014-December@ICAN Page 157 of 158


RTP-CAP II –-2014-December@ICAN Page 158 of 158

Potrebbero piacerti anche